Public (SQE) Flashcards

1
Q

Which one of the following statements is incorrect?

The judiciary is responsible for the adjudication of legal disputes and the enforcement of the law.

The executive is responsible for the running of government and the creation and implementation of policy.

The legislature is the supreme law-making body in the UK.

The judiciary is responsible for how statute is interpreted.

The executive is responsible for proposing and approving all legislation.

A

The executive is responsible for proposing and approving all legislation.

Correct
Correct. This statement is incorrect. The executive does propose and ‘sponsor’ most legislation but the approval of all primary legislation is a matter for the legislature, i.e. Parliament.

How well did you know this?
1
Not at all
2
3
4
5
Perfectly
2
Q

Which one of the following statements best reflects the make-up of the constitutional rules governing the UK?

The UK’s constitutional rules are dispersed but can all be found over the pages of historic and modern statutes.

The UK’s constitution is a product of the common law.

The UK’s constitution is seen to be ‘flexible’ as it is overwhelmingly governed by conventions.

The UK’s constitutional rules very largely derive from custom and practice.

The UK’s set of governing rules can be found in a variety of legal and non-legal sources, the latter being made up primarily of ‘constitutional conventions’.

A

The UK’s set of governing rules can be found in a variety of legal and non-legal sources, the latter being made up primarily of ‘constitutional conventions’.

Correct
Correct.

How well did you know this?
1
Not at all
2
3
4
5
Perfectly
3
Q

Which one of the following five statements is false?

The UK constitution is significantly reliant on conventions.

The UK constitution has been designed specifically to comply with the principle of separation of powers.

The UK constitution is generally seen to be a ‘flexible’ one.

The UK constitution is unentrenched.

A notable proportion of the rules in the UK constitution are ‘unwritten’.

A

The UK constitution has been designed specifically to comply with the principle of separation of powers.

Correct. The statement is false. There has been no formal ‘design’ to the UK constitution – it has largely evolved over the period since 1688. Also, the UK does not conform to any strict definition of the separation of powers principle.

How well did you know this?
1
Not at all
2
3
4
5
Perfectly
4
Q

Which one of the following statements is false?

Conventions impose rigorous standards of behaviour in public life that must be followed.

Conventions modify the effect of some strict legal constitutional rules.

Conventions fill in the gaps of the UK’s sparse legal constitutional framework.

Conventions operate to create standards of accountability within government.

Conventions regulate the relationship between different bodies of state and those within the different parts of Parliament.

A

Conventions impose rigorous standards of behaviour in public life that must be followed.

Correct. The statement is false. They are intended to create standards by which those in public life ought to conduct themselves. They should impose a feeling of obligation but, very significantly, they are not legally enforceable.

How well did you know this?
1
Not at all
2
3
4
5
Perfectly
5
Q

Which one of the following cannot be regarded as a constitutional convention?

The Westminster Parliament will not normally legislate on matters affecting areas of power which have been devolved to the Scottish Parliament without prior consent from Edinburgh.

The Prime Minister and Chancellor of the Exchequer should sit in the House of Commons.

The House of Lords should respect manifesto commitments made by the governing party in power in the Commons and so not reject any legislation enforcing these.

The Chancellor of the Exchequer carries a ‘Budget Box’ (a leather dispatch box) to each Budget speech.

The monarch will not exercise her strict legal right to refuse to sign a bill of Parliament that has passed both Commons and Lords.

A

The Chancellor of the Exchequer carries a ‘Budget Box’ (a leather dispatch box) to each Budget speech.

Correct. This is more of a tradition or piece of ceremonial than a convention. It has no real constitutional significance and therefore cannot be seen to have a purpose or rationale behind it, even if all Chancellors follow this practice.

How well did you know this?
1
Not at all
2
3
4
5
Perfectly
6
Q

Which one of the following statements is incorrect?

The judiciary should not be politically active.

Government ministers should not criticise judgments made by the courts.

Law prevails over conventions.

In R (Miller) v Secretary of State for Exiting the EU (2017) the Supreme Court upheld the Scottish Government’s challenge to the legality of triggering Article 50 (and therefore the process or leaving) on the basis that the Sewel Convention was enforceable.

In R (Miller) v Secretary of State for Exiting the EU (2017) the Supreme Court recognised section 28(8) of the Scotland Act as an acknowledgment of the Sewel Convention only.

A

In R (Miller) v Secretary of State for Exiting the EU (2017) the Supreme Court upheld the Scottish Government’s challenge to the legality of triggering Article 50 (and therefore the process or leaving) on the basis that the Sewel Convention was enforceable.

Correct. This is not an accurate summary of this point – the Supreme Court was not prepared to depart from the well-established position that conventions are unenforceable, even if they are acknowledged in legislation.

How well did you know this?
1
Not at all
2
3
4
5
Perfectly
7
Q

Which one of the following statements is not a key function of the UK Parliament?

Scrutinise public expenditure and taxation.

Determine sentences in relation to the most serious criminal offences.

Propose amendments to legislation.

Extract information from the executive and hold it to account on its policies and actions.

Debate and scrutinise proposed legislation.

A

Determine sentences in relation to the most serious criminal offences.

Correct. This is not a function of Parliament: all criminal sentencing is done by the judiciary.

How well did you know this?
1
Not at all
2
3
4
5
Perfectly
8
Q

Which one of the following statements is correct?

The powers of government departments exclusively originate from the Royal Prerogative.

Secretaries of State are senior government ministers who have the right to sit in Cabinet and who head their respective departments of state.

The exercise of prerogative powers must be personally approved by the monarch before the executive can make use of them.

When considering the role and actions of the executive, this refers to central government only, in the form of departments run by Secretaries of State.

If the government acts in excess of its powers, the mechanism for challenging government action is by joint action by the House of Commons and the House of Lords.

A

Secretaries of State are senior government ministers who have the right to sit in Cabinet and who head their respective departments of state.

Correct.

How well did you know this?
1
Not at all
2
3
4
5
Perfectly
9
Q

Whichone of the following statements is incorrect?

Prior to the CRA the Lord Chancellor had a dual role, as the effective ‘Speaker’ of the House of Lords and as a sitting judge and head of the judiciary.

The CRA transferred judicial functions from the Lord Chancellor to the Lord Chief Justice.

The judiciary had been an independent force in the constitution long before the reforms introduced by the CRA 2005.

The CRA brought about the creation of the UK Supreme Court.

The CRA introduced a new method of judicial appointment based on an independent Appointments Commission.

A

Prior to the CRA the Lord Chancellor had a dual role, as the effective ‘Speaker’ of the House of Lords and as a sitting judge and head of the judiciary.

Correct. This statement is not fully accurate, as the old Lord Chancellors also had a very significant role as a government minister in relation to the administration of justice.

incorrect
The judiciary had been an independent force in the constitution long before the reforms introduced by the CRA 2005.

Incorrect. This is an accurate description. The CRA had the effect of formally separating the highest court in the land from the legislature (i.e. the House of Lords as the second chamber of Parliament), but it is wrong to draw from this that the UK judiciary was not independent until that time. It was effectively independent from the rest of the state as long ago as 1701 when the Act of Settlement confirmed security of tenure for all judges, meaning that they could not be dismissed by the government.

How well did you know this?
1
Not at all
2
3
4
5
Perfectly
10
Q

Which one of the following statements is false?

The United Kingdom’s constitution is most accurately described as uncodified.

A constitution is designed primarily to legitimise the actions and power of the Executive.

The separation of powers is an important principle that informs the relationship and balance of power between the bodies of state.

Constitutions are designed to regulate the functions and powers of the key bodies of state.

Constitutions define and regulate the relationships between the state and its citizens.

A

A constitution is designed primarily to legitimise the actions and power of the Executive.

Correct. The statement is false. A constitution should have a wider function than this, including to determine the limits on executive power and to lay out the rights and responsibilities of citizens.

How well did you know this?
1
Not at all
2
3
4
5
Perfectly
11
Q

Which one of the following legal documents or developments has not played a significant role over the span of UK history in placing limits on monarchical power?

The Case of Proclamations (1610)

The Act of Settlement (1700)

The Bill of Rights (1689)

The Domesday Book (1086)

The Magna Carta (1215)

A

The Domesday Book (1086)

Correct. This was a detailed survey of England and some parts of Wales, the purpose of which was to see how much money could be raised in taxes.

How well did you know this?
1
Not at all
2
3
4
5
Perfectly
12
Q

Whichone of the following statements is incorrect?

The judiciary will naturally be wary about crossing notional constitutional boundaries if the matter before a court is one affecting a societal issue over which there is not a clear or established consensus.

The judiciary in the UK has two main constitutional functions, namely the interpretation of statute and the development of the common law.

The modern judiciary in the UK invariably applies an activist approach to its adjudicative role.

The judiciary will be far less inclined to develop a common law position if there is an absence of fundamental established principle already in place.

The judiciary has significant power to shape the law, but this is subject to an important constitutional safeguard resting in Parliament’s hands.

A

The modern judiciary in the UK invariably applies an activist approach to its adjudicative role.

Correct. Whilst it could well be said that today’s judiciary is generally more activist than its predecessors thirty or more years ago, this is not an accurate generalisation. Courts will largely tailor their approach depending on the nature and subject matter of the issue before them. And it should be borne in mind that “activism” isn’t a measurable or objective constant in any case; it tends to be used as a term of criticism.

How well did you know this?
1
Not at all
2
3
4
5
Perfectly
13
Q

Which one of the following statements is incorrect?

The balance between the three bodies of state in the UK is an informal one, largely the product of constitutional development and political initiatives over a long historic period.

There is a sufficient separation of powers balance in the UK to maintain its status as a constitutional state.

The legislative role of the executive in the UK is purely to suggest and propose new legislation; it has no law-making powers of its own.

Separation of powers is not an absolute concept – there are notably different forms or degrees of attachment to the theory in different democratic and constitutional states.

A well-ordered and balanced state is generally viewed as one in which there are three bodies of state with their own distinct functions and different sets of personnel.

A

The legislative role of the executive in the UK is purely to suggest and propose new legislation; it has no law-making powers of its own.

Correct. This is not a fully accurate statement, as the executive is able to draft secondary legislation. This means that it does have a form of law-creating authority; one which has become more significant too in recent decades.

How well did you know this?
1
Not at all
2
3
4
5
Perfectly
14
Q

Which one of the following statements is correct?

In the Gillick case and that of R v R, the courts clearly showed that they view the common law as a living body of principles, thereby allowing them effectively to modernise the law in responding to new societal standards.

In the case of Evans, the Supreme Court established that the Attorney General was not entitled to impose the certificate, vetoing the disclosure of Prince Charles’ letters, because his position as the government’s chief legal advisor meant that this offended the principle of separation of powers.

The case of Shaw v DPP has been widely criticised because Mr Shaw was seen to have been misled in advance by incorrect advice about the lawfulness of his plans.

In the Bland case the law lords effectively transferred the responsibility for making a decision on the family’s application to Parliament.

In the case of Evans, the Supreme Court found against the Attorney General on the basis that he had not proved his case for non-disclosure of Prince Charles’ letters to a sufficient degree before the Upper Tribunal.

A

In the Gillick case and that of R v R, the courts clearly showed that they view the common law as a living body of principles, thereby allowing them effectively to modernise the law in responding to new societal standards.

Correct. ​ In a number of the speeches in both cases the judges clearly felt that the apparent older common law positions had become deeply anachronistic. Significantly too, the decisions show that the judges considered that they were entitled to remedy these situations.

How well did you know this?
1
Not at all
2
3
4
5
Perfectly
15
Q

Which one of the following statements is incorrect?

As it is Parliament which is responsible for primary legislation, this means that the executive does not have any direct law-making function - Parliament holds sole control over this element of our constitution.

It has been argued by commentators that governments with sizeable Commons majorities can be seen as a weaker element in the UK’s constitutional framework.

The role of the executive is diverse, but it is fundamentally responsible for the administration of policy and the exercise of legal powers to achieve this.

Government ministers do not have to be Members of Parliament but, by convention, they usually are.

There is a significant amount of overlap between the executive and Parliament, especially in terms of their membership.

A

As it is Parliament which is responsible for primary legislation, this means that the executive does not have any direct law-making function - Parliament holds sole control over this element of our constitution.

Correct. This statement is incorrect: the executive carries out legislative functions (albeit under the authority of primary legislation which is passed by Acts of Parliament) by the creation of secondary legislation in the form of rules, orders and regulations, which largely take the form of statutory instruments.

How well did you know this?
1
Not at all
2
3
4
5
Perfectly
16
Q

Which one of the following statements is correct?

The Home Secretary fulfils a quasi-judicial role in being able to set sentences for certain special categories of prisoners.

The Lord Chancellor’s office remains the most prestigious one in the House of Lords.

The Lord Chancellor, as the government’s chief legal adviser, continues to play an overlapping constitutional role.

The tribunal system remains an area in which the constitutionally damaging practice persists of government departments effectively overseeing the legality of some of their own practices.

The Attorney General’s role remains a controversial one in relation to the principle of separation of powers.

A

The Attorney General’s role remains a controversial one in relation to the principle of separation of powers.

Correct
Correct. ​ This is a commonly held conviction, caused by the AG’s joint role as both the government’s main legal adviser and as a member of the central executive. In the last two decades, the suspicion that Attorney Generals can be pressured politically into supporting a central government line has been directed particularly at Lord Goldsmith (in relation to the legality of the war in Iraq in 2003) and to Geoffrey Cox (in relation to Brexit, in 2019.)

How well did you know this?
1
Not at all
2
3
4
5
Perfectly
17
Q

Which one of the following statements is incorrect?

Secondary legislation often takes the form of statutory instruments.

The affirmative resolution procedure represents the weakest form of scrutiny of secondary legislation available in Parliament.

Secondary legislation can also be referred to as delegated or subordinate legislation.

Several thousand statutory instruments are made by the government each year.

If the government does not have the power from a primary Act of Parliament to make secondary legislation, it can be said to have acted ultra vires and therefore unlawfully.

A

The affirmative resolution procedure represents the weakest form of scrutiny of secondary legislation available in Parliament.

Correct. This gives a misleading picture – this procedure does not offer particularly high levels of scrutiny, but it does involve the active scrutiny by MPs and Lords of such legislation, in contrast to the looser negative procedure.

How well did you know this?
1
Not at all
2
3
4
5
Perfectly
18
Q

Which one of the following statements is incorrect?

The phrase used in the Act of Settlement, that judges could retain office ‘during good behaviour’, was so important because it removed the threat of direct political interference in the judicial role.

The proceedings of Parliament cannot form the basis of a case in the independent courts.

The judiciary are protected from being sued in the civil courts even if they have failed to show a complete knowledge of the law.

Security of tenure for judges was not formally established in the UK until the Supreme Court Act of 1981 provided for this in terms.

The changes in the appointment system for judges has arguably enhanced the accessibility of the profession and the independence of the judiciary from any form of political influence.

A

Security of tenure for judges was not formally established in the UK until the Supreme Court Act of 1981 provided for this in terms.

Correct. This is not accurate, as this vital safeguard for judicial independence was enacted shortly after the Glorious Revolution through the Act of Settlement of 1701.

How well did you know this?
1
Not at all
2
3
4
5
Perfectly
19
Q

Which one of the following statements is correct?

Judicial review is a mechanism that has been developed specifically to deal with problems caused by the incompatibility or inconsistency of UK law with both European Union and European Convention on Human Rights standards.

The process of judicial review effectively gives an aggrieved party an additional opportunity, as long as certain criteria are fulfilled, of appealing against factual findings by a lower court.

All new legislation has to be formally reviewed by senior members of the judiciary under the HRA 1998 to check if it is compatible with the European Convention on Human Rights (‘ECHR’).

The Administrative Court and the higher appeal courts are able to quash secondary legislation if it does not have the necessary legal authority.

Judicial review is an important process designed to ensure that the decisions of the judiciary are themselves subject to scrutiny in order to ensure the best standards of justice are upheld.

A

The Administrative Court and the higher appeal courts are able to quash secondary legislation if it does not have the necessary legal authority.

Correct. ​ This is correct – this is one of the measures that can be taken, where appropriate, by the courts through the judicial review process.

How well did you know this?
1
Not at all
2
3
4
5
Perfectly
20
Q

Which one of the following statements is incorrect?

Devolution as a policy was designed to reflect the desire of people in the constituent parts of the United Kingdom to be governed by regional/national organisations, and to promote efficiency in government by decentralising it so that it is closer and more responsive to regional needs.

All three of the devolved ‘nations’ within the UK have their own legislative bodies and executive bodies.

‘Reserved matters’ are those which remain within the legislative competence of the UK Parliament.

The programme of devolution introduced since 1998 means that the UK can no longer be classified as a unitary state.

The devolution model in the UK has given the three Parliaments/Assemblies power to pass their own statutes but has not prevented the Westminster Parliament from legislating for all parts of the UK on any matter, including those involving devolved powers.

A

The programme of devolution introduced since 1998 means that the UK can no longer be classified as a unitary state.

Correct. This is inaccurate, as it is over-stating the position. Devolution has undoubtedly led to greater decentralisation of power away from Westminster, but the UK remains a unitary state as ultimate sovereignty remains at the centre in Westminster.

How well did you know this?
1
Not at all
2
3
4
5
Perfectly
21
Q

Whichone of the following statements is incorrect?

If the Westminster Parliament wishes to legislate on a matter which touches on areas within the legislative competence of a devolved Parliament or Assembly, it should firstly obtain the prior approval of the relevant legislature.

If the Westminster Parliament wishes to legislate on a matter which touches on areas within the legislative competence of a devolved Parliament or Assembly, it must firstly obtain the prior approval of the relevant legislature.

The Sewel Convention not only applies to the relationship between Westminster and Edinburgh but also to the relationship between Westminster and the other devolved legislatures of the United Kingdom.

Under the Welsh devolution model, the Assembly has competence over all matters not reserved to the UK as outlined in Schedule 7A of the amended Government of Wales Act (GOWA) 2006.

Section 28(7) of the Scotland Act 1998 is the key provision of that Act, preserving the power of the UK Parliament to legislate on any matter affecting Scotland.

A

If the Westminster Parliament wishes to legislate on a matter which touches on areas within the legislative competence of a devolved Parliament or Assembly, it must firstly obtain the prior approval of the relevant legislature.

Correct. This is not entirely accurate, based on the word “must”. This is because this ‘rule’ is only a conventional one and therefore not legally binding.

How well did you know this?
1
Not at all
2
3
4
5
Perfectly
22
Q

Which one of the following statements is correct?

If a devolved legislature passes a statute, which is later deemed to fall within the relevant list of ‘reserved powers’, the court will determine that this statute does not have legal effect.

Jurisdiction over any disputes relating to the extent of devolved powers rests with the Privy Council in London.

Any decisions relating to the extent and scope of devolved powers are determined in the highest appeal court in the relevant part of the UK.

The position of Scotland in the UK is unique, as it is the only constituent part with a different legal system and jurisdiction.

In the case of R (Miller) v SoS for Exiting the EU (2017) the Supreme Court accepted the argument of the Scottish devolved administration that its consent was legally necessary before the UK triggered the Article 50 mechanism leading to an exit from the EU.

A

If a devolved legislature passes a statute, which is later deemed to fall within the relevant list of ‘reserved powers’, the court will determine that this statute does not have legal effect.

Correct. ​ This comes within the jurisdiction that the UK Supreme Court has to determine such disputes over the extent of devolved powers.

How well did you know this?
1
Not at all
2
3
4
5
Perfectly
23
Q

Which one of the following statements is incorrect?

Direct reference to the rule of law as a constitutional principle in the first section of the Constitutional Reform Act 2005 does not mean that this creates a statutory standard or defined requirement that the UK state has to live up to.

It is necessary according to all conceptions of what a ‘rule of law’ state should be for that society to be governed by liberal, democratic principles with a strong degree of protection for human rights.

It is desirable in rule of law theory for legislation to be framed as clearly and as narrowly as possible, but the realities of modern public administration have meant that an increasing amount of legislation has been produced which gives significant discretionary powers to government.

Equality before the law is a vital feature of the principle of the rule of law although it is subject in reality and practice to some marginal, though still noteworthy exceptions.

The War Damage Act 1965 was an entirely valid piece of legislation. However, it was a notable example of government and Parliament overriding a significant feature of the principle of the rule of law, as it had retrospective application.

A

It is necessary according to all conceptions of what a ‘rule of law’ state should be for that society to be governed by liberal, democratic principles with a strong degree of protection for human rights.

Correct
Correct. This is not a fully accurate statement, as there are some formalist (or procedural) conceptions of the rule of law that do not prescribe any particular substantive features or qualities for the law, merely that there is a functioning and independent legal system administering law that is clear and certain.

How well did you know this?
1
Not at all
2
3
4
5
Perfectly
24
Q

Which one of the following statements is correct?

Equality before the law is only a minor and very modern feature of the rule of law.

Equality before the law is a characteristic only of the so-called “substantive” school of thought on the rule of law.

The principle of parliamentary privilege is not completely consistent with the ideal and practice of equality before the law.

The UK can describe itself as a “rule of law” state because the law is equally applicable to all citizens in the state.

The rule of law is undermined because judges of all ranks are immune from prosecution and civil liability.

A

The principle of parliamentary privilege is not completely consistent with the ideal and practice of equality before the law.

Correct
Correct. This statement is accurate, as the inability to sue MPs for defamation for things they say in Parliament, for instance, means that the general law does not apply entirely equally to all citizens. (Note that there may be very good reasons for why parliamentary privilege is valuable for other reasons.)

How well did you know this?
1
Not at all
2
3
4
5
Perfectly
25
Q

Whichone of the following statements is incorrect?

The legality principle acts as a statutory presumption that seeks to safeguard fundamental constitutional rights and freedoms.

With the exception of prerogative powers, governmental bodies do not hold powers in their own right; they are able to exercise powers only because Parliament has directly bestowed them.

The ‘legality principle’ means that courts are bound to uphold fundamental rights in all conceivable situations, as the judiciary perceive this to be their central constitutional role.

In its purest form the rule of law dictates that governmental bodies must act within the limits of the powers that they have been given.

The legality of governmental actions is challengeable through the courts using the process of judicial review.

A

The ‘legality principle’ means that courts are bound to uphold fundamental rights in all conceivable situations, as the judiciary perceive this to be their central constitutional role.

Correct. This is not entirely accurate as the courts will not feel able to protect such rights in a situation where Parliament’s express intention has been made crystal clear in legislation. In the Simms case, Lord Hoffmann said that Parliament could legislate contrary to human rights (thereby exercising its sovereignty) but only if it did so using the clearest possible language.

How well did you know this?
1
Not at all
2
3
4
5
Perfectly
26
Q

Which one of the following statements is correct?

The courts are completely powerless in protecting the rule of law when they are obliged to interpret statute which appears to undermine fundamental rights.

There is a potential for friction to occur between the principles of parliamentary sovereignty and the rule of law, notably the more modern, substantive variety.

All forms of law in the UK have equal status and authority in creating rules by which citizens live.

When we refer to the rule of law, we are merely referring to the status and quality of statutory law.

The principles of parliamentary sovereignty and the rule of law sit harmoniously as the two key guiding principles of the UK constitution.

A

There is a potential for friction to occur between the principles of parliamentary sovereignty and the rule of law, notably the more modern, substantive variety.

Correct. This can occur when Parliament passes a statute which may undercut some of the features of the rule of law – for instance a discriminatory piece of legislation or one that restricts rights and freedoms.

How well did you know this?
1
Not at all
2
3
4
5
Perfectly
27
Q

Which one of the following statements is correct?

The heightened scrutiny given by judges when interpreting discretionary powers is a reflection of their view that the greater the discretion given to governmental bodies, the higher the potential for misuse of such powers.

In the Rossminster case the appeal courts were united in the view that the steps taken by the Inland Revenue against the company’s officers were unlawful because the discretionary powers given to the Revenue were not sufficiently clear and particularised.

The principle of the rule of law requires that discretionary powers are seen as inherently illegitimate by the courts.

The majority decision in the case of Liversidge v Anderson reflects current broadly held judicial views in relation to the level of judicial deference that it is appropriate to show to the government.

As long as governmental bodies act within the parameters of the powers given to them by Parliament, courts cannot scrutinise whether they have used the powers for the specific policy objective that Parliament sought to promote.

A
How well did you know this?
1
Not at all
2
3
4
5
Perfectly
28
Q

Which one of the following statements is incorrect?

The devolved parliaments and assemblies of the UK can be described as subordinate legislatures.

The devolution legislation is legally irreversible.

The Sewel Convention exercises a significant political influence on the operation of devolution but is not legally enforceable.

The devolution legislation is legally reversible, but it seems politically highly unlikely that the Scotland Act will be repealed.

The original devolution relating to Scotland, Wales and Northern Ireland was passed by Parliament in 1998-99 in the early years of the Blair government.

A

The devolution legislation is legally irreversible.

Correct
Correct. This is not an accurate picture as key provisions in all the devolution legislation – such as s. 28(7) of the Scotland Act – specifically allow Westminster to be able to legislate for the relevant devolved country.

How well did you know this?
1
Not at all
2
3
4
5
Perfectly
29
Q

Which one of the following statements is incorrect?

The Human Rights Act 1998 is no more securely in place than any other piece of legislation on the statute books.

Even though a stronger system of rights protection has been established in the UK over the last two decades, the values and mechanisms of the HRA have not been entrenched in the constitutional system.

The Human Rights Act 1998 represents a lesser threat to parliamentary sovereignty than that which was posed by the ECA 1972.

If the violation of a human right has occurred as a result of, or has been enabled by, an existing piece of domestic legislation, the courts effectively have a choice of two avenues to take.

A declaration of incompatibility is a significant power that has been given by Parliament to the judiciary but one which ultimately has more of a political effect than a legal one.

A

The Human Rights Act 1998 is no more securely in place than any other piece of legislation on the statute books.

Correct. This statement is not entirely accurate. The Act is far from being entrenched, as it is clearly subject to possible express repeal. However, it is arguably a little more securely in place than the majority of other pieces of legislation, as it is seen now as a “constitutional statute” and cannot therefore be impliedly repealed by inconsistent new legislation.

How well did you know this?
1
Not at all
2
3
4
5
Perfectly
30
Q

Whichone of the following statements is correct?

In the case of R v A (Complainant’s Sexual History, the court did not consider that the provision in the Youth Justice and Criminal Evidence Act 1999 was incompatible with the European Convention.

In the case of R v A (Complainant’s Sexual History) Lord Steyn interpreted the relevant provision in such a way as to make it compatible with article 6 of the European Convention.

In the case of A & Others the law lords did not feel that it was appropriate to make a declaration of incompatibility.

In the case of R v A (Complainant’s Sexual History), Lord Steyn made a declaration of incompatibility in relation to the provision in question in the Youth Justice and Criminal Evidence Act 1999.

The section 2 HRA requirement to take the ECHR jurisprudence into account means that the UK Supreme Court is obliged to accept all cases decided in the European Court of Human Rights (“ECtHR”).

A

In the case of R v A (Complainant’s Sexual History) Lord Steyn interpreted the relevant provision in such a way as to make it compatible with article 6 of the European Convention.

Correct. Lord Steyn used the new s. 3 powers under the HRA in exactly this way.

How well did you know this?
1
Not at all
2
3
4
5
Perfectly
31
Q

Which one of the following statements is incorrect?

Prerogative power is described as residual in nature, simply because those areas of state power that used to be governed by the prerogative have fallen into disuse as they are historically no longer relevant.

It has been the courts’ role for several hundred years to determine under common law whether the government of the day possesses a particular prerogative power or not.

The most influential definition of the nature and scope of prerogative power was provided in the late 19th century by Professor A. V. Dicey.

Prerogative powers pertain to the Crown but are nowadays mostly exercised by high-ranking members of the Executive.

Prerogative power can be distinguished from statutory power on the basis of where it originated from.

A

Prerogative power is described as residual in nature, simply because those areas of state power that used to be governed by the prerogative have fallen into disuse as they are historically no longer relevant.

Correct. This is not accurate as a general statement though it may be around the margins. Those areas of power that are no longer part of the prerogative have instead mostly come to be regulated by newer statutory powers.

How well did you know this?
1
Not at all
2
3
4
5
Perfectly
32
Q

Whichone of the following statements is incorrect?

The power of the government to negotiate treaties is not the only aspect of the broad foreign policy or foreign affairs prerogative.

The ability of the Home Secretary to give pardons to those convicted of crime is a product of the prerogative and not of statutory power.

The most significant of the prerogative powers still ‘owned’ by the Executive is that of “defence of the realm”.

The granting and revocation of passports was formerly a prerogative power but is now governed by statute.

The government and administration of the few remaining British Overseas Territories scattered around the world are still regulated by prerogative power.

A

The granting and revocation of passports was formerly a prerogative power but is now governed by statute.

Correct. The granting and revocation of passports continues to be a Ministerial prerogative power.

How well did you know this?
1
Not at all
2
3
4
5
Perfectly
33
Q

Which one of the following statements is correct?

The Crown is not bound by the terms of legislation unless the statutory provision in question expressly states that it should be, or it is deemed to be by “necessary implication”.

There are essentially two types of prerogative power – those that have been expressly approved by Parliament and those that have not.

In the modern age, in accordance with the principle of the rule of law, the Monarch no longer has any personal legal immunity.

The only way in which use of the prerogative can be controlled is through challenges in the courts.

Those few prerogative powers that are still technically exercised by the Monarch today – such as giving the royal assent to legislation – require her to exercise her own discretion free from any influence by government politicians.

A

The Crown is not bound by the terms of legislation unless the statutory provision in question expressly states that it should be, or it is deemed to be by “necessary implication”.

Correct. This remains the case – the matter is dealt with by the courts by applying a statutory presumption against the Crown being bound.

How well did you know this?
1
Not at all
2
3
4
5
Perfectly
34
Q

Which one of the following statements is incorrect?

The courts have historically been very careful in controlling the boundaries of prerogative power.

Tension between the Crown / Executive and both Parliament and the courts over the use of prerogative powers has been a feature of the British constitution for centuries.

It is important in the modern age for the use of prerogative power to be controlled because these powers have not been shaped or legitimated by Parliament.

The only effective means of controlling prerogative power is through general acceptance of a firm constitutional convention determining how the power in question should be used.

Constitutional conventions can have some influence over how prerogative powers are used as they generally tend to have the effect of modifying how the strict legal power is actually used.

A

The only effective means of controlling prerogative power is through general acceptance of a firm constitutional convention determining how the power in question should be used.

Correct
Correct. This statement does not reflect the relative importance of the courts in subjecting the government’s use of prerogative power to scrutiny through the judicial review process. Arguably, the political control mechanisms through constitutional conventions are more intangible and less clearly defined, as well as not being directly enforceable.

How well did you know this?
1
Not at all
2
3
4
5
Perfectly
35
Q

Which one of the following statements is correct?

In the case of Attorney General v De Keyser’s Hotel, the court upheld the right of the military authorities to stick to the terms of the historic power of defence of the realm.

The courts regard prerogative power and statutory power as equally enforceable sources of law, unless they both operate at the same time in relation to the same subject matter.

In the case of Attorney General v De Keyser’s Hotel, the military authorities were given free rein by the court to apply either prerogative or statutory power.

The court’s only consideration in the Attorney General v De Keyser’s Hotel case was what was in the national interest during a time of war.

The so-called De Keyser principle was very significant in its day, but it has subsequently been superseded by more modern legal developments.

A

The courts regard prerogative power and statutory power as equally enforceable sources of law, unless they both operate at the same time in relation to the same subject matter.

Correct. This is the consequence of the De Keyser principle that determines that, in this situation where a new statutory power has adopted the same ground as a historic prerogative power, the statutory power should prevail.

How well did you know this?
1
Not at all
2
3
4
5
Perfectly
36
Q

Which one of the following statements is correct?

Following the government’s defeat in the Miller v S o S for Exiting the EU case, it was obliged to have a vote in the Commons to obtain formal approval for its intended course of action in triggering Article 50.

In Miller v S o S for Exiting the EU, the challengers argued that the ‘Brexit’ process was unlawful, given the failure of the Prime Minister to apply the correct Article 50 process.

The case of Miller v S o S for Exiting the EU can be seen to fit broadly in a line from De Keyser through to Fire Brigades Union, as it confirmed that significant changes to statutory rights and, on a higher plane, to the UK’s constitutional arrangements could not be brought about solely by use of prerogative power.

In Miller v S o S for Exiting the EU, the challengers argued that the government no longer had the prerogative power to negotiate treaties involving the European Union.

In the Fire Brigades Union case, the Home Secretary’s use of prerogative power to introduce a new tariff-based compensation scheme directly overrode the existing operational scheme legislated for earlier by Parliament.

A

The case of Miller v S o S for Exiting the EU can be seen to fit broadly in a line from De Keyser through to Fire Brigades Union, as it confirmed that significant changes to statutory rights and, on a higher plane, to the UK’s constitutional arrangements could not be brought about solely by use of prerogative power.

Correct. This represents the general consensus of the majority of the Supreme Court justices.

How well did you know this?
1
Not at all
2
3
4
5
Perfectly
37
Q

Which one of the following statements is incorrect?

In GCHQ the court effectively said that, in relation to the reviewability of how prerogative powers were used, the ultimate issue was the nature of the power concerned and not its source.

Until the GCHQ judgment it had not been categorically confirmed that the courts could review the manner in which the executive used its prerogative powers.

The law lords decided in GCHQ that there was no reason in principle why the courts should not be able to judicially review how the executive used its prerogative powers.

The issue at hand in the GCHQ case was whether the Minister for the Civil Service had the powers she claimed to be able to regulate the working conditions of the civil servants.

Underlying the law lords’ view in GCHQ that prerogative powers should be reviewable in principle was the fact that statutory powers had been reviewable for many years.

A

The issue at hand in the GCHQ case was whether the Minister for the Civil Service had the powers she claimed to be able to regulate the working conditions of the civil servants.

Correct. This is not an accurate picture of the case. There was no doubt that the Minister (also the Prime Minister) had the necessary powers – the key point of contention was how she had used them.

How well did you know this?
1
Not at all
2
3
4
5
Perfectly
38
Q

Which one of the following statements is incorrect?

The basis of the challenge made by the civil service trade union in the GCHQ case was that the Prime Minister had banned their union membership.

The way in which the Prime Minister had banned trade union membership was seen to have been unfair.

The basis of the challenge made by the civil service trade union in the GCHQ case was that the Prime Minister had imposed a ban on their trade union membership without any prior consultation.

The prerogative power on which the PM was relying in GCHQ was that relating to the regulation of the terms and conditions of civil service ‘employment’.

The ban on trade union membership was imposed on civil servants specifically working in sensitive intelligence work at the government listening centre in Cheltenham.

A

Control of the prerogative — GCHQ

How well did you know this?
1
Not at all
2
3
4
5
Perfectly
39
Q

Which one of the following statements is correct?

Following the main ratio in GCHQ, the law lords confirmed the new orthodoxy that all forms of prerogative power were now justiciable in the courts.

The ratio of GCHQ meant that the Prime Minister lost the case overall.

Even though the court found against the Prime Minister on the main issue of principle in the case, she was nevertheless still able to defend the claim successfully.

In the obiter comments following the main ratio in GCHQ Lord Roskill gave the view that the courts should remain wary about reviewing use of prerogative powers if these were the monarch’s personal powers rather than executive prerogative powers.

In their obiter comments following the main ratio in GCHQ the law lords gave their view that the courts should no longer determine justiciability based on the nature of the power in question but on its source.

A

Even though the court found against the Prime Minister on the main issue of principle in the case, she was nevertheless still able to defend the claim successfully.

Correct. This is an accurate summary of the “result” of the case. this was because the law lords accepted the PM’s argument that there had been a justifiable public interest factor in not consulting the unions before the ban was imposed.

How well did you know this?
1
Not at all
2
3
4
5
Perfectly
40
Q

Which one of the following statements is incorrect?

The challenger in R v SoS for Foreign Affairs ex p Everett lost his case because the court respected the authority of Lord Roskill’s finding in GCHQ that matters relating to foreign affairs and diplomacy were non-justiciable.

In the case of R v Home Secretary ex p Bentley, Derek Bentley’s family was attempting to challenge the Home Secretary’s decision not to grant Derek Bentley a posthumous pardon.

In R v SoS for Foreign Affairs ex p Everett the court looked to the nature of the power that the Foreign Office had used when refusing the challenger’s passport rather than its origin or source.

In R v Home Secretary ex p Bentley the court differentiated between the granting of full pardons and consideration of other questions falling within the overall prerogative power of mercy, such as the availability of alternative though less substantial remedies such as conditional pardons.

The approach taken by the courts in both the Everett and Bentley cases illustrated a significant progression in judicial policy since the discussion of the justiciability of certain prerogative powers by Lord Roskill in the GCHQ case.

A

The challenger in R v SoS for Foreign Affairs ex p Everett lost his case because the court respected the authority of Lord Roskill’s finding in GCHQ that matters relating to foreign affairs and diplomacy were non-justiciable.

Correct. This statement is inaccurate in two respects. Firstly, it has now been widely accepted that Lord Roskill’s list was part of an obiter discussion and so does not represent an authority that has to be followed. More directly, however, the court in Everett did not conclude that the issue – relating to the refusal to renew his passport – was non-justiciable.

How well did you know this?
1
Not at all
2
3
4
5
Perfectly
41
Q

Whichone of the following statements is incorrect?

In the Abbasi case the detainee’s family did have a legal interest, in addition to their obvious personal concern, in the form of a legitimate expectation of diplomatic assistance.

The difference in the courts’ approach to the Everett andAbbasi cases relates to the different sources of power that the government was using in each.

In general terms the case of Abbasi illustrates how the modern courts will seek to enable challengers to contest important government decisions through the JR mechanism but that they will vary the intensity of review according to the nature of the issue before them.

The challengers won the case of R v SoS for Foreign Affairs ex p Abbasi, because they were able to show that the UK Foreign Office had conducted its negotiations with the United States in an unlawful and negligent way.

In the Abbasi case, the UK government was challenged for allowing the suspect to be tried under international law following his transfer to US custody.

A

The challengers won the case of R v SoS for Foreign Affairs ex p Abbasi, because they were able to show that the UK Foreign Office had conducted its negotiations with the United States in an unlawful and negligent way.

Correct. This is not an accurate summary of the nature or result of the case.

How well did you know this?
1
Not at all
2
3
4
5
Perfectly
42
Q

Which one of the following statements is correct?

In the case of Smith, Allbutt and Ellis v Ministry of Defence, the Supreme Court struck out the challengers’ cases, as it considered it vital to avoid judicialising warfare and undermining the principle of combat immunity.

In the case of R (Campaign for Nuclear Disarmament) v Prime Minister of the UK, the court found that a challenge to the lawfulness of the decision to take military action in Iraq in 2003 was a non-justiciable matter.

The House of Lords found against the challengers in R v Ministry of Defence ex p Smith and Others because the judges considered that the appropriate intensity of review to be applied in the context of the case was a very low one.

Any case involving consideration of military policy or operation will still be seen to be non-justiciable, even in the modern courts.

The case of R v Ministry of Defence ex p Smith involved a challenge to the policy of banning homosexuals from service in the armed forces based on a purported legitimate expectation that all serving personnel had a right to remain members of the armed services in the absence of any disciplinary findings made against them.

A

Control of the prerogative — post GCHQ

How well did you know this?
1
Not at all
2
3
4
5
Perfectly
43
Q

Your client wants to challenge a decision of the (fictitious) English Croquet Council (the “Council”) to ban him from playing croquet for six months due to unacceptable behaviour. The Council was set up by a group of local croquet clubs around fifty years ago in order to regulate croquet. It governs the rules of the game; the codes of conduct under which your client received a sanction; and the relationships between local clubs and itself. All Council members, including your client, agree to be subject to its rules. It is the sole regulator of croquet in the UK.

Which of the following summarises the most accurate advice to your client on whether the Council’s decision is amenable to judicial review?

Select one alternative:

The Council’s decision is not amenable to judicial review because the Council is a private body.

The Council’s decision is amenable to judicial review because its functions and powers are public in nature and, if it did not exist, the government would need to step in to regulate croquet.

The Council’s decision is not amenable to judicial review because the Council’s functions have not been contracted to it by a public body.

It is likely that the Council’s decision will be amenable to judicial review because its powers associated with the regulation of a sporting activity are governmental in nature.

It is unlikely that the Council’s decision is amenable to judicial review. On the facts, the rules affecting the client come from an agreement between clubs, their members and the Council, giving rise to private rights and remedies.

A

It is unlikely that the Council’s decision is amenable to judicial review. On the facts, the rules affecting the client come from an agreement between clubs, their members and the Council, giving rise to private rights and remedies.

This question requires knowledge about the type of decisions that can be challenged in judicial review, known as their amenability to judicial review. These are not simply decisions taken by public bodies. They can also include decisions taken by ostensibly private organisations if the decision involves matters that are of a sufficiently public nature. However, in this situation, involving a sporting association, it is unlikely that the decision taken will be seen to involve a public function. Also, it is clear that the clubs and members have agreed to abide by the organisation’s own rules. It is therefore unlikely that this matter will be challengeable in public law through JR.

How well did you know this?
1
Not at all
2
3
4
5
Perfectly
44
Q

Your client wants to challenge a decision of the local District Council to refuse his application for a music venue licence for his café on the High Street. The scheme, allowing local authorities to issue licences, is governed by a provision in an Act, which states:

“In deciding whether to issue a music licence, local authorities must take into account the interests and opinions of residents and businesses who are situated close to the venue.”

The client had submitted several letters of support with his application from local businesses, residents living in and around the town centre, and musical performers. These letters were not mentioned by the District Council in its refusal letter.

On the basis of the information you have, which one of the following summarises the most accurate advice to your client on whether and how he can challenge the decision on the judicial review ground of illegality?

Select one alternative:

It is likely that the District Council acted unlawfully in the sense that its decision was ultra vires its powers.

The District Council’s decision suggests that it failed to take into account a mandatory consideration when assessing your client’s application and so acted unlawfully.

The District Council is not likely to have acted unlawfully in this situation as the Administrative Court does not have the right to scrutinise how it conducted the application process.

It is likely that the District Council has fettered its discretion because it failed to take into account the letters of support submitted by your client, and so acted unlawfully.

Your client will have strong grounds to argue that the District Council made an error of law in relation to a misunderstanding of the wording in the relevant provision.

A

The District Council’s decision suggests that it failed to take into account a mandatory consideration when assessing your client’s application and so acted unlawfully.

This question requires knowledge of the different types of challenge that can be made under the judicial review ground of illegality. One of the key sub-categories involves the duty on bodies making public law decisions to take into account relevant considerations, and conversely to ignore irrelevant ones. This local authority does not appear to have followed a duty in the statute, which is phrased in mandatory terms, and so has acted unlawfully.

How well did you know this?
1
Not at all
2
3
4
5
Perfectly
45
Q

Your client is the managing director (“MD”) of a construction company which submitted an unsuccessful bid to build a new library for a local authority. The MD is very disappointed because his company had far greater experience of these kinds of projects than the only other competitor company. He has also discovered that his costing was lower than the competitor’s.

A few weeks after the decision was made, the MD was surprised to hear that the chair of the competitor company was a long-standing friend of one of the three members of the local authority committee which made the decision.

On the basis of the information you have, which of the following statements summarises the most accurate advice on your client’s prospects of challenging the local authority’s decision on the ground of procedural impropriety?

Select one alternative:

Your client has a potentially strong claim based upon direct bias.

Your client has a potential indirect bias claim, but it will not succeed because there is no evidence that the friend on the decision-making committee said or did anything that proved they were actually biased.

Your client has a potentially strong claim based upon indirect bias.

Your client has a potential indirect bias claim, but it will not succeed because only one member of the decision-making committee could have been biased.

It is unlikely that your client would succeed in a claim because it is not apparent that the member of the decision-making committee had a financial interest in the competitor company.

A

Your client has a potentially strong claim based upon indirect bias.

This question requires knowledge of how public law decisions can be challenged on the ground of bias and of the two different forms of bias. There is a clear connection here between the two individuals involved. The decision-maker does not have a direct, financial interest in the matter (or rather there is no evidence of that which has been disclosed) so this cannot be a matter of direct bias. However, the closeness of their connection gives rise, on an objective basis, of there being a real possibility of bias which is the test for indirect bias. It does not matter that the individual was one of a three-person panel – that is enough to give rise to this ‘real possibility’.

How well did you know this?
1
Not at all
2
3
4
5
Perfectly
46
Q

A production company has been refused a classification certificate for a film because the national licensing authority considered the film to be blasphemous and to contain obscene material contravening various statutory provisions.

The company seeks advice in relation to a possible action under article 10 of the European Convention on Human Rights (‘ECHR’).

Which of the following statements represents the best advice to the company, relating to its freedom of expression and whether it can be interfered with in this way?

Select one alternative:

Freedom of expression can be interfered with by the licensing authority but only if the restriction on it is a proportionate one.

Freedom of expression is an absolute right and so the licensing authority’s actions represent a clear breach of article 10.

Freedom of expression only protects political and journalistic free speech and so there would be no protection under article 10 in relation to the banning of the film.

Freedom of expression can be interfered with by the licensing authority but only if the interference is designed to protect national security or to prevent violence towards others.

Freedom of expression can be interfered with by the licensing authority but only if the restriction on it is prescribed by law and fulfils a legitimate aim in a proportionate way.

A

Freedom of expression can be interfered with by the licensing authority but only if the restriction on it is prescribed by law and fulfils a legitimate aim in a proportionate way.

This question requires knowledge of the nature of article 10 ECHR and the circumstances is which it can be justifiable for the state or a public authority to interfere with it. It is important firstly to appreciate that article 10 is a qualified right and not an absolute one. And also that expression covers a very wide range of expression, not just political. It is not possible on the facts to determine if there has been a definitive breach of article 10 so the question anticipates knowledge of the three hurdles which it is necessary for the state to overcome to justify the restriction, in this case the refusal of the certificate. These are that the restriction must be ‘prescribed by law’ and applied in a proportionate way in pursuit of a legitimate aim. The list of relevant legitimate aims is contained in article 10(2) and are not confined to national security or safety.

How well did you know this?
1
Not at all
2
3
4
5
Perfectly
47
Q

It has been disclosed in the press that the Secretary of State for Health (the SoS) has been involved in heated arguments with the Chancellor of the Exchequer because substantial cuts in the NHS budget are being threatened, and several thousand nursing staff may have to be made redundant. This matter is about to come to a head at the next Cabinet meeting when the final decision has to be made.

Which of the following statements best reflects how the SoS is able to act under the relevant convention and, if a final decision is made to make the cuts, what his realistic options would be?

Select one alternative:

Under the convention of collective ministerial responsibility, the SoS is able to argue as strongly as he wishes against the cuts, as long as he does not later disclose the arguments in Cabinet externally. However, if he loses the debate in the Cabinet meeting, there will have to be a vote of no confidence in the government because of the extent of the disunity in Cabinet.

Under the convention of individual ministerial responsibility, the SoS has to argue against the cuts because they would undermine the effectiveness of the department for which he is responsible, if implemented. However, if he loses the debate, he will be obliged either to accept the cuts and support them as a member of the government or to resign his ministerial role.

The principle of collective ministerial responsibility does not apply in this situation because it is already public knowledge that there is disagreement in Cabinet and so the SoS can disclose details of the Cabinet discussions after the meeting.

Under the convention of collective ministerial responsibility, the Prime Minister is already under an obligation to sack the SoS, as the news that there is disagreement in Cabinet has leaked out.

Under the convention of collective ministerial responsibility, the SoS is able to argue as strongly as he wishes against the cuts, as long as he does not later disclose the arguments in Cabinet externally. However, if he loses the debate, he will be obliged either to accept the cuts and support them as a member of the government or to resign his ministerial role.

A

Under the convention of collective ministerial responsibility, the SoS is able to argue as strongly as he wishes against the cuts, as long as he does not later disclose the arguments in Cabinet externally. However, if he loses the debate, he will be obliged either to accept the cuts and support them as a member of the government or to resign his ministerial role.

This question requires an understanding of the nature and extent of the convention on collective (as opposed to individual) ministerial responsibility. The key to it is to appreciate, firstly, that Cabinet discussions on the issue are intended to allow ministers to exchange their views and opinions openly but, equally, to maintain confidentiality of the matters discussed. Secondly, once a communal decision has been made, it is very important – in order to achieve confidence in government – for the ministers to maintain a united front, failing which there is a moral obligation on any openly dissenting minister to resign.

How well did you know this?
1
Not at all
2
3
4
5
Perfectly
48
Q

A celebrity author of a self-help relationship book, entitled ‘How to Make Your Marriage Succeed’, has been photographed by a tabloid newspaper on a public street, kissing and holding hands with a man who is not her husband. The tabloid’s subsequent enquiries have revealed that they are having an extra-marital affair.

The author would like to issue proceedings against the tabloid newspaper for an infringement of her privacy contrary to article 8 of the European Convention on Human Rights (‘ECHR’).

Which of the following statements represents the best advice to your client and the reason why?

Select one alternative:

She would not be able to make a claim, because freedom of expression will always take precedence over the right to a private life and so the newspaper’s article 10 rights would automatically supersede hers.

It would be possible for her to make a claim but only if she had a pre-existing contractual or other relationship with the newspaper and could show that it had breached her confidence.

She would not be able to make a claim, because the Human Rights Act 1998 only provides protection for individuals against public authorities that interfere with their Convention rights and the newspaper is clearly not such an authority.

It is unlikely that she will succeed in her claim as a court will probably not consider her to have a reasonable expectation of privacy, as she had acted openly in this way in a public place. Even if it were found that she did have such an expectation, it is likely that the court would find that there was a public interest factor in the story that the newspaper could point to, in order to justify publication.

It is very likely that the court would accept that she had a reasonable expectation of privacy in this situation. However, it is also likely that the court would find that there was a public interest factor in the story that the newspaper could point to in order to justify publication, so it is unlikely she would succeed.

A

It is unlikely that she will succeed in her claim as a court will probably not consider her to have a reasonable expectation of privacy, as she had acted openly in this way in a public place. Even if it were found that she did have such an expectation, it is likely that the court would find that there was a public interest factor in the story that the newspaper could point to, in order to justify publication.

This question relates to the ‘indirect horizontal effect’ that has been brought about the Human Rights Act, notably the status of the courts as public authorities. In principle a claim would be possible in this scenario, but it is unlikely to succeed for two reasons. Firstly, an individual will only have article 8 privacy rights engaged, if they are seen to have a ‘reasonable expectation of privacy’ and that is unlikely in this public situation. Secondly, even if a court deemed there was a reasonable expectation of privacy, it is likely to apply the balance between the individual’s privacy rights and the tabloid’s article 10 rights in favour of the latter, as there appears to have been a public interest in the story.

How well did you know this?
1
Not at all
2
3
4
5
Perfectly
49
Q

A man has been charged with a serious criminal offence and is awaiting trial at the Crown Court. He is anxious about his prosecution and would like advice about the process.

Which of the following statements best describes the burden and standard of proof applicable at his trial?

Select one alternative:

It is for the defendant to show that he is not guilty. If the jury is satisfied that sufficient doubt has been cast on the prosecution’s case, it has a duty to acquit the defendant.

It is for the prosecution to prove that the defendant is guilty. If the jury is satisfied on balance that the defendant committed the offence, they should return a guilty verdict.

It is for the prosecution to prove that the defendant is guilty. If the jury is satisfied beyond reasonable doubt that the defendant committed the offence, it should return a guilty verdict.

It is for the defendant to prove that he is not guilty. If the jury is satisfied beyond reasonable doubt that the defendant has been able to do this, it can acquit him.

It is for the prosecution to prove that the defendant is guilty. If the jury considers that there is even the slightest possibility that the defendant did not commit the offence, they should acquit him.

A

It is for the prosecution to prove that the defendant is guilty. If the jury is satisfied beyond reasonable doubt that the defendant committed the offence, it should return a guilty verdict.

This question requires a knowledge of both the burden of proof operating in the criminal law and the degree of proof required. It is vital to appreciate that the burden is on the prosecution and the defendant does not have disprove his alleged guilt. The standard of proof required is beyond reasonable doubt and not ‘on balance’ or there being ‘even the slightest possibility”.

How well did you know this?
1
Not at all
2
3
4
5
Perfectly
50
Q

A civil case has reached the Court of Appeal and it has become apparent that there is a lack of clarity over relevant precedent established by an earlier Court of Appeal judgment.

In which of the following circumstances will the current Court of Appeal be able to depart from a previous Court of Appeal judgment on the same area of law?

Select one alternative:

Where the earlier Court of Appeal decision had led to significant commercial or economic inconvenience.

Where the earlier Court of Appeal decision had been made by a panel of judges without expertise on the subject area.

Where it is aware of significant critical academic reaction to the earlier Court of Appeal decision.

Where it considers that societal values have changed in the interim period and it would be justifiable to develop the law in a different direction.

Where it considers that the earlier Court of Appeal decision had been made per incuriam by failing to pay due regard to a relevant statutory or common law authority.

A

Where it considers that the earlier Court of Appeal decision had been made per incuriam by failing to pay due regard to a relevant statutory or common law authority.

This question requires knowledge of the system of precedent and, more specifically, of the flexibility that has been brought in to allow some departure from historic precedents. In relation to the Court of Appeal the current court is able in defined circumstances to depart from a decision made by an earlier incarnation of the court. This is not allowed where the modern court feels that the law should be modernised in a general sense but instead for more technical reasons. Here, an acceptable reason for doing so would be if the earlier Court of Appeal had acted per incuriam by not paying due regard to a statutory or case law authority that was relevant at the time.

How well did you know this?
1
Not at all
2
3
4
5
Perfectly
51
Q

A company director wishes to challenge the lawfulness of a statutory instrument which imposes onerous conditions on part of the company’s business, affecting its profitability.

Which of the following statements best describes whether and how the lawfulness of the statutory instrument can be challenged?

Select one alternative:

It is not possible to challenge the lawfulness of the statutory instrument, as this is a form of legislation passed by Parliament, but the Administrative Court may read in words to the provision in order to render it lawful.

A statutory instrument is a form of secondary legislation and it is possible to challenge its lawfulness in the Administrative Court. If the challenge is successful, the remedy is a declaration that the statutory instrument is incompatible with the parent Act under which it was made.

A statutory instrument is a form of secondary legislation and it is possible to challenge its lawfulness in the Administrative Court. If the challenge is successful, the Administrative Court can quash the provision in the Act under which the statutory instrument was made.

It is not possible to challenge the lawfulness of the statutory instrument as this is a form of legislation passed by Parliament and such an outcome would undermine the basic principle of parliamentary sovereignty.

A statutory instrument is a form of secondary legislation and it is possible to challenge its lawfulness in the Administrative Court. If the challenge is successful, the Administrative Court is able to quash the statutory instrument.

A

A statutory instrument is a form of secondary legislation and it is possible to challenge its lawfulness in the Administrative Court. If the challenge is successful, the Administrative Court is able to quash the statutory instrument.

This question requires knowledge that primary and secondary legislation have different statutes. A statutory instrument (SI) is a form of secondary or delegated legislation, and it is capable of challenge in court, unlike primary legislation which cannot be directly challenged, given the central importance of parliamentary sovereignty. It is also important to note that, if the Administrative Court does not feel that the SI has been made in accordance with the original ‘parent’ Act, the remedy would be for the court to quash the SI.

How well did you know this?
1
Not at all
2
3
4
5
Perfectly
52
Q

A civil case has reached the Court of Appeal and there is a clear conflict of opinion between the parties over applicable precedent on a particular point of law.

One of the parties wishes to rely on an obiter statement made by a Justice of the Supreme Court ten years previously which was part of a speech outlining the law at that stage but was not central to the ratio of that case.

Which of the following statements best describes the status of the Justice’s obiter statement in relation to the current Court of Appeal case?

Select one alternative:

The Justice’s obiter statement will only have some influence in shaping the judgement in the current case if it had been supplemented by a reasonable number of concurring opinions in the meantime.

The Justice’s obiter statement will have a binding effect in relation to the judgement of the current case as it was made by a judge in a superior court.

The Justice’s obiter statement is likely to have significant persuasive effect in relation to the current case, especially if it related to a closely connected legal issue, but it will not be binding.
Answered and correct

The Justice’s obiter statement has no application at all to the current case, as it was not central to the ratio established ten years ago and so has no precedent value.

The Justice’s obiter statement will only have minimal impact on the current case in framing the parameters of the debate between the parties.

A

The Justice’s obiter statement is likely to have significant persuasive effect in relation to the current case, especially if it related to a closely connected legal issue, but it will not be binding.

This question relates to the significance and status of obiter comments in relation to the development of the common law. It is very important here to appreciate that obiter comments cannot be directly binding on (inferior) courts, adjudicating on the same area. However, they can prove influential in shaping the decisions of future courts, especially if they relate closely to the subject matter at hand, and so they can have a persuasive effect.

How well did you know this?
1
Not at all
2
3
4
5
Perfectly
53
Q

A foreign national, who travelled to the UK to seek asylum, has had his claim rejected and arrangements for his removal have been put in place. New evidence has, however, been identified which will support an appeal against the man’s refusal of asylum.

The court has made an order that the man should not be removed from the UK until the court has given further consideration to this new evidence. The Home Office has stated that the Home Secretary is not willing to allow the man to remain in the UK and that the court has no right to prevent his removal.

Which of the following statements best reflects the constitutional legal position in this situation?

Select one alternative:

The executive branch cannot be subject to any form of control by the judiciary and therefore the removal cannot be prevented in these circumstances.

The courts will be prepared to exercise oversight and control over the executive in relation to this matter and may find the Home Secretary’s office to have been in contempt of court if the order is ignored.

The courts will be prepared to exercise oversight and control over the executive in relation to this matter and, if the order is ignored, will immediately confirm the success of the individual’s application for asylum.

The courts will be prepared to scrutinise the action by the Home Secretary but are likely to defer to the Home Secretary’s expertise in relation to cases concerning asylum and so the individual’s prospects of success in resisting removal from the country will be minimal.

The ultimate decision to grant or refuse asylum will be seen by the courts as a political matter and so, if the order is ignored, they will not be prepared to intervene any further.

A

The courts will be prepared to exercise oversight and control over the executive in relation to this matter and may find the Home Secretary’s office to have been in contempt of court if the order is ignored.

This question relates to the general area of separation of powers in the UK constitution and in particular the relationship between the executive and the judiciary. In order to ensure that the ‘rule of law’ is followed, it is important that the courts scrutinise the actions of the executive to ensure that it is acting in accordance with the law. In some situations, where the government is given discretion to act, the courts will defer to the expertise of the executive. However, that is not the issue here where the court has made a clear order which the Home Office is intent on ignoring. The court cannot override the Home Office’s decision and substitute its finding for the Home Office’s but it can find the Home Office in contempt of court.

How well did you know this?
1
Not at all
2
3
4
5
Perfectly
54
Q

You are advising an educational research centre which has applied for a grant from a national funding agency to help it employ a research assistant to analyse data from GCSE and A-level exams. The client’s application has been turned down, as it has been told that it is not a qualifying institution. The client wants to challenge the refusal of its application.

The process is governed by the following provision from [fictitious] regulations: “The agency will distribute grants to institutions undertaking research into programmes leading towards degree, Masters, post-graduate doctorate or other similar educational qualifications.”

Which of the following represents the best advice to your client?

Select one alternative:

It is unlikely that the court will quash the decision reached by the agency based on the linguistic presumption expressio unius est exclusio alterius, as the client is an institution not specifically named or classified in the provision.

It is likely that the court will quash the decision reached by the agency based on the linguistic presumption ejusdem generis, as the client’s application also relates to research into educational qualifications.

It is unlikely that the court will quash the decision reached by the agency, as the linguistic presumption ejusdem generis points towards an interpretation of qualifying institutions as those researching graduate and postgraduate qualifications and not qualifications taken at secondary school level.

The court would apply the literal rule and determine that the client is seeking to research into educational qualifications and so be eligible for a grant.

The court will consider the purpose of the regulations which is to encourage educational research and follow the purposive form of interpretation favoured by all modern courts, thereby quashing the decision of the agency.

A

It is unlikely that the court will quash the decision reached by the agency, as the linguistic presumption ejusdem generis points towards an interpretation of qualifying institutions as those researching graduate and postgraduate qualifications and not qualifications taken at secondary school level.

This question requires a knowledge of the maxims that apply to statutory presumptions that can be made by the courts when interpreting statutes with particular features. In particular, this provision contains a form of open list of ‘similar’ educational qualifications, and so it is most likely that the maxim “ejusdem generis” will be applied. As the named qualifications in the list all involve tertiary education, it seems unlikely that Parliament would have intended the client’s research – involving secondary education – to be granted funding.

How well did you know this?
1
Not at all
2
3
4
5
Perfectly
55
Q

The Secretary of State for Health has therefore announced his intention to introduce a Bill into Parliament transferring responsibility for dealing with serious public health emergencies from the Scottish Government to Westminster, even though this is currently a devolved matter in Scotland.

Which of the following statements best describes the legal consequences of this proposed change?

Select one alternative:

It will be legally possible for the UK government to carry out this change. This is because the Sewel Convention is not enforceable. Therefore, Westminster can legislate in this way even without the political approval of the Scottish Parliament.

It will be legally possible for the Scottish Parliament to resist this change but only if it passes an Act of the Scottish Parliament expressly repealing the legislation proposed in Westminster.

It will not be legally possible for the UK government to carry out this change, as the Sewel Convention is an enforceable legal document with constitutional status equal to that of the European Convention on Human Rights, meaning that the UK Parliament is prevented from legislating on devolved Scottish matters.

It will be legally possible for the UK government to carry out this change, but it will firstly have to persuade the Westminster Parliament to repeal the devolution legislation passed since 1998, thereby reverting to the pre-devolution position.

It will be legally possible for the UK government to carry out this change but only if the Westminster Parliament complies with the Sewel Convention and firstly obtains a legislative consent motion passed by the Scottish Parliament.

A

It will be legally possible for the UK government to carry out this change. This is because the Sewel Convention is not enforceable. Therefore, Westminster can legislate in this way even without the political approval of the Scottish Parliament.

This question relates to the relationship between the UK Parliament and government in Westminster and the devolved administrations, in this instance, Scotland. It is important to appreciate that the Sewel Convention would apply in this instance as the UK government is seeking to introduce legislation that cuts across a devolved area. It should therefore initially ask the Scottish Parliament to pass a legislative consent motion. However, it is equally important to know that, because this is just a convention and therefore not legally enforceable, Westminster can effectively override this and introduce the Bill in the manner proposed.

How well did you know this?
1
Not at all
2
3
4
5
Perfectly
56
Q

Which one of the following suggested definitions best describes the concept of equity as it is used in the English legal system?

A body of rules which began in the 15th century.

An area of law which provides discretionary remedies.

A body of rules and principles which began in the Court of Chancery.

A body of rules and principles designed to remedy the rigidity of the common law.

An application of the principles of fairness, designed to replace the common law.

A

A body of rules and principles designed to remedy the rigidity of the common law.

Correct. This is effectively the original purpose and function of this branch of law. The other answers do not describe the essence or purpose of the law on equity as effectively.

How well did you know this?
1
Not at all
2
3
4
5
Perfectly
57
Q

Which one of the following hypothetical situations best illustrates a judge distinguishing a precedent?

In a case currently being heard in the Supreme Court the justices consider that a precedent established by their predecessors over half a century ago in the old House of Lords should be revised, because it is no longer appropriate in today’s changed social and moral environment.

In a case currently being heard in the Supreme Court the justices have found on closer analysis that a precedent established several years ago in the Court of Appeal was based on an incomplete understanding of the material facts in that case. They therefore decide not to apply that precedent in the current case.

In a current High Court case, the judge finds that the material facts, though similar in nature to that of an old Court of Appeal case which established a precedent on a particular point of law, are sufficiently different to justify not being bound by that precedent.

In a case currently being heard in the Supreme Court the justices consider that an old precedent established by the Court of Appeal was wrongly decided, and so do not apply it to the case under consideration.

A High Court judge finds that a precedent set by the Court of Appeal in an old case does not apply to the current case under consideration.

A

In a current High Court case, the judge finds that the material facts, though similar in nature to that of an old Court of Appeal case which established a precedent on a particular point of law, are sufficiently different to justify not being bound by that precedent.

Correct. Note that the judge is the person responsible for assessing the extent and scope of the old precedent, as it should currently apply. This is very much a matter of judicial skill and the ability to vary the level of abstraction gives the judge a degree of freedom to depart from an old precedent if s/he considers it appropriate. The other answers do not show a correct or as clear an understanding of the relevant terminology used in the analysis of case law as this one does.

How well did you know this?
1
Not at all
2
3
4
5
Perfectly
58
Q

After a serious road traffic accident, the driver of the car who caused the accident faces a number of legal problems. Unfortunately, his careless driving has brought about the death of a young child and the parents wish to claim bereavement damages as well as to see the driver prosecuted.

Which of the following statements best describes the key features of the legal processes that are likely to be embarked upon?

The driver of the car will be prosecuted for the relevant offence by the Crown Prosecution Service and, if found, beyond reasonable doubt to be guilty, he will be sentenced by the court and be ordered to pay bereavement damages to the parents.

The driver of the car will be prosecuted for the relevant offence by the Crown Prosecution Service but, if he pleads guilty at an early stage in proceedings, he will have the benefit of being sentenced in a Crown Court in front of a jury. He will also face a civil law claim in relation to his negligence and, if found on the balance of probabilities to be responsible, he will liable in damages.

The driver of the car will be sued in the civil courts by the parents and, if found guilty, on the balance of probabilities, he will be liable to them for bereavement damages. He will also be prosecuted for the relevant offence by the police and, if found guilty beyond reasonable doubt, will be sentenced by the court.

The driver of the car will be prosecuted for the relevant offence by the police and, if the parents so choose, they can call for him to be tried in the Crown Court in front of a jury. The driver will also face a civil law claim in relation to his negligence and, if found on the balance of probabilities, to be responsible, he will liable in damages.

The driver of the car will be prosecuted for the relevant offence by the Crown Prosecution Service and, if found beyond reasonable doubt to be guilty, he will be sentenced by the court. He will also face a civil law claim in relation to his negligence and, if found on the balance of probabilities to be responsible, he will be liable in damages.

A

The driver of the car will be prosecuted for the relevant offence by the Crown Prosecution Service and, if found beyond reasonable doubt to be guilty, he will be sentenced by the court. He will also face a civil law claim in relation to his negligence and, if found on the balance of probabilities to be responsible, he will be liable in damages.

Correct. Note the differing standards of proof in criminal and civil law and the other terminology associated with both. The other answers do not adequately describe the civil and criminal law processes and consistently refer to the correct terminology associated with both.

How well did you know this?
1
Not at all
2
3
4
5
Perfectly
59
Q

A corporate client of a large law firm alleges that negligence by the firm, when advising on a high value and complex case, has caused it to lose more than £500,000. It wishes to sue its solicitors, therefore, to recover this loss.

Which of the following courts and judges is the client company most likely to encounter in its proposed litigation, assuming that there is one appeal after the first instance trial?

It is likely that the initial procedural hearings will be heard in the County Court by a district judge who will then send the case up to the High Court for the trial. Any subsequent appeal will then be heard in Civil Division of the Court of Appeal.

It is likely that the case will start in the Commercial Court section of the High Court. Any procedural hearings will be heard by a Master and then the first instance trial will be heard by a High Court judge, with any appeal going to the Civil Division of the Court of Appeal.

It is likely that the case will start in the High Court in the Queen’s Bench Division. Any procedural hearings will be heard by a Circuit Judge and then the first instance trial will be heard by a High Court judge, with any appeal going to the Civil Division of the Court of Appeal.

It is likely that the case will start in the County Court, where any initial procedural hearings will be heard by a District Judge. The first instance trial will be heard by a Circuit Judge, with any appeal going to the High Court.

It is likely that the case will start in the Queen’s Bench Division of the High Court. Any procedural hearings will be heard by a Master and then the first instance trial will be heard by a High Court judge, with any appeal going to the Civil Division of the Court of Appeal.

A

It is likely that the case will start in the Queen’s Bench Division of the High Court. Any procedural hearings will be heard by a Master and then the first instance trial will be heard by a High Court judge, with any appeal going to the Civil Division of the Court of Appeal.

Correct
Correct. It is most probable that the claimants will seek to issue proceedings in the High Court, given the potential value of the claim. The other answers do not accurately or as effectively describe the likely route of the litigation.

How well did you know this?
1
Not at all
2
3
4
5
Perfectly
60
Q

Which one of the following hypothetical situations best describes the most appropriate use of the leapfrog process?

A very complex clinical negligence case has been heard by the High Court. The defendant health authority, which lost the case, and which faces liability for more than £5 million in damages, wishes to dispute the legal basis on which the judgment was reached, as this involved a novel point of law and potentially has implications for numerous other health authorities. The health authority therefore applies for permission to appeal directly to the Supreme Court.

A legally complex environmental nuisance case has recently been decided in the County Court in favour of the claimants. As the case has implications for a significant number of other industrial companies, the defendant company has applied to appeal directly to the Supreme Court because its directors consider that it engages a matter of significant public interest.

A professional footballer has been convicted in the Magistrates’ Court for drink-driving. He wishes to appeal on a technical legal matter relating to the interpretation of the relevant statute governing the offence. As he is very well-known and therefore the conviction is a very serious matter in relation to his career, he has applied for his case to bypass the High Court and be heard in the Court of Appeal.

A very complex clinical negligence case has been heard by the High Court, in which the medical evidence has been strenuously contested. The defendant health authority, which lost the case, and which faces liability for more than £5 million in damages, wishes to dispute the medical evidence. As it is highly likely that the matter will be appealed again by whichever side loses the appeal, the health authority applies for the case to be heard again in the Supreme Court.

After a controversial prosecution of a well-known businessman for fraud, he is sentenced to imprisonment for 10 years by the Crown Court. He wishes to appeal against his conviction and, because it is very likely that this matter will be contested as far as possible, permission is given for the appeal to bypass the Court of Appeal and to be heard by the Supreme Court.

A

A very complex clinical negligence case has been heard by the High Court. The defendant health authority, which lost the case, and which faces liability for more than £5 million in damages, wishes to dispute the legal basis on which the judgment was reached, as this involved a novel point of law and potentially has implications for numerous other health authorities. The health authority therefore applies for permission to appeal directly to the Supreme Court.

Correct. This is the correct leapfrog process, as it bypasses the Court of Appeal. It also involves an issue of law that is a matter of public importance. The other answers do not correctly or adequately describe the process.

How well did you know this?
1
Not at all
2
3
4
5
Perfectly
61
Q

Which one of the following statements best describes the linguistic presumption that should be used in the context to aid the process of statutory interpretation?

In order to assist in a literal interpretation of a (fictitious) statute, the judge applies the presumption ‘expressio unius est exclusio alterius’ when interpreting the statutory phrase: ‘the imposition of the tax on income, share dividends and bond yields.’

In order to assist in the interpretation of a (fictitious) statute, the judge applies the ‘noscitur a sociis’ rule when construing the phrase: ‘clubs, associations and other organisations’.

In order to assist in the purposive interpretation of a (fictitious) statute, the judge applies the presumption ‘ejusdem generis’ by considering not just the provision in question but other neighbouring provisions in the statute.

In order to assist in a purposive interpretation of a (fictitious) statute, the judge applies the presumption ‘expressio unius est exclusio alterius’ when interpreting the phrase: ‘the imposition of the tax on income, share dividends and bond yields.’

In order to assist in the interpretation of a (fictitious) statute, the judge applies the presumption ‘expressio unius est exclusio alterius’ when interpreting the phrase: ‘construction of domestic houses, apartments, flats and other such dwellings’.

A

In order to assist in a literal interpretation of a (fictitious) statute, the judge applies the presumption ‘expressio unius est exclusio alterius’ when interpreting the statutory phrase: ‘the imposition of the tax on income, share dividends and bond yields.’

Correct. This effectively deals with the situation where there is a ‘closed list’ of descriptive items, as in this statute – the presumption is that the tax applies only to those matters listed. Additionally, this assists a literal approach to interpretation as the court is seeking to achieve a proper construction of the words in the statute. The other answers do not correctly or adequately describe the correct application of the various linguistic presumptions that aid the process of statutory interpretation.

How well did you know this?
1
Not at all
2
3
4
5
Perfectly
62
Q

Which one of the following statements best describes the purposive approach to statutory interpretation?

Judges will look at the strict meaning of the words contained in a statute, as well as the purpose or objective of the Act in question. If there is a conflict between the two, the judges will interpret the words in the context of and in accordance with the wider purpose of the Act.

Judges will consider the overall design and configuration of the Act in question. However, if they consider that this creates a situation where a literal interpretation of the legislation is incoherent or absurd, they will be entitled to apply a different interpretation to the words so that implementation of the Act becomes more workable and effective.

Judges will examine the social and economic context of the Act in question as a starting point in interpreting legislation.

Judges will look at the purpose of the Act in question when construing the statute.

It is possible to see the origins of the purposive approach in the older ‘mischief rule’.

A

Judges will look at the strict meaning of the words contained in a statute, as well as the purpose or objective of the Act in question. If there is a conflict between the two, the judges will interpret the words in the context of and in accordance with the wider purpose of the Act.

Correct. Judges do not automatically use a purposive approach. However, particularly in the modern day, they naturally tend to assess the overall context within which the Act came about so that, if a literal reading of the words in the statute does not produce a plausible interpretation, they will be informed by this wider context. The other answers did not fully or clearly describe the context in which, and the objective for which, the courts use a purposive approach to statutory interpretation.

incorrect
Judges will consider the overall design and configuration of the Act in question. However, if they consider that this creates a situation where a literal interpretation of the legislation is incoherent or absurd, they will be entitled to apply a different interpretation to the words so that implementation of the Act becomes more workable and effective.

Incorrect. This approach is closer to the old ‘golden rule’. Revisit your materials on statutory interpretation and your understanding of the different ‘rules’ on or approaches to interpretation that can be taken by the courts. Revisit your materials on statutory interpretation and your understanding of the different ‘rules’ on or approaches to interpretation that can be taken by the courts.

How well did you know this?
1
Not at all
2
3
4
5
Perfectly
63
Q

Which one of the following statements best describes the relevant stage of the legislative process?

The committee stage of the bill gives MPs from the relevant select committee the chance to scrutinise the detail of the bill and to propose necessary amendments.

The committee stage of a bill gives MPs appointed to the relevant legislative committee the chance to scrutinise the detail of the bill and to propose necessary amendments.

The third reading of the bill allows MPs a final chance to amend a bill and to negotiate over its final shape and content.

The second reading of a bill gives MPs the greatest chance to scrutinise it in detail.

The report stage of the bill involves a specially selected legislative committee making final amendments to the bill.

Which one of the following statements best describes the relevant stage of the legislative process?

The committee stage of the bill gives MPs from the relevant select committee the chance to scrutinise the detail of the bill and to propose necessary amendments.

The committee stage of a bill gives MPs appointed to the relevant legislative committee the chance to scrutinise the detail of the bill and to propose necessary amendments.

The third reading of the bill allows MPs a final chance to amend a bill and to negotiate over its final shape and content.

The second reading of a bill gives MPs the greatest chance to scrutinise it in detail.

The report stage of the bill involves a specially selected legislative committee making final amendments to the bill.

A

The committee stage of a bill gives MPs appointed to the relevant legislative committee the chance to scrutinise the detail of the bill and to propose necessary amendments.

Correct. Note that MPs are selected on the basis of party strength in the Commons to scrutinise an individual bill. Also note that this type of committee used to be called a ‘standing’ committee and is sometimes still referred to as such. The other answers do not correctly describe in all respects the legislative process operating within Parliament.

How well did you know this?
1
Not at all
2
3
4
5
Perfectly
64
Q

Which one of the following statements best describes the key characteristics of secondary legislation?

Secondary legislation is necessary in the modern world in order to ensure that different branches of national and local government have the detailed powers necessary to administer the state. The delegation of the ability to create these detailed powers is heavily controlled, however, by Parliament through tight scrutiny procedures.

Secondary legislation is not made by Parliament itself but usually by government departments under powers given to them by Parliament in primary legislation. It is subject to detailed initial political scrutiny by MPs after it has been created to ensure that it has been created within the powers originally approved by Parliament.

Secondary legislation is not made by Parliament itself but usually by government departments under powers given to it by Parliament in primary legislation. It is subject to varying degrees of initial political scrutiny but can be judicially reviewed by the courts to ensure that it has been created within the powers originally approved by Parliament.

Secondary legislation can be referred to as delegated or subordinate legislation.

The most common form of delegated authority given to the government in an Act of Parliament relates to the commencement date for applying the Act, either as a whole or in different stages.

A

Secondary legislation is not made by Parliament itself but usually by government departments under powers given to it by Parliament in primary legislation. It is subject to varying degrees of initial political scrutiny but can be judicially reviewed by the courts to ensure that it has been created within the powers originally approved by Parliament.

Correct. Note that the original powers are given by Parliament in a so-called Parent Act. The other answers do not correctly or adequately describe key features of secondary legislation including the degree to which it is scrutinised before implementation.

How well did you know this?
1
Not at all
2
3
4
5
Perfectly
65
Q

An incident recently occurred in which a young man was shot dead by the police as they feared he was brandishing a dangerous weapon in a shopping mall and behaving in an alarming way. It transpired that he had a serious mental health condition and that he was waving a long plastic knife and not a real one. The man has left a widow and a young child who do not have any income, as he was the only one in employment. The Department for Work and Pensions (‘DWP’) has refused anything other than the most basic form of benefit assistance, because it claims that the man’s actions effectively brought about his own death.

Which one of the following statements best describes the judicial bodies that his widow is likely to encounter in investigating and challenging what has happened?

There will be a Coroner’s Court inquest, as the death occurred in public and was not due to natural causes, and there is also a possibility of a statutory inquiry taking place. His widow may also need to issue proceedings in the County Court against the DWP for its decision to deny her benefit assistance.

There will be a Coroner’s Court inquest, as the death occurred in public and was not due to natural causes, and the police officer who shot the man will subsequently be prosecuted in the Crown Court. The widow may also need to challenge the DWP’s decision to deny her benefit assistance through the Magistrates Court.

There will be a Coroner’s Court inquest, as the death occurred in public and was not due to natural causes, and in addition there will be a statutory inquiry because of the seriousness of the matter. The man’s widow may also need to challenge the DWP’s decision to deny her benefit assistance through the Social Entitlement Chamber of the First Tier Tribunal.

As the man has been killed, and so his right to life has been engaged, there will be an automatic referral of this matter to a judicial review in the Administrative Court. His widow may also need to challenge the DWP’s decision to deny her benefit assistance through the Magistrates Court.

There will be a Coroner’s Court inquest, as the death occurred in public and was not due to natural causes, and there is also a possibility of a statutory inquiry taking place. The man’s widow may also need to challenge the DWP’s decision to deny her benefit assistance through the Social Entitlement Chamber of the First Tier Tribunal.

A

There will be a Coroner’s Court inquest, as the death occurred in public and was not due to natural causes, and there is also a possibility of a statutory inquiry taking place. The man’s widow may also need to challenge the DWP’s decision to deny her benefit assistance through the Social Entitlement Chamber of the First Tier Tribunal.

Correct
Correct. A statutory inquiry could be ordered in this situation but is only likely to happen if there are exceptional features to this incident or there has been a spate of such incidents, suggesting systemic problems in police handling of such matters. The other answers do not demonstrate as clear a picture of the likely involvement of the key statutory judicial bodies and tribunals.

How well did you know this?
1
Not at all
2
3
4
5
Perfectly
66
Q

Which one of the following best describes the mix of sources of constitutional rules in the UK?

The UK’s constitutional ‘rules’ comprise a mixture of legal rules, which come from constitutional statutes, EU law and the common law (including the recognition of prerogative powers), as well as non-legal rules, which comprise conventions, the law and customs of Parliament, and academic opinion.

The UK’s constitutional ‘rules’ comprise a mixture of legal rules, including constitutional statutes, EU law and the common law, as well as non-legal rules including conventions, the law and customs of Parliament, and academic opinion.

The UK’s constitutional ‘rules’ comprise a mixture of legal rules, including constitutional statutes, EU law and the common law, as well as non-legal rules including conventions, the law and customs of Parliament, and academic opinion, all of which have equal status.

The UK’s constitutional ‘rules’ comprise a mixture of legal rules, which come from statute, and the common law, as well as non-legal rules, which comprise constitutional conventions, the law and customs of Parliament, and academic opinion.

The UK’s constitutional ‘rules’ primarily derive from non-legal sources, notably constitutional conventions, which are far more prevalent in the UK than in other states, although these rules have been supplemented in recent decades by a number of “constitutional statutes”.

A

The UK’s constitutional ‘rules’ comprise a mixture of legal rules, which come from constitutional statutes, EU law and the common law (including the recognition of prerogative powers), as well as non-legal rules, which comprise conventions, the law and customs of Parliament, and academic opinion.

Correct.

How well did you know this?
1
Not at all
2
3
4
5
Perfectly
67
Q

At the recent (fictitious) general election, the new government party obtained a majority of seats on the basis of a manifesto in which “the need for enhanced security measures” was emphasised strongly. A few months afterwards the new Home Secretary sponsored legislation in relation to anti-terrorism powers. This passed through the House of Commons, but the Bill was met with opposition in the House of Lords which rejected the Bill on its second reading.

Which one of the following statements describes the best course of action open to the Home Secretary?

The Home Secretary should impress on the House of Lords that it has a constitutional obligation under the Sewel convention to respect the mandate of the Commons, meaning that it should consent to the Bill when it is presented before it on the next occasion.

The Home Secretary should impress on the House of Lords that it has an obligation to consent to the Bill when it presented before it on the next occasion, as the matter relates to national security which is a matter on which the Lords should defer to the electoral mandate of the Commons.

The Home Secretary should impress on the House of Lords that since the House of Lords is an unelected body, it is unconstitutional for it to refuse to consent to the Bill.

The Home Secretary should impress on the House of Lords that it is a legal obligation under the Salisbury convention to respect the mandate of the Commons, meaning that it must consent to the Bill when it is presented before it on the next occasion.

The Home Secretary should impress on the House of Lords that it has a constitutional obligation under the Salisbury convention to respect the mandate of the Commons, meaning that it should consent to the Bill when it is presented before it on the next occasion.

A

The Home Secretary should impress on the House of Lords that it has a constitutional obligation under the Salisbury convention to respect the mandate of the Commons, meaning that it should consent to the Bill when it is presented before it on the next occasion.

correct

incorrect
The Home Secretary should impress on the House of Lords that it has an obligation to consent to the Bill when it presented before it on the next occasion, as the matter relates to national security which is a matter on which the Lords should defer to the electoral mandate of the Commons.

This is a point that may be argued by the government in the Commons, but it is rather intangible and not a recognised convention as such.

How well did you know this?
1
Not at all
2
3
4
5
Perfectly
68
Q

Which one of the following statements best describes the essential character of the UK’s constitution?

The UK’s constitution is famously a highly flexible one with effectively no controls over how the political leaders of the day wish to exercise their power other than purely conventional ones.

The UK’s constitution is dominated by relics of its historical past in the form of a monarch as head of government, a hereditary House of Lords as a significant part of Parliament, and the continued survival of royal prerogative powers.

The primary source of constitutional law in the UK is case law, although other sources of constitutional law, such as Acts of Parliament and constitutional conventions also play an important role.

The UK is a constitutional state with no formal constitution. Its lack of entrenched constitutional rules in a founding document is significant but this does not mean to say that there are no legal or conventional protections preserving constitutional values and standards.

The UK effectively has no constitution at all, given the absence of any codified and entrenched set of constitutional rules and protections.

A

The UK is a constitutional state with no formal constitution. Its lack of entrenched constitutional rules in a founding document is significant but this does not mean to say that there are no legal or conventional protections preserving constitutional values and standards.

Correct

How well did you know this?
1
Not at all
2
3
4
5
Perfectly
69
Q

Which one of the following statements is an accurate description of a key aspect of the constitutional structure of the United Kingdom?

Ministers who intentionally breach the Ministerial Code will be directed by the Speaker of the House to resign.

The powers of the UK Supreme Court are more limited than that of the US Supreme Court, as it is not possible for the UK Supreme Court to invalidate primary legislation.

Certain constitutional conventions, such as the Sewel convention, which regulates aspects of the relationship between England and Scotland, have such significance that the courts will be willing to enforce them in certain circumstances.

The existence of prerogative powers, being powers held in the name of the monarch, and including the power to declare war, means that the political power of the monarch in the UK is significant.

In the UK the Executive is responsible for the enactment of law. The legislature is responsible for scrutinising and (where necessary) amending the law, and the judiciary is responsible for the administration of justice according to the law.

A

The powers of the UK Supreme Court are more limited than that of the US Supreme Court, as it is not possible for the UK Supreme Court to invalidate primary legislation.

correct

incorrect
Certain constitutional conventions, such as the Sewel convention, which regulates aspects of the relationship between England and Scotland, have such significance that the courts will be willing to enforce them in certain circumstances.

This is incorrect. While certain constitutional conventions may have particular importance in allowing the UK’s constitution to function, constitutional conventions are not legally enforceable. This was confirmed in the case of Miller in which the Supreme Court stressed that conventions such as the Sewel convention are political rather than legal in nature.

How well did you know this?
1
Not at all
2
3
4
5
Perfectly
70
Q

The Chancellor of the Exchequer has been given powers by Parliament under the (fictitious) Emergency Revenue Regulations (“the Regs”) to impose a windfall tax on internet search companies which “the Chancellor deems to have paid a disproportionately low amount of corporation tax in the preceding financial year.” This year the Chancellor is very busy preparing the Budget and has delegated responsibility for the Regs to a senior Treasury civil servant.

Which one of the following statements best describes the constitutional position if a company alleges that these powers have been misused?

The general principle is that powers given by Parliament to particular members of the Executive should be exercised by that person. When these are precise powers, outlined in statute, as here, it is necessary, legally and politically, for the Chancellor to have made the relevant decision in respect of the use of the powers.

Before deciding to delegate powers under the Regs, the Chancellor was legally obliged to report to Parliament and request the necessary authority to delegate.

Although there is a degree of conventional pressure on the Chancellor to make the decisions to exercise these powers, in reality there are no legal controls or rights of review over who has actually exercised them.

The general principle is that powers given by Parliament to particular members of the Executive should be exercised by that person. However, there is a recognition in both constitutional and legal terms, that delegation is necessary in the interests of governmental efficiency and so it would not be unlawful for delegation in this situation to the civil servant.

The general principle is that powers given by Parliament to particular ministers in the Executive should be exercised by that person. However, there is a recognition in both constitutional and legal terms, that delegation is necessary in the interests of governmental efficiency and so it would not be unlawful for delegation in this situation to the civil servant. The Chancellor would, however, remain responsible for these actions in the political arena.

A

The general principle is that powers given by Parliament to particular ministers in the Executive should be exercised by that person. However, there is a recognition in both constitutional and legal terms, that delegation is necessary in the interests of governmental efficiency and so it would not be unlawful for delegation in this situation to the civil servant. The Chancellor would, however, remain responsible for these actions in the political arena.

How well did you know this?
1
Not at all
2
3
4
5
Perfectly
71
Q

Which one of the following statements best describes the constitutional structure of the United Kingdom?

The constitutional reforms introduced in the years after 1998 have clearly created new forms and levels of representation in Edinburgh, Cardiff and Belfast but there have been few concrete practical effects of the reforms.

The UK’s structure has changed from being highly centralised in the second half of the twentieth century to a form of federation of four constituent parts, albeit with a varying degree of autonomy between them, following reforms in the last quarter century.

Since 1998 the process of devolution has been a developing one with significant practical effects on government and the division of responsibilities between the constituent countries of the UK, albeit one which has not changed the ultimate centralised sovereignty of Westminster.

In spite of a number of constitutional reforms carried out since 1998, the structure of the UK has essentially remained the same since the Irish Free State left the UK in the early 1920s.

The UK’s structure has changed from being highly centralised in the second half of the twentieth century to a form of federation of four constituent parts following reforms in the last quarter century.

A

Since 1998 the process of devolution has been a developing one with significant practical effects on government and the division of responsibilities between the constituent countries of the UK, albeit one which has not changed the ultimate centralised sovereignty of Westminster.

correct

How well did you know this?
1
Not at all
2
3
4
5
Perfectly
72
Q

Which one of the following statements best describes the effect of the Constitutional Reform Act 2005 (‘CRA’) on the UK’s separation of powers?

Although the CRA did a great deal to bolster the sense of independence and the prestige of the UK judiciary it had little or no effect on the other two bodies of state.

The CRA introduced a significant provision in that it provided for judicial security of tenure, providing members of the judiciary with crucial protection from politically motivated dismissal by the executive.

The changes brought about by the CRA were purely symbolic and were of little practical consequence.

The CRA clearly did not introduce the idea of an independent judiciary to the UK but it arguably enhanced the status of the higher judiciary as well as introducing a series of reforms that ironed out some of the more confusing functional overlaps in the UK’s traditional separation of powers model.

There was effectively no separation of powers to speak of within the UK constitution before the CRA created a set of more precise mechanisms for dividing power up between the three bodies of state and for recognising the independence of the judiciary.

A

The CRA clearly did not introduce the idea of an independent judiciary to the UK but it arguably enhanced the status of the higher judiciary as well as introducing a series of reforms that ironed out some of the more confusing functional overlaps in the UK’s traditional separation of powers model.

correct

How well did you know this?
1
Not at all
2
3
4
5
Perfectly
73
Q

The court has just made an order putting a temporary stop on the deportation of an asylum seeker who has been given permission to appeal against his imminent planned removal from the country. Enquiries in the Home Office reveal that the individual is in a holding cell at Gatwick Airport and the flight is due to leave. The matter is referred to the personal office of a Minister of State in the Home Office which directs that the deportation should go ahead as it is too late to delay the process.

After the asylum seeker’s lawyers issue an application to court to review these actions, which one of the following steps is the court most likely to take?

The court is likely to find that the office of the Minister in the Home Office had been in contempt of court.

The court is likely to decline to make any formal order but is likely to declare that Parliament ought to amend the law so that the Home Office could not behave in this way in the future.

The court is likely to show judicial deference, and decline to make any order, as the Home Office Minister and staff in his office would be seen as best placed to make decisions in relation to deportation.

The court is likely to express concern about the way the Home Office Minister behaved but this would only have political consequences, namely considerable criticism in the House of Commons and in select committees, rather than legal ones.

The court would only make any order if it was satisfied that the Minister was personally involved in the making of the decision as it would be inappropriate to hold the Minister responsible for a power which had been exercised by another government official.

A

The court is likely to find that the office of the Minister in the Home Office had been in contempt of court.

Correct. This is similar to what happened in the leading case of M v Home Office. It represents a clear example of the separation of powers in operation: an independent judiciary performing its primary function of impartially applying the law, in this case against a high office of state.

How well did you know this?
1
Not at all
2
3
4
5
Perfectly
74
Q

The principle of separation of powers is motivated by the need to limit and control the power of the different bodies of state. Which one of the following statements best describes the means by which the UK’s executive branch can be controlled by the judiciary?

The UK’s central executive is ultimately uncontrollable by the judiciary, as, in contrast to most democratic countries, the UK Supreme Court does not have the power to enforce compliance with a codified constitution.

The separation of powers in the UK constitution ensures that there is a significant degree of separation between the legislature and the executive, as this is crucial in achieving the protection of liberty as described by Montesquieu.

The UK’s executive is potentially very powerful, but the judiciary is able to exercise a degree of control and oversight through the process of judicial review, which has expanded notably in the last 40 years or so. Significantly, this also permits the courts to invalidate secondary legislation, created by the executive, if this is deemed outside or ‘ultra vires’ the powers given by Parliament.

Given the expansion in the remit of judicial review in the last 40 years or so, the UK’s higher judiciary is able to effectively check the executive in all respects, providing a crucially important barrier against the occasionally over-mighty reach of government.

A government enjoying a workable majority in the Commons may face judicial obstruction to some of its policies, even if it has a political free hand. Ultimately, however, the UK judiciary is not able to check the actions of the executive, as long as it is acting within the powers given to it by Parliament, because the courts are not able to quash any form of legislation.

A

The UK’s executive is potentially very powerful, but the judiciary is able to exercise a degree of control and oversight through the process of judicial review, which has expanded notably in the last 40 years or so. Significantly, this also permits the courts to invalidate secondary legislation, created by the executive, if this is deemed outside or ‘ultra vires’ the powers given by Parliament.

How well did you know this?
1
Not at all
2
3
4
5
Perfectly
75
Q

The court is presented with a public law dispute which involves consideration of a particular social issue on which Parliament has not yet enacted any legislation. The only threads of legal authority that can be traced come from 19th century cases, at a time when social attitudes were far more ‘traditional’.

When adjudicating on the issue, which of the following represents the most likely approach that the court will take and the most accurate explanation for doing so?

The court will look to see if there is any legislation on a comparable area and seek to apply that by analogy to the case before it.

The court is entitled to develop common law principles and will attempt to do so by seeking to take into account contemporary social attitudes as long as there is a reasonable degree of consensus over these.

The court has the right to develop the common law in any way it chooses but, because the matter is one that relates to social policy, will automatically find in favour of the individual against the ‘government’ side in the dispute.

The court has the right to develop the common law in any way it chooses but, because the matter is one that relates to social policy, will automatically find in favour of the ‘government’ side in the dispute.

The court will simply apply the most recent case precedent from the 19th century case as this is the only authority it possesses.

A

The court is entitled to develop common law principles and will attempt to do so by seeking to take into account contemporary social attitudes as long as there is a reasonable degree of consensus over these.

This is the best answer as it shows that, in the absence of clear (or any) statutory guidance, the courts can develop legal principles through the common law. However, when doing so, the courts are mindful of the need to represent contemporary social attitudes and values. See, for example, the case of R v R.

How well did you know this?
1
Not at all
2
3
4
5
Perfectly
76
Q

Two students were having a debate about how the principle of the Rule of Law could be described. Which of the following statements would NOT be considered as an important aspect of the Rule of Law?

Government should act within the legal authority given to it.

It is important that citizens should be able to have effective access to the law.

There should be equality before the law.

Parliament has the right to make or unmake any law whatsoever.

It is important that legal processes are fair.

A

Parliament has the right to make or unmake any law whatsoever.

Correct. This relates to the principle of Parliamentary Sovereignty rather than to that of the Rule of Law, as it is about where law comes from rather than the nature of it.

How well did you know this?
1
Not at all
2
3
4
5
Perfectly
77
Q

Under a (fictitious) Act of Parliament passed in the 1950s, county councils were given the power to make loans to farmers to help them purchase additional animal feed in times of exceptionally difficult weather conditions. Over the last year farmers in one county in south-west England have encountered a sustained period of flooding. The situation has been so dire that the local county council has gone ahead to give out non-repayable grants to more than 100 local farmers. This expenditure has been contested, however, by a local Council Taxpayers’ association, which has now taken the county council to court to review its decision.

Which of the following is most likely to be the court’s reaction?

The court will find that the county council has technically acted outside the strict powers given to it by Parliament. However, it will not find its actions unlawful because it will be able to take into account the fact that the Act is many decades old. It is therefore likely to interpret its provisions in the light of more modern economic conditions, in which it is imperative to support the agricultural sector in rural areas.

The court will adopt the so-called ‘legality principle’, associated with the case of Simms, and declare that Parliament, when passing the relevant Act, could not possibly have intended that more favourable financial assistance, in the form of non-repayable grants, would be refused in exceptional cases.

The court will find the dispute to be non-justiciable, as it relates solely to a matter of local government finance.

The court will fully interpret the relevant legislation, under which the county council was acting, but it appears at face value that it was not given the statutory power to give grants, only to provide loans. It is likely, therefore, that its actions will be declared ultra vires and therefore unlawful.

The court will fully review the relevant legislation and will do its utmost to interpret it in such a way as to favour the individual farmers, concluding that the payment of grants was lawful.

A

The court will fully interpret the relevant legislation, under which the county council was acting, but it appears at face value that it was not given the statutory power to give grants, only to provide loans. It is likely, therefore, that its actions will be declared ultra vires and therefore unlawful.

Correct. The actions taken appear to have been beyond the powers given to the council.

How well did you know this?
1
Not at all
2
3
4
5
Perfectly
78
Q

In a bid to reduce expenditure the Ministry of Justice (‘MoJ’) has adopted a new policy on costs in the Social Security Tribunal. Under a recently introduced statutory instrument (‘S.I.’) the MoJ has imposed a differential fee structure, meaning that claimants who lose their applications to the Tribunal have to pay a ‘closure fee’ of £100 at the end of the case, though there is discretion for judges to disregard this in ‘exceptional circumstances’. The legality of this measure, which has led so far to a 50% reduction in the number of cases reaching the Tribunal, is now being challenged by a pressure group acting for families with children living in poverty.

Which of these outcomes is the most likely when the case reaches the Administrative Court for judicial review?

The court will look at the strict wording of the S.I. and its parent Act and apply this literally so, if it considers that this allows a ‘closure fee’ to be levied, this will be enforced regardless of its effect.

The court will refuse to intervene in this matter, as it will point to the provision in the S.I. allowing for discretion to disregard the fee, as well as being conscious that it should be deferential in an area involving finance.

The court is likely to put considerable emphasis on the impact caused by the introduction of the closure fee and find that this has severely undermined the important constitutional principle of access to justice. There is a strong possibility that the court will consider the fee to be unlawful, in the sense that Parliament cannot have intended to allow such a change in the parent Act, and therefore quash the S.I.

The court is likely to take into account the 50% reduction in cases and find that this demonstrates how the principle of access to justice has been damaged. It will therefore make an order reducing the ‘closure fee’ by 50%.

The court will examine the relevant S.I. in detail and is likely to put a significant amount of store by the importance of the principle of access to justice. However, it will decline to take any evidence on the overall impact of the fee on numbers of cases, as it will be obliged to look at this issue purely as a matter of principle.

A

The court is likely to put considerable emphasis on the impact caused by the introduction of the closure fee and find that this has severely undermined the important constitutional principle of access to justice. There is a strong possibility that the court will consider the fee to be unlawful, in the sense that Parliament cannot have intended to allow such a change in the parent Act, and therefore quash the S.I.

Correct. Refer back to your materials and in particular the leading case of Unison.

How well did you know this?
1
Not at all
2
3
4
5
Perfectly
79
Q

One of the most acute forms of tension in public law is caused by the widespread presence of extensive discretionary power in the hands of executive decision makers.

Which of the following statements best summarises the approach of the modern judiciary to this issue?

The courts have come to accept that discretionary powers are an unavoidable feature of modern government, as Parliament cannot possibly legislate for every given situation and therefore needs to provide the executive with powers that it can decide how to use. Consequently, the courts will allow the executive free rein to use these powers in any manner that it sees fit.

The courts will interpret discretionary statutory powers in a way that upholds key rule of law principles, such as fairness, legality and reasonableness, so that they can ensure some degree of legal control over how such powers are exercised by different parts of government.

The courts will always interpret discretionary statutory powers by applying a test of proportionality so that they can better balance the interests of the state and the individual.

The courts have become increasingly reluctant in recent decades to intervene in the exercise of statutory discretionary power, in order to respect separation of powers.

The courts will be prepared to invalidate powers given to different parts of the executive if they contain broad discretionary powers, as these are seen to be damaging to the rule of law.

A

The courts will interpret discretionary statutory powers in a way that upholds key rule of law principles, such as fairness, legality and reasonableness, so that they can ensure some degree of legal control over how such powers are exercised by different parts of government.

Correct

How well did you know this?
1
Not at all
2
3
4
5
Perfectly
80
Q

In order to safeguard against financial fraud Parliament passed a (fictitious) Act allowing the (fictitious) Data Security Agency (‘DSA’) to ‘search any building in which data is kept or stored electronically which the DSA believes could reveal information relevant to a criminal investigation’. The DSA has recently forcibly entered and searched the premises of a financial services company in the City without notice or search warrant, as its Director believed that the company’s managing director had unjustifiably refused to disclose financial data to the Crown Prosecution Service.

Which of the following best describes the likely approach of the court if the company decides to take up a legal case against the DSA for its actions?

The court faces a difficult issue here which will involve it in balancing its assessment of the purpose and intent of these powers, given by Parliament, with the impact upon the company of what appear to be quite draconian actions. It is probable that a modern court will put the DSA to proof in relation to the lack of ‘due process’ and detailed accusations against the company, and be prepared to declare the search unlawful, if insufficient evidence can be supplied that such action was necessary.

On the basis that the powers relate to potential evidence gathering for criminal investigation, the court is likely to reject any challenge by the company as the question of what action is necessary in this context is a question for the DSA and not for the court.

The court is unlikely to accept jurisdiction to hear the challenge to the DSA’s actions, as it will stress that these actions were taken under clear statutory powers and that any rule of law concerns about the nature of these powers have to take second place, constitutionally, to the higher authority of Parliament.

As the power to enter and search the premises is a discretionary power, based upon the DSA’s belief that a search is needed to reveal information relevant to a criminal investigation, the court is likely to be very reluctant to allow any challenge to DSA’s action, owing to the discretionary nature of DSA’s power.

The court would take a literal approach to the legislation and is accordingly likely to find that the DSA has no liability for taking this action.

A

The court faces a difficult issue here which will involve it in balancing its assessment of the purpose and intent of these powers, given by Parliament, with the impact upon the company of what appear to be quite draconian actions. It is probable that a modern court will put the DSA to proof in relation to the lack of ‘due process’ and detailed accusations against the company, and be prepared to declare the search unlawful, if insufficient evidence can be supplied that such action was necessary.

Correct. This answer reflects how the modern courts will interpret and assess the legislative position in the light of substantive rule of law concerns over how discretionary powers have been exercised. It is possible to see a precursor of this kind of approach in Lord Denning’s judgment in the Court of Appeal in the Rossminster case.

81
Q

In order to prepare for future national health emergencies, the Secretary of State for Health and Social Care (SoS) has sponsored (fictitious) legislation through Parliament. Section 17 of this Act allows the SoS and civil servants in his department unimpeded and immediate access to every citizen’s NHS records. (For your information: this legislation is incompatible with Article 8 of the European Convention on Human Rights.)

Your client does not want the SoS to have unimpeded access to her health records, and she instructs you to challenge the exercise by the SoS of his powers under s.17.

Which one of the following statements best describes the likely approach the court will take using its powers under the Human Rights Act (‘HRA’)?

As s.7 is incompatible with Article 8, a declaration to that effect will immediately be made by the court. This would need to be responded to by the SoS, but it would not compel the government to seek a change in the law.

Under s.3 of the HRA, the court will first consider if it can interpret s.17 in a way that makes it compatible with Article 8. If the court does not consider this is possible, it may make a declaration of incompatibility in relation to s.17. This would compel the SoS to change the law in order to remove the incompatibility with Article 8.

Under s.3 of the HRA the court will first consider if it can interpret s.17 in a way that makes it compatible with Article 8. If the court does not consider it is possible to do so, it may make a declaration of incompatibility in relation to s.17. This would need to be responded to by the SoS, but it would not compel the government to seek a change in the law.

Under s.3 of the HRA the court must read in the words it considers necessary to make s.17 compatible with Article 8.

If s.17 is found to be incompatible with Article 8, a declaration to that effect will be made by the court. This would compel the SoS to change the law to remove the incompatibility with Article 8.

A

Under s.3 of the HRA the court will first consider if it can interpret s.17 in a way that makes it compatible with Article 8. If the court does not consider it is possible to do so, it may make a declaration of incompatibility in relation to s.17. This would need to be responded to by the SoS, but it would not compel the government to seek a change in the law.

Correct. If a statutory provision is incompatible with a right in the ECHR, the courts have two alternative routes they can take, as outlined in this answer. Note too the effect of a DOI.

82
Q

Which one of the following statements most accurately reflects the position in relation to the traditional conception of parliamentary sovereignty?

In a situation where Parliament has passed a piece of legislation but where it is clear to the court that the Act was not passed according to the correct procedure, it is open to the court to quash the legislation because of the procedural irregularity.

Although in general, Parliament cannot bind its successors, it is possible in certain cases, where a statute is of a constitutional nature, for Parliament to be bound by legislation which has been passed by a predecessor Parliament.

The effect of the judgment in the case of Jackson is that it is now possible for a Parliament to ‘redefine itself upwards’ by requiring a particular parliamentary majority or a popular referendum for a particular type of measure.

If it wishes, Parliament may pass legislation that prohibits conduct in places outside the jurisdiction of the UK, even if the act in question would not be prohibited by the authorities in that territory.

Where the content of a particular provision within a piece of primary legislation is contrary to international law, it will be possible for a UK court to quash such a provision, provided that the quashing of the provision does not impact on the function of the statute as a whole.

A

If it wishes, Parliament may pass legislation that prohibits conduct in places outside the jurisdiction of the UK, even if the act in question would not be prohibited by the authorities in that territory.

Correct. There are no geographical limitations upon the legislation that Parliament may pass. This would not be invalid legislation, (though its political purpose would be highly dubious!)

incorrect
In a situation where Parliament has passed a piece of legislation but where it is clear to the court that the Act was not passed according to the correct procedure, it is open to the court to quash the legislation because of the procedural irregularity.

This is not the correct answer. If a bill has been enrolled – in other words, it has become an Act of Parliament – it is impossible for a court to go behind that. Any departure from normal procedure during the passage of the bill cannot be corrected by the courts. See the cases of Edinburgh and Dalkeith Railway and Pickin.

The effect of the judgment in the case of Jackson is that it is now possible for a Parliament to ‘redefine itself upwards’ by requiring a particular parliamentary majority or a popular referendum for a particular type of measure.

This is an overstatement of the effect of Jackson as in that case, the court suggested only (very much in obiter) that this course of action may be possible, not that it is in fact now possible.

83
Q

A (fictitious) Prime Minister (“PM”), who has just entered office, wishes to sponsor legislation to prohibit a future Parliament from bringing back the death penalty for crimes of murder. She will call the legislation the ‘Prevention of Capital Punishment Act’ (the “PCPA”.)

Which one of the following statements represents the best advice that could be given to the PM by her legal advisors?

It is certainly possible for Parliament to pass legislation to this effect and the PCPA would remain in operation until such time as a future Parliament wished to reverse the policy. In order to do so, that future Parliament would have to pass new legislation expressly re-introducing the death penalty.

The doctrine of parliamentary sovereignty means that it is certainly possible for the PM to sponsor the proposed PCPA. The PM should also be reassured that, if a future government wished to reverse the policy, it would find it highly difficult to do so, because the PCPA would be seen as a “constitutional statute”. Therefore, in accordance with the rule in the Thoburn case, a future court would not accept that it could be repealed. Re-visit your materials on parliamentary sovereignty.

It is not possible for Parliament to pass legislation to this effect because it is directly infringing Dicey’s theory on sovereignty, which says that it is impossible for one Parliament to restrict the future freedom of a successor Parliament.

The doctrine of parliamentary sovereignty means that it is certainly possible for the PM to sponsor the proposed PCPA. The PM should also be reassured that, if a future government wished to reverse the policy, it would find it impossible to do so, because repealing the PCPA would be deemed incompatible with the European Convention on Human Rights.

The doctrine of parliamentary sovereignty means that it is certainly possible for the PM to sponsor the proposed PCPA and, if this legislation contains terms written clearly and precisely enough, its provisions will endure because it represents an unambiguous expression of parliamentary will.

A

It is certainly possible for Parliament to pass legislation to this effect and the PCPA would remain in operation until such time as a future Parliament wished to reverse the policy. In order to do so, that future Parliament would have to pass new legislation expressly re-introducing the death penalty.

Correct. The PM will be able to get the Act passed, assuming there is a majority for her to do so. However, the doctrine of parliamentary sovereignty means that today’s Parliament cannot bind a future Parliament, so the PCPA and its ban on bringing back capital punishment cannot be permanently entrenched.

84
Q

The (fictitious) Flood Defences Act 1978 stated that, in the event of serious flooding in any local government area, the local authority could claim a relief grant of £1000 per hectare (subject to annual inflationary uplifts), “such sum to be fixed as a minimum and not to be reduced in future”.

In 2019 Parliament passed the (fictitious) Local Emergency Relief Act, which has no mention at all in it of the 1978 Act. It imposed a relief grant lower than that in the 1978 Act.

Which one of the following statements best describes the likely outcome if a local authority challenges the amount of relief grant it receives under the terms of the 2019 Act in future, and the rationale behind it?

The local authority would be unsuccessful as the court is likely to find that the 1978 Act had been expressly repealed by the 2019 Parliament, and so the terms in the earlier Act have been amended.

The court is likely to send the matter to an alternative form of arbitration, as it will not be possible for it to determine whether the 1978 or the 2019 Act should prevail, as both statutes represent legitimate expressions of parliamentary will.

The local authority would be successful in challenging the amount as the court would interpret the words in the 1978 Act literally and therefore not permit the rate of relief grant to be reduced.

The local authority would have reasonable chances of success in challenging the 2019 grant level, as the 1978 Act would be seen as a constitutional statute, as flooding is such a serious national issue, and so would not be subject to implied repeal.

The local authority would be unsuccessful in challenging the amount and would be obliged to accept the figure in the 2019 Act, as this Act represents the most recent expression of Parliament’s sovereign will.

A

The local authority would be unsuccessful in challenging the amount and would be obliged to accept the figure in the 2019 Act, as this Act represents the most recent expression of Parliament’s sovereign will.

Correct
Correct. This accurately reflects the operation of the principle of implied repeal.

85
Q

Which one of the following statements best describes the relationship between the principle of parliamentary sovereignty and the UK’s current devolution arrangements?

The various devolution statutes passed from the late nineties onwards have meant that the model of a federal UK has been firmly entrenched in the country’s constitution.

The core constitutional principle of parliamentary sovereignty means that there is no legal limitation on the Westminster Parliament legislating to reverse the devolution settlement in the UK, in spite of the existence of the Sewel convention.

In legal theory the devolution settlements in all of the devolved parts of the UK could be repealed tomorrow but this is prevented from happening in reality by the operation of the Sewel convention.

The core constitutional principle of parliamentary sovereignty means that there is and can be no legal or political limitations on the ability of the Westminster Parliament to legislate to reverse the devolution settlement in the UK.

The devolution settlements in Wales and Northern Ireland could be repealed by the Westminster Parliament at any time but the same would not be possible for Scotland because of the higher degree of nationalist support in that part of the UK.

A

The core constitutional principle of parliamentary sovereignty means that there is no legal limitation on the Westminster Parliament legislating to reverse the devolution settlement in the UK, in spite of the existence of the Sewel convention.

Correct. This is an accurate summary of the position regarding the status of the devolution legislation. Note that this does not mean there are no political limitations.

86
Q

olidation MCQs
My Modules
Public Law (SQE)
Which one of the following statements best describes the broad historical processes at work in the development of prerogative power in the period since the Glorious Revolution in 1688?

Prerogative power remains a very significant form of governmental power. Although it is not seen to have the same degree of status as statutory power, it can only be controlled legally to a certain extent, because of the presumption that the Crown is not bound by statute in the absence of express words to the contrary.

Prerogative power remains the predominant form of governmental power and, though legal controls over it have been enhanced over the last century, it can still be seen as a special form of power resting in the hands of the Prime Minister and Cabinet with only limited legal controls over it.

Prerogative power is not as broad a source of governmental power as it was, given the increase in the number of statutes over the last century in particular. Its constitutional significance is now purely a matter of historical interest.

Prerogative power remains a very significant form of governmental power and, although political controls over it have been enhanced over the last century, it can still be seen as a special and unique form of power resting in the hands of the Prime Minister and Cabinet with only limited legal controls over it.

Prerogative power is not as broad a source of governmental power as it was, given the increase in the number of statutes over the last century in particular. The prerogative remains significant constitutionally, but it cannot be seen as a special form of power anymore, as it is no longer immune from the legal controls exercised over other governmental powers.

A

Prerogative power is not as broad a source of governmental power as it was, given the increase in the number of statutes over the last century in particular. The prerogative remains significant constitutionally, but it cannot be seen as a special form of power anymore, as it is no longer immune from the legal controls exercised over other governmental powers.

Correct
Correct. This briefly describes both the narrowing in scope of the prerogative and the change that has happened over time in the degree to which this form of power is reviewable by the courts.

incorrect
Prerogative power remains a very significant form of governmental power and, although political controls over it have been enhanced over the last century, it can still be seen as a special and unique form of power resting in the hands of the Prime Minister and Cabinet with only limited legal controls over it.

This is not the best answer as it overstates the proportionate size of the prerogative within the overall range of powers held by government. Importantly it only refers to the increasing political controls over its use; there is no reference to the enhanced legal reviewability of how the government uses these powers. Re-visit your materials on the control of the prerogative.

87
Q

The Home Secretary recently decided to use the prerogative to apply an intrusive search power for terrorism suspects detained at airports on entry into the UK. This was justified as a measure to maintain national security and was introduced even though a statutory power had been given to the Home Secretary over a year ago to implement a provision allowing similar but less intrusive searches to take place. After the prerogative search power was used for the first time, the affected party challenged the use of this power through judicial review.

Which one of the following statements best describes the likely approach to be taken by the court?

The Court will decide that the prerogative power will prevail through implied repeal.

The Court will decide that the prerogative power has been used lawfully because it represents the will of the Home Secretary as a key member of the elected government.

The Court will decide that statute and prerogative powers can co-exist and can be applied interchangeably on a case by case scenario.

The Court will decide that prerogative power can legitimately be used because the statutory power has not yet been implemented.

The Court will decide that the statute should prevail over the prerogative power, even if the statutory provision has not yet been put into implementation.

A

The Court will decide that the statute should prevail over the prerogative power, even if the statutory provision has not yet been put into implementation.

Correct
This is correct because the courts have clearly held that prerogative powers cannot frustrate the will of Parliament – most notably in the case of R v Home Secretary ex p Fire Brigades Union.

88
Q

Which one of the following statements best describes the current position with regard to the reviewability of prerogative powers by the courts?

The courts will accept applications to review the existence, scope and manner of exercise of all prerogative powers, as long as the nature of the power in question is administrative rather than political.

The courts have in theory accepted applications to review the manner in which the government exercises all of its prerogative powers since the GCHQ case. However, jurisdiction is not accepted in practice if the case falls within the excluded areas which Lord Roskill deemed not amenable to judicial review.

The courts will accept applications to review the manner of exercise of all types of prerogative powers. However, they will apply a sliding-scale approach of varying intensity of review dependent upon the subject matter of the case before them.

The courts will accept applications to review the existence and scope of prerogative powers, except in situations where the matter is simply not appropriate for judicial deliberation, such as the appointment of ministers or granting of honours. However, they will apply a sliding-scale approach of varying intensity of review dependent upon the subject matter of the case before them.

The courts will accept applications to review the manner in which the government exercises all of its prerogative powers, except those which are simply not appropriate for judicial deliberation, such as appointing ministers or granting honours or taking military action against an enemy state or organisation. However, if justiciable, the courts will apply a sliding-scale approach of varying intensity of review dependent upon the subject matter of the case before them.

A

The courts will accept applications to review the manner in which the government exercises all of its prerogative powers, except those which are simply not appropriate for judicial deliberation, such as appointing ministers or granting honours or taking military action against an enemy state or organisation. However, if justiciable, the courts will apply a sliding-scale approach of varying intensity of review dependent upon the subject matter of the case before them.

Correct
Correct. This statement captures the current development of this area of judicial review of prerogative power.

89
Q

A company director, born and brought up in the UK, though now living abroad, has recently fallen foul of strict UK regulations about selling security and riot control equipment to a hostile foreign government. The UK government is seeking to extradite him back to the UK on criminal charges. It has now also rejected his application to renew his UK passport, which he made shortly before the charges were issued.

Which one of the following statements best describes the likely approach that the court will take if the director challenges the UK Foreign Office in court over its refusal to issue a new passport?

The court will accept jurisdiction for the case, even though the power to issue passports falls under the broad foreign affairs prerogative, because it is seen as falling at the administrative end of the spectrum and there is therefore no doubt nowadays that is a ‘justiciable’ matter.

The court will not accept jurisdiction for this matter because it involves legal and diplomatic relations with another state, from which the UK is attempting to extradite the director.

The court will not accept jurisdiction for the case because the power to issue passports falls under the broad diplomatic relations and foreign affairs prerogative and this was signposted as a non-justiciable matter by Lord Roskill in the GCHQ case.

The court will accept jurisdiction for the case but, because the power to issue passports falls under the broad foreign affairs prerogative, it will only apply the least intense or lightest touch degree of review.

The court will accept jurisdiction for the case because Lord Roskill specifically stated in the GCHQ case that the issue of passports was a matter that could be seen as administrative in nature and was therefore a justiciable matter.

A

The court will accept jurisdiction for the case, even though the power to issue passports falls under the broad foreign affairs prerogative, because it is seen as falling at the administrative end of the spectrum and there is therefore no doubt nowadays that is a ‘justiciable’ matter.

Correct. Note that this was very much the approach taken by the court in the case of R v Secretary of State for Foreign and Commonwealth Affairs, ex parte Everett.

incorrect
The court will accept jurisdiction for the case but, because the power to issue passports falls under the broad foreign affairs prerogative, it will only apply the least intense or lightest touch degree of review.

This is not a correct answer, as it does not take into account the particular nature of the function being carried out by the Foreign Office when issuing or renewing passports. There is a very broad range of functions carried out in the diplomatic service and the issuing of passports falls at the administrative rather than policy end of the spectrum, meaning that the intensity of review would not be at the low end in this instance. Re-visit your materials on control of the prerogative post-GCHQ and in particular the case of R v SoS for Foreign and Commonwealth Affairs, ex parte Everett.

90
Q

A British citizen was detained by the security services in an authoritarian country three months ago and has since been held in an unspecified location without any legal process. He has been accused of spying by the government of that country. His family has now issued proceedings in the Administrative Court against the UK Foreign Office for what they regard as its failure to represent his interests in trying to secure his immediate release.

Which one of the following statements best describes the likely approach to be taken by the court?

It would consider the matter justiciable and review the policy and actions of the Foreign Office using a relatively high intensity of review.

It would only consider the matter justiciable if the claimant family could rebut the initial presumption that this case fell within the “forbidden area” of international relations.

It would consider the matter justiciable and review the policy and actions of the Foreign Office using the universal standard of review.

It would not consider that this was a justiciable matter, given the international relations context of the case.

It would consider the matter justiciable and review the policy and actions of the Foreign Office using a relatively light intensity of review.

A

It would consider the matter justiciable and review the policy and actions of the Foreign Office using a relatively light intensity of review.

Correct. It is very likely that the modern courts would accept jurisdiction for such a case, as long as a clear basis for legal challenge could be established, but that the intensity of review would be light, given that the subject matter of the dispute relates to diplomatic policy.

incorrect
It would consider the matter justiciable and review the policy and actions of the Foreign Office using a relatively high intensity of review.

This is not the best answer. It is very likely that the modern courts would accept jurisdiction for such a case, as long as a clear basis for legal challenge could be established. However, given that the subject matter of the dispute relates to diplomatic policy and is therefore more of a political than an administrative issue, the intensity of review would be low rather than high. Re-visit your materials on control of the prerogative post-GCHQ and in particular the case of R v SoS for Foreign and Commonwealth Affairs, ex parte Abbasi.

91
Q

You are a trainee with a firm of solicitors. You have been asked to deliver an introductory PowerPoint presentation on European Union law to a delegation of lawyers from the United States. You will need to explain the sources of EU law as part of this presentation.

Which of the following should you refer to as being capable of being legally binding secondary forms of EU legislation in relation to the member states of the European Union? (Choose ONE option only.)

Regulations, judgments of the EU courts, and Opinions.

Directives, the Charter of Fundamental Rights, and Regulations.

Decisions, the Charter of Fundamental Rights, and Directives

Regulations, Directives and Decisions.

Judgments of the EU courts, the Charter of Fundamental Rights, and Opinions.

A

Regulations, Directives and Decisions.

This is correct. They are legally binding secondary legal acts which are provided for by article 288 TFEU. They are classified as legislation where they are made using a legislative procedure (article 289(3) TFEU).

incorrect
Directives, the Charter of Fundamental Rights, and Regulations.

This is incorrect – Directives and Regulations are both legally binding secondary forms of EU legislation. However, the Charter of Fundamental Rights is a primary source of EU law. This is because article 6(1) TEU provides that the Charter has the same legal status as the treaties, which are primary sources.

92
Q

In preparing advice for a client, you have been reading a case in which the court took into account a general principle of EU law. Which ONE of the following provides an accurate definition of the concept of ‘general principles of EU law’?

All legal principles which are addressed to the Member States in general.

Legal principles which produce legal effects on persons identified in a generalised and abstract manner.

Any legal principles which are expressed in broad or abstract terms.

Legal principles laid down by such acts as declarations, notices, programmes, and resolutions.

Overarching legal principles which apply generally across EU law.

A

Overarching legal principles which apply generally across EU law.

This is correct – The concept of ‘general principles of EU law’ refers to a particular category of overarching legal principles which apply generally across EU law. They are used to aid the interpretation of the more specific rules of EU law, to assess the lawfulness of the activities and secondary legislation of the European Union, and to determine the lawfulness of those activities of the Member States which fall within the scope of EU law.

incorrect
All legal principles which are addressed to the Member States in general.

This is incorrect – This is not an accurate definition of the concept of ‘general principles of EU law’. It fails to capture the central characteristic of general principles of EU law. Moreover, not all legal principles which are directed at the Member States in general are classified as general principles of EU law. There are also general principles of EU law which are not directed solely to the Member States.

93
Q

You are advising a client who wishes to bring a legal action in relation to a dispute concerning employment rights. The dispute concerns events that happened two months ago. The subject matter is not covered by the Withdrawal Agreement, but it does raise issues relating to direct EU legislation. In relation to that, you would like to rely upon the following: the judgment in a case decided by the Court of Justice six months ago, a general principle of EU law which was first established by the Court of Justice in the 1970s, and the Charter of Fundamental Rights.

Which, if any, of them will be binding on the High Court of England and Wales? (Choose ONE option only.)

Only the Charter of Fundamental Rights will be binding.

Both the general principle of the EU law, and the Charter of Fundamental Rights will be binding.

Both the judgment in the case, and the Charter of Fundamental Rights will be binding.

Both the judgment in the case, and the general principle of EU law will be binding.

Only the general principle of EU law will be binding.

A

Only the general principle of EU law will be binding.

This is correct– The High Court continues to be bound by the general principle of EU law as this had effect in EU law immediately before the end of the transition period on 31 January 2021 (ss.6(3) & 6(7) EUWA 2018). However, the High Court is not bound by the judgement in the case decided by the Court of Justice six months ago as this was after the end of the transition period, although they may have regard to that judgment (s.6(1)(a)). Neither is the High Court bound by the Charter of Fundamental Rights as under s.5(4) of the EUWA, the Charter has not been part of UK law since the end of the transition period.

incorrect
Only the Charter of Fundamental Rights will be binding.

This is incorrect – Under s.5(4) of the EUWA 2018, the Charter has not been part of UK law since the end of the transition period on 31 January 2021. It is not binding on the High Court as a result. In contrast, while the High Court is not be bound by the judgment in the case decided by the Court of Justice six months ago as this was decided after the end of the transition period (s.6(1)(a)), it continues to be bound by the general principle of EU law as that principle had effect in EU law immediately before the end of the transition period (ss.6(3) & 6(7)).

94
Q

You are advising a client who is a national of an EU member state and, therefore, is a Union citizen under EU law. She has resided in the London since 2011. She has sought your advice about an Act of Parliament which was enacted two months ago. She maintains that the Act of Parliament discriminates against her and other Union citizens in relation to their employment rights. Your research has led you to conclude that this falls within the scope of an EU Regulation which has been preserved by the Withdrawal Agreement. You conclude that a court in England and Wales will be likely to find that the Act is incompatible with that EU Regulation.

If a court in England did find that the Act is incompatible, what would the court be required to do? (Choose ONE option.)

The court would be required to issue a declaration of incompatibility which formally notifies Parliament that the Act of Parliament is incompatible with EU law.

The court would be required to refer the issue to the Supreme Court as only the Supreme Court is able to rule on when an EU Regulation will prevail over the Act of Parliament.

The court would be required to invalidate the Act of Parliament as it is incompatible with the EU Regulation.

The court would be required to apply the Act of Parliament as the principle of the supremacy of EU law no longer applies in the United Kingdom.

The court would be required to disapply the Act of Parliament as it is incompatible with the EU Regulation.

A

The court would be required to disapply the Act of Parliament as it is incompatible with the EU Regulation.

This is correct – The issue concerns the rights of a Union citizen who was resident in the United Kingdom at the end of the transition period. The Withdrawal Agreement provides for some EU law to continue to apply in relation to such Union citizens and, indeed, the question confirms that the issue falls within the scope of an EU Regulation which was preserved by the Withdrawal Agreement. As it is preserved by the Withdrawal Agreement, it is governed by s.7A of the EUWA 2018. This, together with s.5(7) of the EUWA 2018, ensures that the supremacy of EU law will continue to apply in this situation. Under that principle, where national law is incompatible with EU law, the national courts are required to disapply national law (Simmenthal; see also the House of Lords in ex p. Factortame).

incorrect
The court would be required to issue a declaration of incompatibility which formally notifies Parliament that the Act of Parliament is incompatible with EU law.

This is incorrect – Nothing in EU law, the Withdrawal Agreement or the European Union Withdrawal Acts provides for such a remedy in these circumstances.

95
Q

You have been asked to advise a client who is a UK national and has resided in Manchester for the whole of his life. He has sought advice about an Act of Parliament which was enacted last month. He maintains that the Act of Parliament has undermined his right to privacy in relation to the internet. You are aware that this right was protected by an EU Regulation enacted before the United Kingdom left the European Union. You conclude that a court in England and Wales is likely to find that the Act of Parliament is incompatible with that EU Regulation. However, the EU Regulation has not been preserved by the Withdrawal Agreement.

If a court in England did find that the Act is incompatible with the EU Regulation, what would that court be required to do? (Choose ONE option.)

The court would be required to apply the Act of Parliament as the principle of the supremacy of EU law no longer applies in this context in the UK.

The court would be required to issue a declaration of incompatibility which formally notifies Parliament that the Act of Parliament is incompatible with EU law.

The court would be required to invalidate the Act of Parliament as it is incompatible with the EU Regulation.

The court would be required to apply the Act of Parliament as the EU Regulation will have ceased to have effect in the United Kingdom following Brexit.

The court would be required to disapply the Act of Parliament as it is incompatible with the EU Regulation.

A

The court would be required to apply the Act of Parliament as the principle of the supremacy of EU law no longer applies in this context in the UK.

This is correct – The EU Regulation forms part of the bulk of EU law which is not preserved by the Withdrawal Agreement. It will be classified as Direct EU Legislation under s.3 EUWA 2018 and, as such, will form part of the EU law retained by ss. 3 to 4 of the EUWA 2018. In relation to that retained EU law, s.5 of the EUWA 2018 specifies that the principle of the supremacy of EU law does not apply to any domestic law made after the end of the transition period. The Act of Parliament in the question was clearly passed last month and therefore after the end of that period. It can therefore be amended or repealed by any of the methods in s.7 and Sch.8 of the EUWA 2018. This includes by an Act of Parliament.

96
Q

Which of the following statements best describes the operation of the margin of appreciation doctrine?

The primary responsibility for the protection of Convention rights lies with the contracting states and consequently, they are allowed a degree of discretion when taking legislative, administrative, and judicial actions that restrict Convention Rights. The doctrine relies upon the premise that the ECHR scheme should respect the contracting states’ different political, social, and cultural traditions, which are best understood by the states themselves. A wider margin of appreciation will be afforded to contracting states in cases involving, for example, questions of morality and national security. However, state discretion is not unlimited, and it is subject to the ultimate supervision of the ECtHR.

The ECtHR has found that, as state authorities have a better knowledge than it does of the political, social, and cultural traditions that influence their countries, they should be afforded absolute discretion with respect to determining the level of state interference with Convention rights where that involves, for example, questions of morality and national security.

The ECtHR has a clear supervisory role over the legislative, administrative, and judicial actions and decisions of the contracting states, and it will only be in rare cases involving high-level policy decisions that the court will allow considerable discretion to the contracting state to determine whether the relevant interference was justified.

Where a case concerns sensitive policy areas, for example: questions of morality, national security or the economy, the ECtHR will withdraw its supervision over whether the relevant decision was a proportionate interference with the person’s Convention rights.

The margin of appreciation doctrine provides that the contracting states should be afforded a degree of discretion when taking judicial decisions because the domestic courts are best placed to understand their countries’ political, social, and cultural traditions. A wider margin of appreciation will be allowed in cases involving, for example, questions of morality or national security, although the discretion is not limitless, and the contracting states are subject to the supervision of the ECtHR.

A

The primary responsibility for the protection of Convention rights lies with the contracting states and consequently, they are allowed a degree of discretion when taking legislative, administrative, and judicial actions that restrict Convention Rights. The doctrine relies upon the premise that the ECHR scheme should respect the contracting states’ different political, social, and cultural traditions, which are best understood by the states themselves. A wider margin of appreciation will be afforded to contracting states in cases involving, for example, questions of morality and national security. However, state discretion is not unlimited, and it is subject to the ultimate supervision of the ECtHR.

Correct. This statement is an accurate synopsis of key aspects of the margin of appreciation doctrine developed by the Strasbourg court.

97
Q

You see a new client who is being held at an immigration detention centre. He recently found out that all of his personal letters and legal correspondence were being opened and read by detention officers.

You discover that the immigration detention centre is being run by a company called ForceCorps Ltd. It has entered into contracts with the Home Office to provide immigration detention and transport services. The (fictitious) Contracts for Immigration Services Act 2015 regulates these services and sets out the powers of detention officers which include powers to detain, and to use reasonable force to restrain.

Your client wants to challenge the interference with his correspondence (which engages Article 8 ECHR). Which of the following summarises the most accurate advice to your client on the procedural question of whether and how ForceCorps Ltd is operating as a public authority for the purpose of section 6(1) HRA 1998?

As a private company, ForceCorps has a defence to potential liability under section 6(5), HRA 1998, and therefore it will not be liable for any of its actions.

The court is likely to find that ForceCorps is a functional public authority because it is inherently, by its very nature, a public authority.

As a private company, ForceCorps Ltd does not come under the remit of the HRA 1998 at all.

It is very likely that ForceCorps Ltd, though a private company, would be treated as a functional or hybrid public authority in this context, as it is clearly exercising a public function with respect to the provision of immigration detention services.

ForceCorps is acting as a functional/hybrid public authority because it is performing public functions and therefore, it is potentially liable for all of its actions under s. 6 HRA 1998.

A

It is very likely that ForceCorps Ltd, though a private company, would be treated as a functional or hybrid public authority in this context, as it is clearly exercising a public function with respect to the provision of immigration detention services.

Correct. ForceCorps would be seen in this context as a functional or hybrid public authority because of the nature of the function it is carrying out in administering immigration services under statutory powers. It is effectively taking the place of central government in delivering these services. Refer back to the key case of Aston Cantlow.

98
Q

You are contacted by a new client, a recently established group called ‘Stop Unfair Intrusion Now’ (‘SUIN’), which lobbies on surveillance issues. Its stated aim is to ‘roll back the surveillance state’. The charity is seeking advice in relation to a possible challenge it wishes to bring against the government.

This possible challenge relates to a family which has made contact with SUIN. The family believe that they have been placed under surveillance by their local council because of claims made to the council that the family were not living in their child’s school catchment area when they applied for her school place and continue not to live at their claimed address. SUIN claims that this covert surveillance violates the family’s article 8 rights. SUIN tells you that, as far as the family is aware, the surveillance started around 14 months ago and it is ongoing.

On the basis of the information you have, which of the following statements summarises the best advice to your clients in relation to whether SUIN will have standing to bring this challenge under the HRA, and in relation to HRA claim time limits with reference to their case?

Pursuant to s. 7(5) HRA, 1998, HRA claims must be brought within one year from when the act complained of took place. As the interference with the family’s article 8 rights started over one year ago, any claim by SUIN under the HRA is now out of time.

SUIN will not have standing to bring a challenge under the HRA because Convention rights can only attach to natural persons.

SUIN will not have standing to challenge the surveillance activity because it is not directly affected by this activity as an organisation in its own right; it is only interested in the general policy behind it.

It is likely that SUIN will have standing to challenge the surveillance activity under Art 8 HRA 1998 because the family are directly affected victims.

SUIN will be able to bring a claim under the HRA because although the surveillance commenced over one year ago, a claim will still be in time because the interference with rights, in the form of surveillance, is still ongoing.

A

SUIN will not have standing to challenge the surveillance activity because it is not directly affected by this activity as an organisation in its own right; it is only interested in the general policy behind it.

Correct. This statement reflects the test for standing to bring a claim under the HRA 1998 which is that the applicant must be a directly affected victim of an interference with one of its Convention rights. Refer to sections 7(1) and 7(7), HRA 1998, article 34 ECHR, and the case of Klass v Germany. Under the HRA, the case would need to be brought by individuals affected by the surveillance activity, in other words, by the family.

99
Q

You are advising a client who wishes to challenge the refusal of the Registrar of Births, Deaths and Marriages to register the identity of her deceased partner in accordance with her partner’s chosen gender (which was genderfluid), as opposed to their biological sex.

Your client informs you that her partner was born female, but several years ago they disclosed that they had always felt that their true identity encompassed both female and male attributes. When the client’s partner was diagnosed with a terminal illness, her partner expressed a clear wish for the death certificate to record their gender identity instead of their biological sex.

You have already advised your client that it is arguable that the statutory scheme which prevents the Registrar from registering her deceased partner as genderfluid rather than female is incompatible with articles 8 and 14, ECHR. As far as you are aware, the Gender Recognition Act 2004 does not assist your client as that recognises the legal right to change gender from female to male and vice-versa, not to legally identify as ‘genderfluid’.

Subject to the court accepting the incompatibility argument, which of the following statements reflects the best advice to your client on the likely approach by the court to its powers under sections 3 and 4, HRA and its reasons?

It is not likely that the court will decide to use s. 3 power to read the incompatible legislation in a Convention-compatible way because of the policy context of the case, including its potentially broad ramifications outside of the issue of the registrations of deaths. Therefore, the court will have to make a declaration of incompatibility.

Whilst the court will acknowledge that the s. 3 duty is a strong one, it is more likely that it will decide it is unable to use s. 3 to read the legislative scheme in a Convention-compatible way and, instead, it may grant a declaration of incompatibility. This is because the case involves matters of complex policy which the court may well feel it is not the appropriate body to deal with.

The court is unlikely to use its section 3 power because of the policy context of the case, and there are no further steps it can take beyond that.

The court is likely to use its s. 3 power to read the incompatible provisions in a Convention-compatible way. The s. 3 duty is a strong one which should be used wherever it is possible to do so. Here, it would be possible to read down the legislation in a way that provides for a broader understanding of gender identity.

The strength of the s. 3 duty is such that the court would be required to use its s. 3 power to read the incompatible legislation in a Convention-compatible way. In addition, the court could also grant a declaration of incompatibility.

A

Whilst the court will acknowledge that the s. 3 duty is a strong one, it is more likely that it will decide it is unable to use s. 3 to read the legislative scheme in a Convention-compatible way and, instead, it may grant a declaration of incompatibility. This is because the case involves matters of complex policy which the court may well feel it is not the appropriate body to deal with.

Correct
Correct. Although the approach of the courts is that they will clearly try to use s. 3 wherever possible, they have identified limits to the proper use of s. 3. In Bellinger Lord Nicholls set out the reasons why the court could not use its s.3 power in that case, which are arguably analogous to the facts here.

Whilst the Gender Recognition Act, which post-dates Bellinger, provides rights to be legally identified as a gender that is different from one’s biological sex, it does not deal with the range of gender identities that fall outside of male and female and are sometimes grouped together under umbrella descriptions such as ‘non-binary’. The court may decide to make a declaration under s. 4, therefore.

incorrect
The court is likely to use its s. 3 power to read the incompatible provisions in a Convention-compatible way. The s. 3 duty is a strong one which should be used wherever it is possible to do so. Here, it would be possible to read down the legislation in a way that provides for a broader understanding of gender identity.

This is not the best answer. Whilst it is accurate to state that the s. 3 power is a strong one, this answer fails to acknowledge the limits of s. 3 and the policy context of the case. In Bellinger, Lord Nicholls set out the reasons why the court could not use its s.3 power in that case, which are arguably broadly analogous to the facts here.

100
Q

Which one of the following statements is NOT correct in relation to the various remedial effects of the HRA?

Under section 8 HRA the domestic courts can apply a wide range of remedies including damages, declarations of the law, injunctions, and an array of other public law orders.

In determining whether a statutory provision is compatible, the court will simply consider and give weight to any declaration the relevant minister may have made under s. 19 HRA.

If a declaration of incompatibility is made in a case, that will not necessarily mean that the claimant will never get a substantive remedy, as s/he will still be able to take their case to the European Court of Human Rights for a final ruling.

If a court uses its s. 3 power to interpret a statutory provision in a Convention-compatible way, this will have the effect of nullifying any s. 6(2) defence a public authority otherwise would have had.

If a declaration of incompatibility is made in a case, the claimant will not get an immediate substantive remedy in a domestic court because of the operation of the s. 6(2) defence.

A

In determining whether a statutory provision is compatible, the court will simply consider and give weight to any declaration the relevant minister may have made under s. 19 HRA.

Correct. This is not an accurate description of the status of a s. 19 declaration. Whether a particular provision is ultimately seen as compatible with the relevant Convention right is a judgement call for the courts and not for the minister sponsoring the original legislation.

101
Q

You are acting for a new client whose daughter was killed by armed police. She was a member of the group, ‘Arm4Rights’, which presents itself as a peaceful lobbying organisation, campaigning for an end to the UK’s sale of arms to foreign regimes with poor human rights records. The police say that Arm4Rights is a militant organisation and claim they had credible intelligence that the group intended to attack the Foreign Secretary.

She attended a Members of Arm4Rights demonstration outside the Foreign Office. As the Foreign Secretary left to get into a waiting car, your client’s sister waved a toy gun in the air as she and the other Arm4Rights members chanted: ‘Feel a British bullet!’ Because of the noise, she did not hear warnings from armed police to ‘put down your gun’, and she was shot and killed.

Your client has now been told that the matter has been closed because the police officer in charge of policing the demonstration had investigated the incident and concluded that there was no fault on the part of the police. Your client wants to know if there is anything he can do to challenge the authorities over his daughter’s death.

Which one of the following represents the best advice to your client on the prospects of bringing an action in relation to article 2 ECHR?

The state has a duty under article 2 to refrain from using lethal force except in very particular circumstances. The police are likely to argue that it was absolutely necessary to use such force in this situation in order to protect the Foreign Secretary from unlawful violence. Your client may have a reasonably strong argument that the state has failed in its investigative duty under article 2.

The state has a duty under article 2 to refrain from killing. However, it is very likely that the police could justify having done so as your client’s daughter was behaving in an alarming way. The matter has been investigated and therefore the state has complied with its obligations under Article 2.

The state has a duty under article 2 to refrain from killing. As this is an absolute right, given the sanctity of life, there is a very strong case to put forward that there was a breach of article 2.

Since the police action was to defend the Foreign Secretary from unlawful violence, the state action in this case is justified. However, it appears that the state has failed to fully comply with its investigatory duties in this case.

Where an incident involves a death at the hands of the police, the courts will be very cautious about intervening. The only relevant consideration for the court will be whether the police have complied with the investigatory obligation under Art 2 and there appears to be no issue about that on these facts.

A

The state has a duty under article 2 to refrain from using lethal force except in very particular circumstances. The police are likely to argue that it was absolutely necessary to use such force in this situation in order to protect the Foreign Secretary from unlawful violence. Your client may have a reasonably strong argument that the state has failed in its investigative duty under article 2.

Correct. This answer best reflects the law under article 2 and the likely merits of the client’s case. The police could argue that they were justified in taking life in this situation, but your client is likely to be able to successfully argue that the state has failed in its investigative duty. The investigation must be public, independent and involve the full participation of the family. At least two of these requirements appear to have been breached. Refer to the case of R (Amin).

102
Q

On arrival at Gatwick Airport, your clients – a mother with two boys aged 13 and 16 –applied for asylum in the UK. After an initial period of assessment by the National Asylum Support Service (‘NASS’), the family were ‘dispersed’ to a UK city around 250 miles away pending a decision on their asylum application. They are housed by NASS in temporary local authority accommodation in a large block of flats in poor condition. Within a few weeks, the family began to complain of sustained harassment and threats by two other families in the same block. This harassment, which includes damage to the outside of their flat and severe noise disturbance, is based on the family’s nationality and ethnicity. A local gang tried to recruit the 16-year-old but he refused to be associated with it. You have now heard that he has been stabbed by another pupil at his school. The family are very anxious and wish to be moved.

Which of the following represents the best legal advice based on article 3 of the ECHR?

You explain to the family that, although the harassment is very unfortunate and must be reported to the police, there is no obligation here on NASS or the local authority to move the family, as the only form of positive obligation falling on states is in relation to attempts to deport or extradite people to countries which expose them to risk of torture or inhuman and degrading treatment in those countries.

You explain to the family that, although the harassment is very unfortunate and must be reported to the police, the seriousness of the harm to them is not sufficient for article 3 to be engaged, because the definition of torture requires sustained and systematic violence. They will therefore have no remedy under the HRA in relation to the ECHR.

You explain to the family that NASS as a public authority has a positive obligation on it to take reasonable measures to prevent people under its care from being exposed to the risk of inhuman and degrading treatment at the hands of third parties. This is not an absolute duty but, if the extent of the harm to the family reaches this threshold, article 3 will be engaged and, if NASS had actual or constructive knowledge of what was happening, legal action on the basis of article 3 could result in an order that the family be moved to more suitable accommodation.

You explain to the family that, although the harassment is very unfortunate and must be reported to the police, there is no obligation here on NASS or the local authority to move the family, as the alleged harm to them has been brought about by third parties. As there is no liability on a public authority, it will not be possible to take action under the HRA in relation to article 3.

A

You explain to the family that NASS as a public authority has a positive obligation on it to take reasonable measures to prevent people under its care from being exposed to the risk of inhuman and degrading treatment at the hands of third parties. This is not an absolute duty but, if the extent of the harm to the family reaches this threshold, article 3 will be engaged and, if NASS had actual or constructive knowledge of what was happening, legal action on the basis of article 3 could result in an order that the family be moved to more suitable accommodation.

Correct. This is the best answer as it explains the operation of the positive obligation in this kind of context.

103
Q

A ‘deprivation of liberty’ is required in order to engage article 5(1), ECHR. Which of the following statements best reflects the approach and principles of the ECtHR when determining whether there has been a deprivation?

The starting point should be to consider the concrete situation of the person who is subject to the measures that limit their liability, before concentrating on how long the measures were applied for.

In assessing whether measures amount to a deprivation of liberty the court will primarily consider the degree of physical resistance put up by the affected parties.

There is a presumption, when measures taken do not involve a formal arrest, that article 5(1) protection is not engaged at all.

The starting point should be to consider the concrete situation of the person who is subject to the measures that limit their liberty. Whether these measures amount to a deprivation rather than a restriction will depend upon the degree of their intensity. Relevant factors to this assessment include the type, duration, effects, and manner of implementation of the measure in question.

The approach of the court in assessing whether measures amount to a deprivation of liberty is to ask whether the measures in question are a proportionate means of achieving the state’s aims.

A

The starting point should be to consider the concrete situation of the person who is subject to the measures that limit their liberty. Whether these measures amount to a deprivation rather than a restriction will depend upon the degree of their intensity. Relevant factors to this assessment include the type, duration, effects, and manner of implementation of the measure in question.

Correct. This statement reflects the principles established by the ECtHR in the cases of Engel and Guzzardi.

104
Q

Your client was recently arrested on suspicion of having been involved in an overnight break-in at a pharmacist’s shop.

He was arrested on the basis of intelligence information the following morning and taken to the local police station at 10am (on a Monday). He was then interviewed on several occasions over the period of the next 3 days. He was finally released from police custody at midday on the Thursday without charge, once the police had completed all their investigations.

Which of the following represents the best advice to your client on the prospects of bringing an action in relation to article 5(3) ECHR?

Your client would have an arguable but not particularly strong case that there had been a violation of his article 5(3) right. Even though the ECtHR has accepted that the police may in some situations detain a suspect in custody for up to four days, shorter periods can still be seen as incompatible, depending on the context of the case. As the suspected offence is a serious one and the investigation appears to have been protracted, it is unlikely that 74 hours would be seen as excessive in terms of article 5(3).

There would have been no breach of article 5(3) in this situation as the ECtHR has specifically approved detentions by the police for any period up to 4 days. This was the relevant period of custody in the leading case of McKay v UK and so the same precedent would apply in this case.

There was a clear breach of article 5(3) in this situation as any extension of custody in excess of the basic time limit of 24 hours allowable under the Police and Criminal Evidence Act (‘PACE’) will not be seen as compatible with the standard in the ECHR.

There would have been no breach of article 5(3) in this situation as the ECtHR has approved detentions for up to 4 days maximum in police custody for all suspects, other than minors.

Your client would have a reasonably strong case that there had been a violation of his article 5(3) right. Even though the ECtHR has accepted that the police may in some situations detain a suspect in custody for up to four days, shorter periods can still be seen as incompatible, depending on the context of the case. It is unlikely in this context, involving a significant but not a violent, armed or terrorism-related crime, that a court would accept that a detention of 74 hours without any form of judicial oversight was justified.

A

Your client would have a reasonably strong case that there had been a violation of his article 5(3) right. Even though the ECtHR has accepted that the police may in some situations detain a suspect in custody for up to four days, shorter periods can still be seen as incompatible, depending on the context of the case. It is unlikely in this context, involving a significant but not a violent, armed or terrorism-related crime, that a court would accept that a detention of 74 hours without any form of judicial oversight was justified.

Correct. This summarises the best advice. Refer to the case of McKay v UK in particular. Note in that case that a significantly shorter period - of 36 hours - was seen to fall within the art 5(3) right.

105
Q

You are acting for a client who was arrested on suspicion of aggravated burglary. Following his arrest, he was detained at the police station for 24 hours and during this time he was questioned on three occasions. Your client repeatedly asked to see a legal representative, but this was refused, and a police officer told him, “You’ll just get some bent lawyer who will warn your gang that we’re onto them”. After the 24-hour period your client was released without charge. He now wishes to know if he can challenge the decision of the police not to allow him access to a lawyer.

Which of the following statements represents the most accurate advice to your client on the prospects of bringing an action in relation to article 6 ECHR?

The client cannot challenge the decision of the police to deny him access to a lawyer because they provided a good reason as to why access was withheld.

The client cannot bring a legal challenge against the police on this basis, because following his arrest and detention he was not charged with an offence.

The client can challenge the decision because everyone has the right to access free legal representation when they have been arrested.

The client cannot challenge the police’s decision to deny him access to a lawyer because there is no absolute right to legal representation.

There is good merit in challenging this decision under articles 6(1) and 6(3)(c) because the police’s justification for denying the client access to a lawyer did not amount to a good reason for doing so.

A

The client cannot bring a legal challenge against the police on this basis, because following his arrest and detention he was not charged with an offence.

Correct. In Murray v UK, the ECtHR clarified that article 6 can apply to the whole criminal process, including the pre-trial stage from the point of arrest. However, this is contingent upon the applicant being charged with an offence. On out facts, the client was released without charge and therefore, his article 6 rights were not engaged. The requirement for a charge to have made is clear in the wording of all three parts of article 6.

incorrect
There is good merit in challenging this decision under articles 6(1) and 6(3)(c) because the police’s justification for denying the client access to a lawyer did not amount to a good reason for doing so.

Incorrect
Incorrect. Whether or not the police’s reason amounts to ‘good cause’ (Murray v UK’) is academic. The client’s article 6 rights were not engaged because he was not charged with an offence. In Murray v UK, the ECtHR clarified that article 6 can apply to the whole criminal process, including the pre-trial stage from the point of arrest. However, this is contingent upon the applicant being charged with an offence.

The client cannot challenge the decision of the police to deny him access to a lawyer because they provided a good reason as to why access was withheld.

Incorrect. The police did provide a reason but whether it is ‘good cause’ (Murray v UK’) or not to have denied the client access to legal representation is academic. His article 6 rights were not engaged because he was not charged with an offence. In Murray v UK, the ECtHR clarified that article 6 can apply to the whole criminal process, including the pre-trial stage from the point of arrest. However, this is contingent upon the applicant being charged with an offence.

106
Q

Which of the following statements best describes the scope of the protection given to “private life” under article 8 ECHR?

The right to private life solely concerns the right to privacy with respect to searches of the person and surveillance in its many forms by the state.

Private life is a broad and evolving concept which covers a variety of interests and human characteristics, including a person’s physical, mental, and moral integrity.

The right to private life has been definitively classified as covering the following interests: physical, mental, and moral integrity; sexual orientation; gender recognition; searches of the person and surveillance by the state.

The right to private life is the only one of the four protected interests under article 8 which is applied in absolute terms by the ECtHR.

The concept of private life has been given a fixed meaning by the ECtHR which can be described as relating to a person’s physical, mental, and moral integrity.

A

Private life is a broad and evolving concept which covers a variety of interests and human characteristics, including a person’s physical, mental, and moral integrity.

Correct. This statement accurately reflects the nature and scope of ‘private life’ under article 8 and provides an example of one of the categories that is protected under private life.

107
Q

You are acting for a client who is serving a prison sentence. He has recently discovered that the letters you send to him and the letters he sends to you are being opened and read by prison officers. He is worried about this, as you are advising him on an appeal against a recent conviction for assaulting one of the prison officers. When he asked a prison officer about the opening of his letters, he was told: ‘The Governor has told us that we can read your letters. You’re trouble and she wants to know what you’re up to’. You discover that, in principle, the Governor does have the power to open letters in accordance with policy issued under the relevant statutory framework.

Which of the following statements represents the best advice to your client in challenging the opening of his letters, in relation to article 8 ECHR?

There is good merit in a challenge because the courts have found that a high degree of protection applies to prisoners’ communications with their legal advisors, and privileged legal correspondence cannot be opened in any circumstances.

It is unlikely that the client can successfully challenge the decision to open and read his letters because the courts allow a wide discretion to public authorities to decide what amounts to a proportionate mechanism to achieve a legitimate aim.

There is good merit in a challenge because the courts have found that a high degree of protection applies to prisoners’ communications with their legal advisors. Privileged legal correspondence should only be opened where there is reasonable cause to believe it contains a prohibited item, and letters should only be read where the authorities have reasonable cause to believe that the legal privilege is being abused. There is nothing to suggest that these circumstances apply to the client.

The client will not be able to successfully challenge the opening of his letters because the facts indicate that this interference with his correspondence by the prison authorities is in accordance with the law.

There is little merit in a challenge against the governor’s decision to open and read the client’s letters because it is likely that a court would find such an interference to be proportionate with the legitimate aim of the prevention or disorder of crime, particularly as the client was convicted of an offence that happened whilst he was in prison.

A

There is good merit in a challenge because the courts have found that a high degree of protection applies to prisoners’ communications with their legal advisors. Privileged legal correspondence should only be opened where there is reasonable cause to believe it contains a prohibited item, and letters should only be read where the authorities have reasonable cause to believe that the legal privilege is being abused. There is nothing to suggest that these circumstances apply to the client.

Correct. This statement accurately reflects the ECtHR’s position in Campbell v UK, that the opening of prisoners’ privileged legal correspondence will be a disproportionate interference, unless the particular circumstances identified in Campbell apply. Refer also to the domestic case of Daly.

108
Q

You see a new client, an author, who has written a book about a far-right political group. The publishing company has told him that it has been banned from releasing the book because the Home Office considers that it disseminates and endorses unacceptable views held by the organisation. The organisation has called for direct action against some migrant communities in the UK, and it has increasingly engaged in organised violent protests. Further, the client has just found out that he has been charged with a criminal offence on similar grounds.

Your client is shocked. He tells you that he is a well-known, respected journalist and he has no affiliation to the group or its ideas. On the contrary, he wrote the book to try to expose the offensive nature of the group’s views, although this necessarily involved setting out those views in order to criticise them.

Which of the following statements represents the best advice to your client on the prospects of challenging the book ban, and any prosecution, under article 10 ECHR?

The client has good prospects of success as the ECtHR has made it clear that article 10 is capable of protecting expression that offends, shocks, or disturbs.

Even if the client can show that his book is a work of proper, critical journalism, it is unlikely that the measures will be found to be disproportionate because courts afford broad discretion to state authorities to decide what measures against journalists are necessary in order to achieve a legitimate aim.

If it can be demonstrated that the client’s book is a work of proper, critical journalism which, whilst discussing and evaluating the group’s views, does not propound or endorse them, there is a reasonably good prospect that a court would find that the authorities’ measures are a disproportionate interference with his right to the freedom of expression.

The client’s prospects are poor because the courts have found that the right to freedom of expression cannot be used to protect views which are themselves, fundamentally incompatible with the Convention.

The client has a very good prospect of successfully challenging the state’s measures because the courts will never find that such measures are proportionate in the context of journalistic freedom of expression.

A

If it can be demonstrated that the client’s book is a work of proper, critical journalism which, whilst discussing and evaluating the group’s views, does not propound or endorse them, there is a reasonably good prospect that a court would find that the authorities’ measures are a disproportionate interference with his right to the freedom of expression.

Correct. This statement is largely analogous with the ECtHR’s approach in Jersild v Denmark. The domestic courts and the ECtHR have recognised that journalistic freedom of expression should be afforded a higher degree of protection than some other areas which do not contribute to credible public debate.

109
Q

A story concerning a former Cabinet minister and now member of the House of Lords has been published in a national newspaper. It reveals that he is the father of a child, now 12 years old, who has a severe genetic condition requiring her to have constant care and medical attention. The child was born as a result of an extra-marital affair he had at the time, which he has kept from the public eye. The story alleges that he has ignored the child and her mother completely since the pregnancy and has not paid anything towards the child’s care. The child’s mother has not cooperated in the disclosure of the story and, so far, the newspaper has not disclosed her or her child’s identity, though it is saying it will do so. The former minister wishes to obtain an injunction against any further publication of details relating to the child (and further indicates that he intends in due course to sue the newspaper for damages).

Which of the following represents the best advice in relation to his case for obtaining an injunction against further publication?

As a child is involved, the court will find in favour of the former minister and protect his article 8 rights. It will grant an injunction against any form of future publication on the matter.

The court is likely to consider that although the story may be considered to be in the public interest because the former minister is a significant public figure, in relation to the further information about the mother and child, it is likely that an injunction may be granted in order to protect the privacy of the child in particular.

The minister has no reasonable expectation of privacy because he has performed, and continues to perform, a public function. His behaviour has fallen below that which expected of a public servant. His application for an injunction is therefore likely to fail.

Section 12(4) of the Human Rights Act requires that priority be given to freedom of expression over all other rights, as it refers specifically to the ‘particular regard’ that must be paid to freedom of expression. Therefore, it is highly unlikely that the former minister would achieve any success in his legal actions.

It is unlikely that the court will grant any form of injunction to the former minister, as this is a discretionary measure only awarded in the most deserving of cases.

A

The court is likely to consider that although the story may be considered to be in the public interest because the former minister is a significant public figure, in relation to the further information about the mother and child, it is likely that an injunction may be granted in order to protect the privacy of the child in particular.

This is the best answer. The courts will pay special attention to the privacy rights of children, as indicated in cases such as Murray v Express and Re S (A child).

110
Q

A prisoner, who is serving a sentence for armed robbery, wishes to have a baby with his long-term partner, using artificial insemination. The prison governor has agreed, after fully reviewing the case. The independent regional TV news channel for the couple’s hometown has discovered from a source in the prison that this is about to happen. They approach his partner, asking for comments from her and ‘justification’ for why this should be allowed. It is apparent that they have filmed her and her house and that they will shortly broadcast the material and the information as a five-minute feature on local news.

Which of the following represents the best advice to her if she seeks advice on attempting to restrain the story in the interests of protecting her and her partner’s privacy?

She has a reasonable case for arguing that broadcasting of the filming and her identity should be restrained, as the matter engages her and her partner’s private and family life in a particularly acute way. The news channel may be able to argue that the story is in the public interest and therefore that publication of the basic information may be appropriate. However, unless it can show a strong public interest justification for disclosing the identity of the couple, it is unlikely that the film footage of her and of their house will be allowed, as this will give away her identity.

The story engages the privacy rights of her and her partner so acutely that the court would almost certainly find in their favour and ban the broadcast of the feature and any associated factual reporting relating to the story.

The court would regard this story to be a matter of notable public interest, as issues connected to the rights and conditions of prisoners are justifiably part of the public debate. It is irrelevant therefore how or where the news channel obtained this information. She would not have any reasonable prospect of being able to restrain this story, therefore.

She would not be able to argue that her or her partner’s article 8 rights could be enforced in the domestic courts as the news channel is not a public authority and so has no duty to act compatibly with Convention rights.

She would have minimal prospects of success as the fact that her partner has been convicted of a violent crime means that he has in reality forfeited any rights he may have enjoyed under the Convention.

A

She has a reasonable case for arguing that broadcasting of the filming and her identity should be restrained, as the matter engages her and her partner’s private and family life in a particularly acute way. The news channel may be able to argue that the story is in the public interest and therefore that publication of the basic information may be appropriate. However, unless it can show a strong public interest justification for disclosing the identity of the couple, it is unlikely that the film footage of her and of their house will be allowed, as this will give away her identity.

This is the best answer. The actual information concerning the proposal to allow artificial insemination for a serious offender may well be considered to be in the public interest. The privacy interests (and possibly the safety) of his partner could be put at risk, however, if the film and the identification of her and where she lives is broadcast. Medical information is also seen as particularly private – see for instance Campbell v MGN – and the manner in which it has been obtained will be a further factor for the court to weigh up. See also Ashworth Security Hospital case.

111
Q

The issue of standing to bring a judicial review claim has been the subject of developments in case law and legislation. Which of the following statements best summarises the approach of the modern judiciary to this issue?

The courts have adopted a restrictive approach to standing so that only those who are directly affected by the decision will be able to bring a challenge.

Whilst in certain circumstances the courts have adopted a broad interpretation of the ‘sufficient interest’ test, case law demonstrates on the whole that there are hard limits and standing will not be granted to those who have not in some way been personally affected by a decision.

The courts have adopted an increasingly liberal approach to individuals and groups who wish to challenge administrative decisions in relation to matters in which their own interests may not necessarily be directly impacted but in which they nonetheless have a genuine concern about a public issue, which it is important to challenge in order to vindicate the rule of law.

The liberalising trend of the courts has progressed to such a degree that anyone who is found not to be a ‘busybody, crank or mischief-maker’ will be given standing to challenge public law decisions.

Whilst the courts have adopted a liberal approach to standing for associations and pressure groups, they have developed a far more restrictive approach to standing for individuals.

A

The courts have adopted an increasingly liberal approach to individuals and groups who wish to challenge administrative decisions in relation to matters in which their own interests may not necessarily be directly impacted but in which they nonetheless have a genuine concern about a public issue, which it is important to challenge in order to vindicate the rule of law.

Correct. This statement reflects the approach of the courts in cases such as Rees-Mogg, Walton, World Development Movement and Corner House Research.

112
Q

Your client wants to challenge a decision of the (fictitious) English Bowling Assembly (“the Assembly”) to ban him from playing bowls for six months due to unacceptable behaviour. The Assembly was set up by a group of bowling clubs around fifty years ago in order to regulate bowling. The Assembly governs the rules of the game; the codes of conduct under which your client received a sanction; and the relationships between clubs, the Assembly, and other bodies including public bodies. All club members agree to be subject to the rules of the Assembly. It is the sole regulator of bowling in the UK.

Which of the following summarises the most accurate advice to your client on whether the Assembly’s decision is amenable to judicial review?

It is likely that the Assembly’s decision will be amenable to judicial review because its powers associated with the regulation of a sporting activity are governmental in nature.

The decision is not amenable to judicial review because the Assembly’s functions have not been contracted to it by a public body.

It is unlikely that the Assembly’s decision is amenable to judicial review. On the facts, the rules affecting the client come from an agreement between clubs and their members and the Assembly, giving rise to private rights and remedies.

The Assembly’s decision is not amenable to judicial review simply because the Assembly is a private body.

The Assembly’s decision is amenable to judicial review because its functions and powers are public in nature and, if the Assembly did not exist, the government would need to step in to regulate bowling.

A

It is unlikely that the Assembly’s decision is amenable to judicial review. On the facts, the rules affecting the client come from an agreement between clubs and their members and the Assembly, giving rise to private rights and remedies.

Correct. This represents the most likely conclusion and the reason for it. This scenario is broadly analogous to cases such as ex parte Aga Khan involving the Jockey Club.

113
Q

In the case of Privacy International the court revisited ‘total ouster’ clauses. Which of the following statements best represents the key principles from the decision?

As the guardians of the rule of law, the courts will never regard it to be constitutionally appropriate to uphold a total ouster clause.

Access to the courts and the interests of the rule of law takes pre-eminence in all situations over the stated intention of Parliament.

The court held that it was appropriate to depart from the strong presumption of statutory interpretation that Parliament does not intend to exclude judicial review.

It is for the court, not the legislature, to determine the level of scrutiny required by the rule of law. The courts strike an appropriate balance between the inferred intention of Parliament and the rule of law.

It is for the legislature to set down the parameters of the relationship between itself and the rule of law, including legislation concerning access to the courts.

A

It is for the court, not the legislature, to determine the level of scrutiny required by the rule of law. The courts strike an appropriate balance between the inferred intention of Parliament and the rule of law.

Correct. This reflects the judgment of Lord Carnwath in Privacy International.

114
Q

You see a new client who is an educational psychologist. She is employed by the local authority and works in schools. She is paid a salary under a statutory scheme and receives a bonus in accordance with terms in her contract with the local authority. The client tells you that, on review, her salary has been reduced by 10% and the bonus scheme has been suspended for twelve months. On the basis of the information given to you by the client, which is not elaborated here, you consider that there are good grounds to challenge the local authority for the way in which these decisions were made.

The client is seeking advice on the proper mechanism through which to challenge the local authority. She wants to know if she can bring a judicial review claim. Which of the following statements provides the best advice to your client on this question?

The client can challenge the salary and bonus scheme decisions by judicial review. The procedural exclusivity rule, which was established in O’Reilly v Mackman no longer applies, and the courts’ position is that claimants can decide what sort of proceedings to instigate in accordance with their personal preferences.

In accordance with the overriding objective in Part 1 of the Civil Procedure Rules the client has no choice but to proceed by judicial review.

The client can challenge the salary and bonus scheme decisions by judicial review. Whilst her claim involves issues of public and private law the courts no longer rigidly apply the procedural exclusivity rule. The facts do not suggest that the client’s chosen proceedings would flout the general principles contained in Part 1 of the Civil Procedure Rules.

The client must bring the claim by ordinary action rather than judicial review as it contains a private law element.

The client should challenge the salary decision by judicial review and the bonus decision by ordinary action.

A

The client can challenge the salary and bonus scheme decisions by judicial review. Whilst her claim involves issues of public and private law the courts no longer rigidly apply the procedural exclusivity rule. The facts do not suggest that the client’s chosen proceedings would flout the general principles contained in Part 1 of the Civil Procedure Rules.

Correct. This position reflects the modern courts’ approach to mixed public and private law claims in cases such as Clark.

115
Q

Your client, a well-known London football club, has been refused a license to sell alcoholic beverages in its stadium after football games have finished. Subsequently, it has been in protracted correspondence with the local authority which made the decision. The last letter from the local authority, which confirmed that it would not alter its decision, was received by the club ten weeks ago. You discover that the Chairman of the club has meanwhile been away for an eight-week holiday during this period, having told his legal team not to do anything until he got back. The club has not accepted the local authority’s offer to put the matter before the (fictitious) Independent Licencing Ombudsman.

Which of the following statements provides the best advice to your client on the likely difficulties associated with challenging the local authority’s decision?

These are serious procedural issues and so it is highly likely that the court will refuse to grant the club permission to bring a judicial review claim.

It is unlikely that any procedural issues will stop the club obtaining permission to bring a judicial review claim. The club can explain that they were unable to seek legal advice sooner because the Chairman was away, and they can still file the application within the three-month back-stop period. There is nothing in the Civil Procedure Rules that forces the club to engage with the Ombudsman scheme.

There are procedural issues which could affect whether the club is granted permission to bring a claim or whether it ultimately receives a remedy for any errors in the decision-making process. Even if a claim is filed straight away, the court could find ‘undue delay’ and could also object to the club’s failure to engage with the Ombudsman scheme. The degree to which these issues will have an impact is a matter for the court’s discretion.

It is highly likely that the client will be given permission to bring the judicial review claim because it is still within the three-month time limit; the Civil Procedure Rules also do not specifically require that alternative remedies must be exhausted firstly.

There are procedural difficulties. The court could find ‘undue delay’, even if the claim is filed within three months of the date of the letter, and the club should have engaged with the Ombudsman scheme, or at least have an explanation as to why the scheme could not offer a suitable and equal remedy. These issues are likely to impact upon the court’s permission decision, but if permission is granted, they will have no impact upon remedies.

A

There are procedural issues which could affect whether the club is granted permission to bring a claim or whether it ultimately receives a remedy for any errors in the decision-making process. Even if a claim is filed straight away, the court could find ‘undue delay’ and could also object to the club’s failure to engage with the Ombudsman scheme. The degree to which these issues will have an impact is a matter for the court’s discretion.

Correct. It is arguable that the club has not acted ‘promptly’. Whilst there is no technical barrier (in the Civil Procedure Rules) to bringing a claim where alternative remedies might be available, the courts will consider this as a factor at the permission stage.

incorrect
There are procedural difficulties. The court could find ‘undue delay’, even if the claim is filed within three months of the date of the letter, and the club should have engaged with the Ombudsman scheme, or at least have an explanation as to why the scheme could not offer a suitable and equal remedy. These issues are likely to impact upon the court’s permission decision, but if permission is granted, they will have no impact upon remedies.

Incorrect
This is not the best answer. The answer fails to appreciate that, even if a procedural issue is not a barrier to a grant of permission, it can still be a barrier to the grant of an interim or final remedy. Refer back to your materials on the preliminary requirements generally.

116
Q

When a court considers a claim involving a substantive ground of review, including unreasonableness, it will also consider the degree to which it should scrutinise the decision under challenge. Which of the following statements provides the best summary of the courts’ rationale for the ‘intensity of review’ approach?

In distinguishing between matters concerning broader policy questions and those involving fundamental rights, the courts are purely intent on promoting the rule of law.

In deciding the level of scrutiny to apply to cases the courts are ensuring that a fair decision is reached.

The intensity of review approach was developed by the courts deliberately to circumvent the high threshold Wednesbury standard.

The low intensity standard of review is based on the courts’ view that parliamentary sovereignty is the ultimate determining factor.

The UK courts seek to ensure that an appropriate separation of powers balance is achieved by considering their constitutional entitlement and institutional competence to review decisions.

A

The UK courts seek to ensure that an appropriate separation of powers balance is achieved by considering their constitutional entitlement and institutional competence to review decisions.

Correct. The court will consider whether the subject matter of the case before it is something it should properly intervene in (constitutional entitlement). This is linked to the question of whether the court has the appropriate knowledge and expertise to review the decision in question (institutional competence). Both considerations reflect the courts’ reluctance to interfere in areas which are better left to executive decision-making and expertise.

incorrect
In distinguishing between matters concerning broader policy questions and those involving fundamental rights, the courts are purely intent on promoting the rule of law.

This is not the best answer. It is possible to argue that, by applying a high intensity of review to cases involving fundamental rights, the courts are promoting the rule of law, but this does not reflect the whole rationale for the sliding scale approach.

117
Q

You are advising a new client who wants to challenge a decision of Renshire County Council (“RCC”) to refuse his application for a music venue licence. The licence scheme is governed by the (fictitious) Local Authority Licensing Act 2018 (“the Act”) which states:

Section 22: In deciding whether to issue a music licence, local authorities must take into account the interests and opinions of residents and businesses who are situated close to the venue.

Your client tells you that he applied for a licence in order to host music concerts at his café, which is located on the High Street in Renshire. In its short refusal decision, RCC stated: “Following careful consideration of your application we have decided not to issue you with a licence. In accordance with the Council’s noise pollution policy, we are prioritising licences for venues that are outside the town centre”. The client had submitted several letters of support with his application from local businesses, residents living in and around the town centre, and musical performers. These letters were not mentioned by RCC.

On the basis of the information you have, which of the following summarises the most accurate advice to your client on whether he can challenge the decision on the ground of illegality?

RCC’s decision suggests that it failed to take into account mandatory considerations, and it may also have fettered its discretion through rigid application of its noise policy at the expense of an individual consideration of the client’s application.

It is likely that RCC has abused its discretion because it failed to take into account the letters of support submitted by the client, which relate to the mandatory factors in section 22.

It is unlikely that the client would succeed in a claim relying upon illegality. RCC is entitled to adopt a relevant policy and it is clear that it discharged its duty under section 22.

Although the client might succeed in a claim regarding RCC’s failure to consider the letters of support, the potential fettering ground has no merit because public bodies are perfectly entitled to adopt relevant policies.

The client will have strong grounds to argue that RCC made an error of law in relation to a misunderstanding of the wording in section 22.

A

RCC’s decision suggests that it failed to take into account mandatory considerations, and it may also have fettered its discretion through rigid application of its noise policy at the expense of an individual consideration of the client’s application.

Correct. On the facts, it looks like RCC did not consider the opinions contained in the letters of support which relate to the mandatory factors in section 22. In addition, we are told that RCC applied a policy which precludes the client from being given a licence; the first issue - the failure to consider the letters - indicates that RCC did not consider individual aspects of the application.

118
Q

Which of the following statements best represents the modern courts’ approach to the Carltona principle?

If government ministers delegate their discretion to a sufficiently senior official within their own department, this will not amount to unlawful delegation of discretion.

Government ministers can delegate their discretion to officials within their own department, or officials within different ministerial departments, so long as the officials are suitably senior, and accountability remains with the minister who has the statutory power.

Government ministers, and some other public office holders, are permitted to delegate their discretion to sufficiently senior officials within their own departments, so long as accountability remains with the minister or the office holder who has the statutory power

Government ministers are able to delegate to officials within their own department so long as they remain accountable.

Government ministers can delegate their discretion to sufficiently senior officials within their own department, so long as accountability remains with the Minister who has the statutory power.

A

Government ministers, and some other public office holders, are permitted to delegate their discretion to sufficiently senior officials within their own departments, so long as accountability remains with the minister or the office holder who has the statutory power

Correct. This accurately represents the principle and its expansion beyond Ministers to other public office holders such as Chief Constables, as per the case of R (Chief Constable of West Midlands Police) v Birmingham Justices, as well as the seniority requirement discussed in cases such as Oladehinde

119
Q

At the annual budget meeting of Brockshire County Council (“BCC”) the Chief Executive reported that savings of 25% would have to be made to the transport budget to reflect a 25% cut in central government funding for regional transport.

Officers on BCC’s Transport and Highways Sub-Committee had been expecting some cuts. They were aware of recent debates in Parliament where the Secretary of State for Transport introduced a new policy to reduce regional transport funding. She said that the policy is intended to contribute to a government strategy to reduce the UK’s national debt. The officers are shocked by the scale of the cuts and have two key concerns. Firstly, they are worried about the impact the cuts will have on local transport provision and safety. Secondly, they have noted that central government funding for transport in neighbouring West Brockshire has been cut by only 5%. BCC has approached your firm for advice.

On the basis of the information you have, which of the following summarises the most accurate advice on whether there is merit in BCC challenging the decision of the Secretary of State for Transport on the ground of unreasonableness?

Because the Secretary of State’s decision concerns central government funding allocations, a court would find that the matter is not justiciable at all.

Whilst the decision to make such an extreme cut to BCC’s regional transport funding might be seen as arbitrary and inconsistent, it is likely that a court would apply a particularly low intensity of review to any claim brought by BCC because of the political and financial context. Consequently, on the facts of this case, a claim is unlikely to succeed.

BCC’s claim is unlikely to succeed. Although there is a cogent case that the Secretary of State made an arbitrary and inconsistent funding decision, the Wednesbury standard is so high that such a decision will never be quashed by the courts.

The Secretary of State’s decision can be seen as both arbitrary and inconsistent, and therefore, BCC will have strong grounds of challenge.

Although the claim concerns high policy, because BCC has concerns over safety relating to the rights of individuals, and there is evidence of inconsistent decision-making, a court is likely to apply a reasonably high intensity of review. There is a good chance the Secretary of State’s decision would be quashed.

A

Whilst the decision to make such an extreme cut to BCC’s regional transport funding might be seen as arbitrary and inconsistent, it is likely that a court would apply a particularly low intensity of review to any claim brought by BCC because of the political and financial context. Consequently, on the facts of this case, a claim is unlikely to succeed.

Correct. Whilst there may be prima facie grounds to argue that the SoS acted unreasonably, it is likely a court would apply a low intensity of review to the claim, meaning that a ‘super-Wednesbury’ standard would apply. This is a high policy matter concerning central government funding allocation linked to deficit reduction. The court is likely to consider that reviewing this type of decision made by a democratically elected official is not within its constitutional remit, and, that it does not have the requisite expertise. Refer to the lead case of Nottinghamshire County Council. Unless there has been a very clear abuse of power, the courts will not intervene in macro-economic or political decisions.

120
Q

You see a client who seeks your advice concerning refusal of her application for an Entrepreneur grant. These grants are governed by the (fictitious) Business and Social Enterprise Act 2019 (“the Act”). The Act states:

Section 15: A grant may be awarded where the applicant can demonstrate that their venture will make a quantifiable difference to the community.

Your client’s application was for funds to support her innovative food bank project, which, in addition to collecting foodstuffs, also seeks to engage local school children in the management of the project. The refusal decision included the following: “You have not shown that your project will make a quantifiable difference to the community. Whilst we have considered the evidence you submitted concerning the alleviation of food poverty in your town, and positive outcomes for the schoolchildren, you have not provided any reports about the financial benefits to your project and the community as a whole”.

On the basis of the information you have, which of the following summarises the most appropriate advice to your client on whether she can challenge the decision on the ground of illegality?

It is likely that the decision-maker has committed an error of law through confining the meaning of ‘quantifiable difference’ to financial benefit. However, the claim is unlikely to succeed because the exception in the case of ex parte Page applies.

The decision-maker might have erred in law through a misunderstanding of section 15 of the Act. The decision suggests that ‘quantifiable’ has been construed too narrowly as only relating to financial benefit. However, it is unlikely that a court would decide to quash the decision, as the provision gives a wide degree of discretion and is capable of bearing a wide range of meanings.

Following the cases of Anisminic and ex parte Page all errors of law have been found to be reviewable and therefore, the claim has a good prospect of success.

The decision-maker has erred in law through its interpretation of section 15 of the Act and therefore, it is highly likely that the decision will be quashed.

As the decision-maker failed to take into account the discretionary factor in section 15, it is likely that the decision will be found to be unlawful.

A

The decision-maker might have erred in law through a misunderstanding of section 15 of the Act. The decision suggests that ‘quantifiable’ has been construed too narrowly as only relating to financial benefit. However, it is unlikely that a court would decide to quash the decision, as the provision gives a wide degree of discretion and is capable of bearing a wide range of meanings.

Correct. Whilst it is arguable that there is an error of law here, the claim may fall foul of the ex parte South Yorkshire Transport principle in that the court could find ‘quantifiable difference to the community’ to be an imprecise phrase capable of bearing a wide range of meanings.

121
Q

Which of the following statements best reflects a summary of the courts’ approach to the duty to act fairly and the level of fairness owed?

The duty to act fairly applies in respect of all judicial decisions and additionally to some administrative-type decisions.

Public law decision-makers have a duty to act fairly with respect to administrative and judicial decisions and the level of that duty will depend upon the character of the decision-making body.

The modern approach of the courts recognises that there is a duty to act fairly in judicial and administrative decision-making, although, in rare circumstances, the duty can be overridden.

Administrative and judicial decision-makers have a general duty to act fairly, which is applied with equal strength across all areas of public law decision-making.

The courts have recognised that there is a general duty to act fairly in administrative and judicial decision-making. The key question, in determining whether that duty has been breached in any given situation, concerns the level of fairness that should apply; this will vary depending on the overall context of the case.

A

The courts have recognised that there is a general duty to act fairly in administrative and judicial decision-making. The key question, in determining whether that duty has been breached in any given situation, concerns the level of fairness that should apply; this will vary depending on the overall context of the case.

Correct
Correct. The existence of a general duty was recognised in Ridge v Baldwin where the court said that the question concerning the court should be the extent of the justice required in any given situation. The significance of the context of the decision and the legal issue in question was discussed in cases such as Durayappah and Lloyd v McMahon.

122
Q

After deciding to build an extension to its council-run leisure centre, Abington City Council (“ACC”) put out the construction work to tender. Two construction companies submitted detailed bids, one of which is an existing client of yours. The Managing Director (“MD”) of your client company tells you that his company’s bid was unsuccessful. He was disappointed because he understood that his company had far greater experience of these kinds of projects than the competitor company, and his company had provided a much lower costing than the competitor company.

A few weeks after the decision was made, the MD was surprised to learn that the competitor company’s MD is the brother of one of the members of the ACC committee which made the decision.

On the basis of the information you have, which of the following statements summarises the most accurate advice on whether there is merit in your client challenging the decision of ACC on the ground of procedural impropriety?

The client has a potentially strong claim based upon indirect bias.

The client has a strong claim based upon direct bias.

The client has an indirect bias claim, but it will not succeed because only one member of the decision-making committee was potentially biased.

It is unlikely that the client would succeed in a claim concerning direct bias or indirect bias, because it is not apparent that the brother on the decision-making committee has a financial interest in the competitor company.

The client has an indirect bias claim, but it will not succeed because there is no evidence that the brother on the decision-making committee said or did anything that proved he was actually biased.

A

The client has a potentially strong claim based upon indirect bias.

Correct. It is likely that a ‘fair-minded and informed observer, having considered the facts, would conclude that there was a real possibility’ that the committee was biased as per the test in Porter v Magill. On the facts, there is a close family relationship between the competitor company MD and one committee member. It is not a mitigating factor that only one committee member was potentially biased, as he would have had influence as a decision-maker.

Refer to cases such as Hannam where three out of ten of the decision-making committee were potentially biased, and ex parte Hook, where the mere presence of a potentially biased person who was not a decision-maker was enough for the court to quash the decision.

123
Q

Your client, a dentist, has sought your advice in relation to a recent decision of the (fictitious) Dental Practice Committee (‘DPC’). One of her patients had made a complaint that she had acted unprofessionally by using unacceptable language in front of her, which the patient had found offensive. The complaint was investigated by the DPC whose powers derive from the (fictitious) Professional Bodies Regulatory Act 1999.

Your client disputed the facts presented by the complainant and asked the DPC for a hearing, including permission for her barrister to question the complainant. The committee chair refused permission, however, and told your client in an email that: “We do not have the time or resources in terms of personnel to have a face to face hearing. We have received the complaint against you, and you are entitled to reply to that in writing. We will then weigh up the respective arguments and notify you of our decision.”

Subsequently, she was very upset to receive the DPC’s decision to suspend her from practice for a 12-month period. Your client is a single parent and tells you that she will not be able to manage if she cannot work for a year. She has been in practice for 15 years following her graduation from dental school.

On the basis of the information you have, which of the following summarises the most accurate advice on whether and why there is merit in your client challenging the decision of the DPC on the ground of procedural impropriety?

On balance it is unlikely that a court would find the process to have been conducted unfairly. Your client would be owed a duty of fairness but, because the case did not involve matters of life and liberty, the more modern emphasis put on fair process in cases involving prisoners and parole conditions would not apply here. Consequently, it would be very difficult to persuade a court that the refusal to allow her an oral hearing, and particularly to deny her use of a barrister, amounted to unfair practice sufficient to justify quashing the overall decision.

Your client’s challenge has very strong prospects of success in relation both to the refusal to call an oral hearing and to allow a barrister to represent her. The level of fairness that would be appropriate for her in this situation would be seen at the very top of the scale, given the serious professional consequences for her. Therefore the court will quash the decision, because it will deem it to have been unfair for her not to have had the opportunity to present her version of events and to try to convince the committee of her good character, experience and bona fides.

Your client’s prospects of being able to challenge the decision to suspend her from practice are unlikely to be high because there is no statutory obligation on the DPC to provide her with the right to have an oral hearing or to employ a barrister to represent her.

It is likely that your client’s challenge will be successful in respect of the failure to call an oral hearing. The level of fairness that would be appropriate for her in this situation can be described as medium-high, given the serious professional consequences for her, and so the court is likely to consider that she should have been given the opportunity to present her version of events and to try to convince the committee of her good character, experience and bona fides. It is a little more debatable whether a court would consider it to have been unfair to have refused her permission to call a barrister, as this was a professional disciplinary rather than a purely legal process.

Your client’s public law challenge to the fairness of the disciplinary process would only have a limited chance of success. She was given the right to respond in writing to all the allegations and so there was no need to spend extra time on a hearing, especially if she was seeking to put the complainant under pressure by employing a barrister.

A

It is likely that your client’s challenge will be successful in respect of the failure to call an oral hearing. The level of fairness that would be appropriate for her in this situation can be described as medium-high, given the serious professional consequences for her, and so the court is likely to consider that she should have been given the opportunity to present her version of events and to try to convince the committee of her good character, experience and bona fides. It is a little more debatable whether a court would consider it to have been unfair to have refused her permission to call a barrister, as this was a professional disciplinary rather than a purely legal process.

Correct. This is the best answer as it stresses the relatively high level of fairness that would be expected in this situation. It also points to the reason why an oral hearing would be appropriate and functionally useful as well, because such a process would have given your client the opportunity to convey her character and her experience to the committee, and enabled her to have maximum opportunity to refute any allegations. See cases such as Smith & West and Osborn. The answer also reflects more caution in respect of the barrister issue – this is more of a moot point, given that the Administrative courts can be wary of making administrative processes too legalistic.

124
Q

Your client, the owner of a local nightclub, has sought your advice in relation to a recent decision of the local authority’s licencing committee to impose a final written warning on him. Several local residents had made complaints that he had kept his premises upon longer into the early morning than permitted under the licence conditions. He is very concerned because, if there are any other infringements, he is at risk of losing his licence completely.

He tells you that the “notice of proceedings” document from the committee, notifying him of the allegations against him, had been emailed to him two weeks beforehand, together with scanned copies of two (anonymised) letters of complaint from local residents.

You look at the relevant Act, governing this situation, which states, at [fictitious] section 13: “Before deciding if enforcement measures can be taken, the subject of the complaint must be given reasonable notice in writing and sent all available evidence against him in order to give him the opportunity to respond.”

Your client, (who had sent a letter in his defence to the committee), believes that the failure to send him written notice through the post could enable him to challenge the decision.

On the basis of the information you have, which of the following summarises the most accurate advice on whether there is merit in your client challenging the committee’s decision on the ground of procedural impropriety and what rationale the court would apply in assessing this?

It is unlikely that your client will succeed on this basis. The failure of the committee to send “notice in writing” appears to be a technical breach of section 13, but it does not seem on face value to have prejudiced him in any significant way

It is likely that your client could successfully argue that the failure to send him evidence collected in writing represented a breach of his common law right to be given notice of the case against him.

Although the committee appears to have breached the procedural requirement in section 13 it is unlikely that Parliament could have intended for its final decision to be quashed because the final consequences for your client – a warning letter – were not particularly serious.

It is arguable that the failure of the committee to observe the procedural rule in section 13 of the Act could result in a court quashing its decision because the statutory obligation is phrased in a mandatory way and statute is the most important form of law.

It is unlikely that your client will succeed on this basis. The failure of the committee to send “notice in writing” appears to be a technical breach of section 13 but it does not seem on face value to have prejudiced him in any significant way. The court is therefore likely to construe from the situation that Parliament could not have intended such a breach to be sufficiently serious to justify the invalidation of the overall decision. It would be prudent, however, to look further into whether the copies of the two anonymised letters represented “all available (relevant) evidence” against him.

A

It is unlikely that your client will succeed on this basis. The failure of the committee to send “notice in writing” appears to be a technical breach of section 13 but it does not seem on face value to have prejudiced him in any significant way. The court is therefore likely to construe from the situation that Parliament could not have intended such a breach to be sufficiently serious to justify the invalidation of the overall decision. It would be prudent, however, to look further into whether the copies of the two anonymised letters represented “all available (relevant) evidence” against him.

Correct
Correct. This best reflects the approach of the modern courts to statutory procedural obligations following the cases of R v Soneji and JN(Cameroon). It would also be wise to check on whether there was any other evidence against your client – maybe of a more damaging nature – that was not disclosed, thereby depriving him of the opportunity to provide some mitigation

125
Q

After being re-elected two years ago, the Minister for the Environment wrote to each local authority in England, in accordance with his party’s election manifesto pledge, assuring them that their concerns regarding the development of green belt sites for new homes were not founded and that the government ‘had no current plans’ to review that decision within the next five years.

Corkminster Borough Council (“CBC”) made this information available to its residents on its official website. It also funded an environmental project to investigate how to better protect its local green belt land.Very recently, however, the Minister for the Environment announced plans for 10,000 new homes to be built on green belt land in the Vale of Cork (which forms part of the CBC area).

On the basis of the information you have, which of the following summarises the most accurate advice on whether CBC would have good prospects of success in challenging the decision of the Minister for the Environment on the ground oflegitimate expectation?

The promise concerns a macro-policy issue and therefore, it is likely that the court will apply a Wednesbury standard of review to the question of whether the legitimate expectation contained in the policy statement was lawfully frustrated. Consequently, it is unlikely that any claim brought by CBC would succeed.

A claim brought by CBC has reasonable prospects of success because it relied upon the promise and decided to fund an environmental project.

Whilst a statement of policy is capable of amounting to a legitimate expectation, CBC’s prospects are adversely affected by two key issues. Firstly, it is debatable whether a statement saying that the government has ‘no current plans’ amounts to a clear and unambiguous promise. Secondly, as this is a macro-policy matter, the court will be reluctant to intervene in any event.

A policy statement is capable of forming a legitimate expectation. However, given that the facts concern a macro-policy issue, the court will be reluctant to intervene. Consequently, the prospects of a successful review are poor.

As the Minister’s letter making the promise was based upon an election manifesto pledge, the court will not treat the promise as having been made by a public authority. Consequently, the expectation cannot be treated as legitimate.

A

Whilst a statement of policy is capable of amounting to a legitimate expectation, CBC’s prospects are adversely affected by two key issues. Firstly, it is debatable whether a statement saying that the government has ‘no current plans’ amounts to a clear and unambiguous promise. Secondly, as this is a macro-policy matter, the court will be reluctant to intervene in any event.

Correct. It is questionable whether the Minister’s policy statement contained in their letter is sufficiently clear. In ex parte MFK Underwriting the court said that promises should be ‘clear, unambiguous and devoid of relevant qualification’. Even if the applicant can demonstrate a legitimate expectation has arisen, it would be unlikely, given the macro-policy nature of the matter, that the court would intervene. In Begbie the court found that, where a promise had policy implications and was made on a ‘macro’ level, review would be restricted to a Wednesbury unreasonableness standard. There was a similar approach in the Re Finucane’s Application case.

126
Q

The committee of the British legion in a provincial town has been informed by the local police that its members are being charged for committing an offence under s. 11(7) of the Public Order Act for failing to give notice of its procession to the local war memorial on Remembrance Day, which took place five days ago. The committee is very surprised about this as they have never contacted the police about this event in any preceding years.

Which one of the following statements best describes the lawfulness or otherwise of the police’s actions under the Public Order Act, and the reasons for this?

The police’s actions are not lawful as the committee will have an exemption from the duty to give the police notice of this type of traditional or customary procession.

The police’s actions are not lawful in this situation as the committee is protected by an exemption for processions relating to the military services.

The police’s actions are not lawful as the committee does not need to inform the police of any event if an equivalent event has happened at any time in the past.

The police’s actions are lawful because all public processions have to be justified by their organisers in advance, in case scarce public resources need to be expended in supervising or controlling them.

The police’s actions are entirely justified as the committee has clearly failed to give the police advance written notice of the procession in accordance with its duty under s. 11(1).

A

incorrect
The police’s actions are lawful because all public processions have to be justified by their organisers in advance, in case scarce public resources need to be expended in supervising or controlling them.

Incorrect. This is the rationale for most public processions but not for all of them, as outlined in the exemptions provision in s. 11(2).

127
Q

Following a particular serious period of political repression in a foreign country, involving serial human rights abuses, a campaign group has begun to organise regular protest marches to the London embassy of the state involved. Large groups of protestors, including refugees from the state, have begun to chant protest songs and to shout abusive slogans at embassy staff who are entering or leaving the building. The situation becomes so heated that embassy staff now come in and out of the compound in embassy cars. There has been no other form of disorder, however.

The police have now responded by only allowing three demonstrators at any one time to congregate outside the embassy, claiming that they are entitled to do this because the protestors’ actions are intimidatory towards embassy staff.

Which one of the following statements best describes the lawfulness or otherwise of the police actions in relation to its powers under the Public Order Act?

The imposition of conditions by the police complies with the spirit of the legislation as the behaviour of the demonstrators is intimidatory but the police are only allowed to impose conditions on the place and duration of assemblies of individuals and not on their maximum size.

The police’s actions are unlawful as they have the effect of restraining the freedom of speech of the demonstrators, which is a fundamental and absolute right they enjoy under both common law and the European Convention on Human Rights.

The police’s actions appear to be unlawful as the effect of the protestors’ allegedly intimidatory behaviour was not sufficiently intense to prevent the staff from leaving and entering the embassy entirely.

The imposition of conditions by the police is lawful as the behaviour of the demonstrators is clearly intimidatory in nature.

The police’s actions are unlawful as they are only allowed to impose conditions on assemblies when there are overriding concerns about the safety of the public at large and not that of individuals who may be targeted for hostility.

A

incorrect
The imposition of conditions by the police complies with the spirit of the legislation as the behaviour of the demonstrators is intimidatory but the police are only allowed to impose conditions on the place and duration of assemblies of individuals and not on their maximum size.

Incorrect. This is not accurate in relation to the nature of the conditions that can be imposed on assemblies under s. 14(1).

128
Q

It has come to the attention of the chief police officer in a large provincial city that a pressure group, campaigning to save sites of Victorian heritage, is planning an occupation of the local botanical gardens. This is after news emerges that the local authority is seeking to demolish the gardens’ Palm House, on the basis that it would cost too much to keep it in the state of repair necessary to allow the public to continue to visit. The pressure group has now confirmed that it will occupy the whole of the town’s park and botanical gardens on the next bank holiday Monday and that it would also congregate in the Palm House, now closed to the public, and occupy it for as long as possible.

The police are contemplating imposing conditions on the proposed demonstration and/or a ban on any form of assembly in the Palm House on the day in question.

Which one of the following provides the best advice to the police in relation to its powers under the Public Order Act?

The police cannot impose restrictions or conditions in advance on planned assemblies but will be able to impose a ban on the proposed occupation of the Palm House as this is situated indoors and so is more manageable and containable as a security measure.

The police have no powers to impose conditions on assemblies and are therefore prevented from imposing even more draconian measures such as outright bans on assemblies.

It will be possible for conditions to be imposed under s. 14A in relation to the place, duration and maximum number of attendees in the park and botanical gardens, as long as the police can show that the group’s activities may give rise to serious disruption to the life of the community. However, it will not be possible under s. 14 for the police to impose a ban on any form of assembly inside the Palm House.

It will be possible for conditions to be imposed under s. 14 in relation to the place, duration and maximum number of attendees in the park and botanical gardens, as long as the police can show that the group’s activities may give rise to serious disruption to the life of the community. However, it will not be possible for the police to impose a ban on any form of assembly inside the Palm House.

It will be possible for the police to impose conditions in relation to the place, duration and maximum number of attendees in the park and botanical gardens, and also to ban any assembly in the Palm House, as long as they can show that these events may give rise to serious disruption to the life of the community.

A

incorrect
It will be possible for conditions to be imposed under s. 14A in relation to the place, duration and maximum number of attendees in the park and botanical gardens, as long as the police can show that the group’s activities may give rise to serious disruption to the life of the community. However, it will not be possible under s. 14 for the police to impose a ban on any form of assembly inside the Palm House.

Incorrect. This is correct in substance but the section numbers of the two provisions in the Public Order Act have been wrongly quoted.

129
Q

A group of seven young men, who were intending to attend an England football match at Wembley, were arrested by police outside the stadium before the match and taken to the local police station. They were identified from information provided by police in their local town who had strong evidence to suggest that they were intending to attack fans of the opposing national team before the match started. They were subsequently released an hour after the end of the match. They were told that this action had been necessary to prevent a breach of the peace.

At the same time, two other men were also arrested in order to prevent a breach of the peace after a police officer heard them shouting abuse in the direction of supporters of the opposing national team.

Which one of the following provides the best combination of advice to the two groups?

The police use of the common law power to arrest to prevent an alleged breach of the peace does not appear to have been lawful in the case of either group of arrested men.

The only basis on which the police are authorised to arrest individuals in this type of situation is if they have reasonable grounds to believe that a specific statutory offence has been committed. Both sets of arrest were therefore unlawful.

The police’s use of the common law power to arrest to prevent a breach of the peace appears to have been entirely legitimate in the case of both groups of arrested men.

It is likely that the arrest of the seven men was a legitimate use of the police’s common law power to prevent a breach of the peace. However, the arrest on the same basis of the two other men does not appear to have been lawful as there seems not to have been an imminent threat of violence.

It is likely that the arrest of the two men was a legitimate use of the police’s common law power to prevent a breach of the peace as they were witnessed shouting abuse at opposing football fans. However, the arrest on the same basis of the seven men does not appear to have been lawful as there does not seem to have been an impending threat of violence.

A

It is likely that the arrest of the seven men was a legitimate use of the police’s common law power to prevent a breach of the peace. However, the arrest on the same basis of the two other men does not appear to have been lawful as there seems not to have been an imminent threat of violence.

130
Q

A trainee journalist is keen to write an article about a march in a large English city, which has been arranged by an organisation protesting against what it sees as inadequate government action to combat climate change. The organisers have given notice to the local police and certain conditions have been imposed including a specific route which is designed to ensure that marchers detour around the central shopping area of the city in order to avoid disruption to the life of the community.

The organisers at the head of the march decide not to obey these instructions, however, and despite loud-hailer warnings from police officers at the scene, march down the central shopping street. The journalist is unable to extricate herself from the large crowd, as it filters through the narrow street, and she is arrested along with a number of others for failing to comply with a condition under section 12 of the Public Order Act.

Which of the following represents the best advice to her?

The journalist can defend herself against the charge by arguing that, even though she knew that she was part of a march which was processing down a forbidden route, the matter was outside her control, as she was unable to extricate herself from the crowd.

The journalist will be guilty of an offence of failing to comply with a condition imposed on a procession by the police, as she was informed along with others that the march was taking an unapproved route.

The journalist can argue that she was the victim of the unlawful police tactic of ‘kettling’ once the march entered the narrow shopping street in the centre of the city, meaning that she had been unjustifiably deprived of her liberty.

The journalist will be guilty of an offence for taking part in what became a trespassory assembly once the march took an unapproved detour.

The journalist can defend herself against the charge by showing that she was not one of the organisers of the march.

A

The journalist can defend herself against the charge by arguing that, even though she knew that she was part of a march which was processing down a forbidden route, the matter was outside her control, as she was unable to extricate herself from the crowd.

Correct. Under s. 12(5) POA this defence would be available to her.

131
Q

ECtHR – judicial principles and techniques

A

positive obligations

margin of appreciation

proportionality

132
Q

positive obligations

A

duty on state to seek to prevent violation of ECHR rights by 3rd parties

133
Q

margin of appreciation

A

ECtHR will afford leeway to national authorities on matters which they are better placed to decide

134
Q

proportionality

A

was it necessary to interfere with rights to the degree carried out?

balance between the individual and collective (legitimate) aim

notably qualified rights (NIADS)

135
Q

HRA – procedural and jurisdictional

A

standing

jurisdiction

time limits defendant authority

136
Q

public authorities

A

core PAs

functional PAs

private bodies

137
Q

core PAs

A

potentially liable in ALL situations

s6(1) HRA – unlawful to act incompatibly with ECHR rights (subject to s6(2) defence)

includes courts and tribunals s6(3)(a)

138
Q

functional PAs

A

s6(3)(b) and s6(5) - Aston Cantlow

liability depends on nature of function/act

context - housing associations, nursing homes…

139
Q

private bodies

A

these are never liable under the HRA

140
Q

statutory defence s6(2) HRA

A

s6(1) –> it is UNLAWFUL for a PA to act INCOMPATIBLY with an ECHR right

= main enforcement mechanism in domestic courts

however, s6(1) does NOT apply if a statutory provision either obliges or allows a PA to act incompatibly

= statutory defence – NB parl sov

141
Q

s3 HRA

A

s3(1) so fas as possible legislation MUST be read and given effect in a way which is compatible with ECHR rights

court power to render legislation compatible with relevant right

DEFEATS statutory defence: remedy

142
Q

s4 HRA

A

s4(2) court MAY declare that a provision is incompatible with relevant right

does not invalidation incompatible law – flags problem for minister (parl) to fix – but not compulsory

statutory defence remains: no remedy

143
Q

s3 vs s4 HRA – which route

A

courts CAN
- interpret a provision even if the language is clear
- adopt a linguistically strained interpretation
- read down language (ie adopt a narrower meaning)
-read in woreds/imply in provision
- DOI: measure of last resort?

BUT court CANNOT
- change substance of provision completely
- make changes which go ‘against the grain’ of legislation
- contradict or delete provisions in the legislation
- make decisions for which the court is not equipped

144
Q

domestic enforcement – HRA

A

is a s6(2) defence engaged?
no: liability under s6(1) = s(8) REMEDY

yes: s3 interpretative obligation – is compatible interpretation possible?

– yes: s6(2) defence fails and PA is liable = liability under s6(1) = s(8) REMEDY

– no: s6(2) defence succeeds and PA is not liable so no remedy

—- s4 DOI?
gov and parl
– can amend offending provision by s10 remedial order
– OR do nothing

145
Q

non qualified HRA rights

A

articels 2, 3, 5 and 6

146
Q

article 2 structure

A

2(1)
everyone’s right to life shall be protected by law. No one shall be deprived of his life intentionally

2(2) – limitations
deprivation of life shall NOT be regarded as inflicted in contravention of this Article when it results from the use of force which is NO MORE THAN ABSOLUTELY NECESSARY

eg in defence of any person from unlawful violence

NOT an absolute right
+ve obligations

147
Q

article 2: scope of duty

A

wider operational application

security planning: managemetnt; “command and control”; training

McCann – ‘Death on the Rock’ case

148
Q

article 2: extensions of duty on state

A

investigatory obligation

proactive; systemic; invovlement of families

Amin case – Feltham YOI

149
Q

article 2: positive obligation

A

to what extent does the state owe a duty to safeguard life?

context
- duty on police to prevent violence by third parties (Osman v UK)
- duty on hospitals to prevent death of patients: Rathbone v Pennnine Care NHS Trust

150
Q

article 3

A

no one shall be subject to torture OR to inhuman or degrading treatment or punishment”

absolute right (no limitations and no derogation from it by state, article 15)

+ve obligations
- removal or extradition of individuals from the UK to countries where they may face torture or inhuman and degrading treatments eg soering

torture needs proof + numerous factors: cumulative

151
Q

article 3 – scope of article

A

torture: aggravated; deliberate and cruel treatment

inhumane and degrading treatment:
- serious and ill-treatment or neglect
- nature and context of the treatment, the manner of its execution, its duration, and its physical and mental effects … its impact on the health of the person involved

152
Q

article 5

A

right to liberty

5(1): deprivation of liberty (arrest)

5(2): reasons for arrest
- Reason for arrest can be indirect ie through nature of questioning

5(3): detention period after arrest

5(4): proceeding to challenge lawfulness of detention

5(5): right to compensation

153
Q

article 5(1) – liberty

A

question + two-stage test

Has there been a deprivation of liberty?
- if so, art 5(1) = engaged
- restriction < deprivation
- Guzzard v Italy

1 – save in the following cases
- 5(1)(c) reasonable suspicion

2 – AND in accordance with a procedure prescribed by law (PBL)

154
Q

article 5(1) – two stage test

A

1 – article 5(1)(c)
“on reasonable suspicion of having committed an offence or when it is reasonably considered to prevent his committing an offence…”

  • Reasonable = tangible not just a hunch

OBJECTIVE standard: fox, Campbell & harley v UK

2 – is arrest PBL?
Sunday Times test
– legal authority
— adequately accessible: ie statute
— formulated with sufficient precision to allow citizen to regulate his conduct

Gillan & Quinton
protection against arbitrary interference by state

155
Q

articel 5(3): detention period after arrest

A

for how long can the police detain a suspect on its own authority before further detention is approved by the (independent) judiciary?

detainees should be: ‘brought promptly before a judge’

Brogan v UK – terrorism related – 4 days 6 hrs – too long

–> McKay v UK: interpreted Bogan guidance as allowing 4 days max

less serious matters or potentially vulnerable detainees (minors) –> lower time limit

156
Q

article 6 – right to a fair trial

A

article 6(1) = over-arching right
- applies to determinations of civil law and criminal law
- ‘FAIR and PUBLIC hearing within a REASONABLE time by an INDEPENDENT and IMPARTIAL TRIBUNAL established by law’

article 6(2): presumption of innocence until proven guilty

article 6(3): set of minimum rights owed to everyone charged with a CRIMINAL offence

157
Q

article 6 – right to a fair trial – process

A

key application of constitutional theory and ideal of the rule of law
= fairness and impartiality in all aspects of the legal process
= ‘access to justice’

article 6(3)(c): most important of the criminal law process rights
= right to legal representation – from early investigation stages onwards, though can be temporarily denied for ‘good reason’ (Murray v UK)

158
Q

article 6 – implied rights

A

the right to silence
– art 6(1) and (2)
– consequences of relying on this - so this isn’t an absolute right
– adverse inferences can be drawn at trial
– not absolute due to adverse inferences

the right against self incrimination
– developed from presumption of innocence
– if a suspect is compelled to confess or give up evidence involuntarily (that is prejudicial)

159
Q

qualified rights HRA

A

articles 8, 9, 10, 11

in accordance with/ prescribed by law (Sunday Times)

+

in pursuit of a legitimate aim (2nd part of article)

+

necessary in a democratic society (proportionate)

160
Q

article 8

A

right to respect for private life (phsical and moral integrity)

right to respect for family life

home (though not the right to a home)

correspondence (all forms of communication)

161
Q

article 8 context

A

personal life
- identity
- dignity with care and in death
- gender
- sexuality
- searches
- surveillance
- personal data

family life
- family settlement
- immigration
- marriage
- civil partnership
- genetics
- reproductive rights

home
- security in one’s home
- prevention of intrusion and disturbance
- monitoring of home
- home as ‘refuge’

correspondence
- all modern forms of communication
- special issues re prisoners
- legal privilege

FAMILY NOW INCLUDES ALL STABLE SOCIAL UNITS

162
Q

article 10 - scope and contexts

A

fundamental right – expression is engaged in many contexts

includes right to receive information too

if the state seeks to restrain it, it has to comply with all 3 parts of ‘qualified rights’ test

written and spoken word

broadcasting

internet

cultural expression: film, plays, art, comedy

journalism and confidentiality of sources

163
Q

article 10 – key features

A

should also provide protection to forms of expression which “offend, shock or disturb” (Handyside v UK)

Nevertheless, some forms of speech – inciting violence and hatred – can be restrained

partly differential approach to FoE that causes cultural/religious offence and that which is politically challenging

Cultural bans more easily upheld than journalists reporting on politicians/public figures

note, too, strong use of margin appreciation (by ECtHR) in this area

164
Q

Indirect horizontal effect

A

Vertical effect: claim against PA by individual

Horizontal effect: individual (eg celebrity) against private media which are not acting as PA so under s6(1) have no liability!
- But court is a PA
- Court acts as an arbiter
- Court has to respect and take into account both parties rights: individual article 8 rights VS media article 10 right

165
Q

misuse of private information

A

broad contextual question depending on nature and circumstances of intrusion

article 10 rights of media will be automatically engaged

is article 8 engaged in this situation?

requires a REP – Reasonable Expectation or Privacy

^no longer a need to show pre-existing relationship of contract/confidence

Only need both parties rights to be engaged under the convention

Then court has to adjudicate between the two competing interests

Usually sell evidence if media’s self-expression right has been engaged

Harder to determine individual article 8 right: reasonable expectation of privacy?

Then balancing exercise

166
Q

the balancing exercise: art 8 v art 10

A

art 8: degree of intrusion
- special circumstances (age or vulnerability)
- prior conduct
- effect on health? Irreparable harm?

art 10
- does the info contribute to public debate?
- nature and form of publication
- importance of information

167
Q

nature of JR

A

review of lawfulness of gov decisions/actions by Admin Court

not a form of appeal

only after exhaustion of any alternatives remedies

168
Q

process of JR

A

amenability to JR

procedural exclusivity

standing

time limits

ouster and limitation clauses

169
Q

amenability to JR?

A

issue = is the decision in question unchallengeable by / amenable to JR?

(what is a public law decision?)

CPR Part 54.1(2)(a)(ii)

‘a claim to review the LAWFULNESS of … a decision, action or failure to act in RELATION TO THE EXERCISE OF A PUBLIC FUNCTION’

Amenability is not about where the power came from but the nature of the function of the power being carried out

Ie functional test

170
Q

amenability to JR – functional test

A

Not asking if body is amenable, rather if the decision amenable therefore bodies which are not inherently amenable in nature may still be amenable due to their functions

But for test: If the body had not existed, would the government have had to create one?

archetypal situation: a public body exercising statutory powers = decisions made will be amenable to JR

what else can give a decision sufficient public for it to be amenable to JR?
– self-regulatory bodies: do they effectively carry out public functions? R v Panel of Takeovers and Mergers ex p Datafin (cf ex p Aga Khan)

– contractual out service providers?
R (A) v Partnerships in Care
On going statutory under pinning?

171
Q

JR – procedural exclusivity

A

is the decision in question only ‘exclusively’ challengeable using the JR procedure?

– O’Reilly v Mackman (1983): case struck out, strict application of “PE rule”

– acknowledgement of mixed cases eg Roy v Kensington and Chelsea FPC

  • flexible influence of the CPR
172
Q

JR – standing

A

Standing = sufficient interest
- Not as restrictive as directly affected under HRA
- But if party is directly affected then it obvs has standing

will a party be given standing to proceed from the permission stage to the main hearing?
senior courts act, s31(3): “the court shall not grant leave [ie permission] unless it considers that the applicant has SUFFICIENT INTEREST in the matter to which the applicant relates”

173
Q

JR – wider public interest challengers

A

‘sufficient interest’ demonstrated by:

  • expertise in issue under challenge
  • demonstrate interest in the matter
  • importance of the rule of law: it is ‘vindicated’ through legal scrutiny of important public law issues

‘concerned citizens’ eg Gina Miller (art 50)

pressure/interest groups: R v Foreign Office ex p World Development Movement

174
Q

JR – time limits

A

very strict time limits

purpose – to protect interest of public bodies

very little leeway allowed

CPR part 54.5(1)
‘a claim must be filed (a) PROMPTLY and (b) in any even NO LATER THAN 3 MONTHS after the grounds to make the claim first arose’

special rules (2013)
- planning 6 weeks max
- procurement 30 days

175
Q

JR – ouster and limitation clauses

A

ouster clauses = purportedly remove the courts’ jurisdiction to review the use of specified powers in an Act

courts see these are undermining the rule of law

if limitation clause = restricted time limit: these are enforced by the courts

Anisminic v Foreign Compensation Commission

R (Privacy International) v Investigatory Powers Tribunal

176
Q

JR – remedies

A

JR-specific remedies
- Quashing order
- Prohibitory order
- Mandatory order

Non JR-specific
- Declaration
- Injunction
- Damages

177
Q

JR – types of illegality (ultra vires)

A

x4

errors of law

errors of fact
– precedent fact
– no evidence
– established fact

abuse of discretion
– relevancy
– improper purpose

‘retention’ of discretion
– fettering discretion
– delegation

178
Q

JR – pure ultra vires

A

public bodies can only take action/ make decisions within the powers they have been given (statutory or prerogative)

= core component of ROL

acting outside authority = unlawful

(unless what they have done is reasonably incidental or consequent upon a power they do have)

179
Q

JR – Errors of law

A

Powers given to public bodies
– must be understood and interpreted properly
– misuse = unlawful

subject to exceptions
1. where an error of law was not decisive to the overall decision
2. where it concerned special or arcane rules
3. where the power concerned is framed in an imprecise or highly discretionary way – courts will not interfere (when imprecise hard for court to say what is right)

180
Q

JR – errors of fact

A

precedent or jurisdictional facts
– extensions of error of law/lack of authority bases of review
– public body mistaken whether actual situation gives it authority (white & collins)

‘no evidence’ facts
- factual assertion without evidence
- if acted on by public body = can be unlawful basis for exercising power

181
Q

Public Order law
Processions
Assemblies
Breach of the peace

A

TO DO

182
Q

JR – errors of established fact

A

error made in assessment of particular situation giving rise to unfairness

existing fact

not fault of claimant (or advisers)

established fact – i.e. capable of objective verification

mistake must have played a MATERIAL part in reasoning of public body

183
Q

JR – abuse of discretion

A

RELEVANCY
- public decision makers must take into account relevant consideration
- must disregard irrelevant considerations - Court decides on relevancy based on statute and circumstances
- Venables v Thompson

IMPROPER PURPOSE
- public decision makers must use statutory powers for the purpose parliament intended them for
- can be express or implied
- Padfield principle

184
Q

JR – retention of discretion

A

FETTERING OF DISCRETION
- decision maker restricts their own discretion
– by not using it all (eg ex FBU)
– or by applying an overly rigid admin policy

UNLAWFUL DELEGATION
- decision maker should not delegate responsibility
– BUT, CARLTONA principle = recognition of modern realities: acceptable for civil servant to take decision in name of minister (s101 Local Govt Act 1972)

185
Q

JR – unreasonableness

A

How the gov has used its powers

More interventionist

Courts are more conscious of their constitutional place and will vary their approach/intensity depending on circumstances

substantive ground of review
+
basis or review
+
intensity of review

186
Q

JR – broad types of unreasonableness

A

1 – material defects in the decision-making process
– wrongly weighing up factors
–irrationality, illogicality

2 – oppressive decisions

3 – decisions that violate constitutional principles

187
Q

JR – unreasonableness – material defects in decision-making process

A

WEIGHING UP FACTORS
- close connection to (crossover with) relevancy in illegality
- correct factors may have been identified but court considers they have been given incorrect weighting

IRRATIONALITY : failure in logical chain of reasoning by decision-maker

188
Q

JR – unreasonableness – oppressive decisions

A

‘unreasonable’ decisions based on the effect or impact of them on affected party

close connection with proportionality principle

ex p Wheeler / Ex p Hook

189
Q

JR – unreasonableness – constitutional principles

A

RULE of LAW
- consistency and certainty in application of the law
- equality before the law

TYPES of UNREASONABLE DECISION
- arbitrary decisions
- inconsistent and unclear

190
Q

JR – unreasonableness – a variable standard of review

A

SUB-WEDNESBURY standard
- high intensity of review
- fundamental rights

SUPER-WEDNESDAY standard
- low intensity review
- social and economic policy

ECHR/EU Law: proportionality-based review

191
Q

JR – procedural impropriety

A

failure to observe procedural STATUTORY rules
+
common law fairness
- general duty to act fairly in public law decision-making processes
- rule against bias

  • Crossover with illegality: failure to observe procedural statutory obligations
  • PI is a hybrid of legality and fairness
192
Q

JR – procedural impropriety – procedural statutory obligations (PSO)

A

Breach of PSO: was it so significant that Parliament would have intended that failure to comply would render the process unlawful? (R v Soneji)

yes: decision/process quashed

no: decision/process undisturbed

193
Q

JR – procedural impropriety – duty to ac fairly

A

duty to act fairly
- ridge v baldwin: applies in all public situations

level of fairness
- varies according to nature of decision/process

content of fairness

194
Q

JR – procedural impropriety – level of fairness

A

Level of fairness: from defendant’s perspective? Subjective
- Eg expectation to have license renewed as a opposed to a new applicant
- Would the lack of X (eg personal meeting) have created a better process?

195
Q

JR – procedural impropriety – content of fairness

A

based on the appropriate level of fairness, did the failure to provide relevant processes ‘rights’ and opportunities justify a quashing of the overall process?

examples of content of fairness
- right of notice of case against you
- right to call witnesses
- right to legal representation
- right to receive reasons for decision against you

196
Q

JR – procedural impropriety – direct bias

A

Direct bias normally applies to judicial proceedings (ie being a judge in one’s own court)

A pecuniary or other direct personal interest in a matter they are determining = automatic disqualification (and decision quashed)

Ex p Pinochet Ugarte (re Lord Hoffman)
- partial extension of principle to judicial situations where connection to a party to the proceedings (eg AI) ??

197
Q

JR – procedural impropriety – indirect bias

A

Made up their mind about a matter before the fact

Porter v Magill
‘… whether the fair-minded and informed observer, having considered the facts, would conclude that there was a real possibility that the public body was biased’

context
- pre-formed opinion or inflexible unauthorised involvement of person in process

198
Q

Police powers under the POA

A
  • Under s.11 POA the organisers of public processions must give advance notice of the event to the police.
  • The police have powers under s. 12 POA to impose conditions upon public processions
  • The police have the power under s. 13 POA to apply to local authorities (other than in London) for an order prohibiting public processions.
  • In London a prohibition order under s. 13 can be made directly by the relevant chief police officer.
  • In relation to public assemblies, the police have the power under s. 14 POA to impose conditions upon them.
  • Following the amendment of the Public Order Act in 1994, the police have powers under s. 14A to seek an order prohibiting ‘trespassory’ assemblies.